Civil Procedure Flashcards

ALA Quizzes, Kaplan Quizzes, Kaplan Qbank, CLS Quizzes

1
Q

A woman sued a man in federal court for intentional infliction of emotional distress after he pulled an inappropriate prank on her at his workplace. The man told his attorney that he bought the snake that he placed on her chair that day at a pet store. He quit his job shortly after the incident. The woman sent interrogatories to the man’s attorney asking whether he had bought a snake on the day of the incident.

Does the attorney have to answer the interrogatories?

(A) Yes, because they involve questions of fact.
(B) Yes, because the man quit his job shortly after the incident.
(C) No, because the statements are protected by the attorney-client privilege.
(D) No, because the statements are protected by the work product doctrine.

A

(A) Yes, because they involve questions of fact.

How well did you know this?
1
Not at all
2
3
4
5
Perfectly
2
Q

One day, a traffic light in a busy intersection malfunctioned because of an electrical problem which could have been prevented if the state had performed reasonable maintenance. A driver was hit by another car because of the confusion created by the malfunctioning traffic light. While the driver’s car was stalled in the middle of the road as a result of the accident, a trucker who had fallen asleep at the wheel crashed into the driver’s car. The driver now seeks to bring suit in federal court against the various parties that caused him injury to his person and damages to his car. Assume that all parties are residents of the state except the driver, who lives in a neighboring state.

Under which of the following circumstances would the federal court have jurisdiction over the case?

A The lawsuit involves diverse litigants in which the amount in controversy is $55,000.
B The lawsuit involves diverse litigants where the amount in controversy is $80,000, which includes property damage and medical expenses against the trucker as well as attorneys’ fees and litigation costs in the amount of $10,000.
C The lawsuit is a negligence action involving diverse litigants, and the driver has incurred property damages of $30,000 and medical expenses of $50,000.
D The lawsuit involves diverse litigants, including the state, the trucker, and the other driver, and the claim against each defendant is $30,000.

A

The correct answer is: The lawsuit is a negligence action involving diverse litigants, and the driver has incurred property damages of $30,000 and medical expenses of $50,000.

Discussion of correct answer: Federal district courts have original subject-matter jurisdiction over matters involving diverse litigants in which the amount in controversy is more than $75,000, exclusive of the costs and expenses associated with trial. A claimant may add together each amount in controversy against a particular defendant to meet the threshold. Thus, a federal district court would have jurisdiction over this action, in which property damages are $30,000 and medical expenses are $50,000, because the aggregate is over $75,000 and the litigants are diverse.

How well did you know this?
1
Not at all
2
3
4
5
Perfectly
3
Q

A man files a defamation claim for $80,000 against a former employee. The man is a citizen of State A and the former employee is a citizen of State A. The action was filed in State A state court. The employee filed a counterclaim for age discrimination based upon federal law.

Can the man have the cased removed to federal court?

A Yes, because the counterclaim is a federal question.
B Yes, because the amount in controversy exceeds $75,000.
C No, because there is not diversity of citizenship between the parties.
D No, because only defendants can remove to federal court.

A

The correct answer is: No, because only defendants can remove to federal court.

Discussion of correct answer: An action brought in state court may be removed by the defendant to federal court if the case originally could have been brought by the plaintiff in federal court [28 U.S.C. Sec. 1441]. Here, the issue is that the plaintiff is trying to remove the case to federal court, and plaintiffs cannot do so.

How well did you know this?
1
Not at all
2
3
4
5
Perfectly
4
Q

A woman, a famous actress, and a man, a noted paparazzo, were in a car accident after the man chased the woman while trying to get a picture with her and a new celebrity boy band. Two weeks later, the man snuck onto the woman’s property to take pictures of a secret wedding between the woman and another famous actor. Finally, two weeks after that, the woman was walking down the street when she saw that the man had sold pictures of her wedding to several companies, who were now using them for their
advertising campaigns in local bus stations. The woman wants to sue the man in federal court for damages caused by the accident, the trespass, and the misuse of her image.

Assuming federal jurisdiction is not an issue, may she do so?

(A) Yes, so long as all of the incidents are part of the same claim or occurrence.
(B) Yes, because a party may join as many claims in a single action as the party has against an opposing party.
(C) No, because the two-week breaks make the actions too attenuated.
(D) No, because the joinder of claims like this would lead to jury confusion, and they should be severed.

A

(B) Yes, because a party may join as many claims in a single action as the party has against an opposing party.

How well did you know this?
1
Not at all
2
3
4
5
Perfectly
5
Q

A woman sued a man in federal court for defamation regarding comments that he made about her when she was running for mayor. The man’s attorney believed there were several problems with the complaint. However, he was unsure about what defenses would be lost if not raised in the initial response. Which potential defense does not have to be raised in the initial response?

(A) Failure to state a claim.
(B) Lack of personal jurisdiction.
(C) Improper venue.
(D) Improper service of process.

A

(A) Failure to state a claim.

How well did you know this?
1
Not at all
2
3
4
5
Perfectly
6
Q

The plaintiff, a man from State A, was injured when the defendant, a citizen of State B, ran a red light in State A, causing $60,000 in damages. After meeting to try and settle the issue without a trial, the plaintiff and defendant began arguing in the parking lot over the cause of the accident. The defendant pushed the plaintiff to the ground and stormed off. The plaintiff sought medical attention after the incident, and believed that he suffered $20,000 in damages from the incident in the parking lot. The plaintiff ultimately filed suit against the defendant for negligence and battery in a State A federal court. The defendant objected, claiming that the federal court did not have jurisdiction to hear the cases.

May the federal court hear the case?

A No, because even though the plaintiff’s damages arose out of the same case or controversy, both claims arose from violations of state law.
B No, because the plaintiff cannot aggregate unrelated claims.
C Yes, because the court has supplemental jurisdiction over the battery action.
D Yes, because the elements for diversity jurisdiction are met.

A

The correct answer is: Yes, because the elements for diversity jurisdiction are met.

Discussion of correct answer: The plaintiff can bring his case in federal court if there is a federal question or diversity of citizenship. Here, there is no federal question involved. However, the plaintiff and the defendant are from different states, and the amount in controversy exceeds $75,000. The amount in controversy can be aggregated even if the claims are unrelated, so long as they are asserted by the same plaintiff against the same defendant. Here, both claims are brought by the same plaintiff against the same defendant. Therefore, aggregation is proper.

How well did you know this?
1
Not at all
2
3
4
5
Perfectly
7
Q

A real estate holding company had several subsidiaries that owned different types of real estate, including one that owned commercial properties in a particular city. The holding company sued several city commercial tenants in federal court over their failure to pay yearly fees under a federal regulatory scheme. The commercial tenants alleged that the fees were included in their previously negotiated commercial rents, but the court found for the holding company. Three months later, the defendants filed a motion under Federal Rule 60(b)(4) for relief from a judgment on the basis that the subsidiary was the proper plaintiff.

Should the court grant the motion?

(A) No, because the defendant did not raise the issue during the suit.
(B) No, because the holding company consented to jurisdiction when it filed the suit.
(C) Yes, because the holding company lacked standing.
(D) Yes, because the motion is timely.

A

(C) Yes, because the holding company lacked standing.

How well did you know this?
1
Not at all
2
3
4
5
Perfectly
8
Q

A visitor from Georgia was drunk after attending a college football game in Florida. As he was driving through Florida on his way home, he ran a red light and hit a pedestrian that was walking across the street. The pedestrian was seriously injured. After spending the night in jail for reckless driving, the distraught visitor returned to Georgia. Several weeks later, after receiving medical bills totaling over $80,000 for extensive physical therapy and medical treatments, the pedestrian filed an action against the visitor in Florida for her injuries.

Will the federal court in Florida have personal jurisdiction over the visitor?

(A) No, because the visitor is not domiciled in Florida.
(B) No, because the visitor was not personally served in Florida.
(C) Yes, because the visitor was present in Florida when the conduct that gave rise to the suit took place.
(D) Yes, because the pedestrian resides in Florida.

A

(C) Yes, because the visitor was present in Florida when the conduct that gave rise to the suit took place.

How well did you know this?
1
Not at all
2
3
4
5
Perfectly
9
Q

While on vacation in Vermont, an Ohio citizen visited an antiques store and saw what was labeled as a packet of Civil War coins. Desperate for a sale, the owner of the store told the Ohio citizen that the coins were rare and worth much more than the $100,000 he was asking for them. However, the woman thought the price was too high.

When the woman returned home to Ohio, she received a call from the store owner reducing the price by 20%, prompting the woman to buy the coins. Once she sent the money to the owner, and after receiving the coins, she discovered that the coins were actually worth only a few hundred dollars. The Ohio woman wished to sue the owner for fraud in Ohio. The store owner argued that the court in Ohio lacked jurisdiction.

Which of the following is true?

A The woman can sue in Ohio, because the owner’s call provided sufficient minimum contacts to support Ohio’s exercise of jurisdiction over him.
B The woman can sue in Ohio, because the amount in controversy is more than $75,000.
C The woman cannot sue in Ohio, because the man is not subject to personal jurisdiction in Ohio based on a single phone call.
D The woman cannot sue in Ohio, because the initial conversation took place in Vermont.

A

The correct answer is: The woman can sue in Ohio, because the owner’s call provided sufficient minimum contacts to support Ohio’s exercise of jurisdiction over him.

Discussion of correct answer: Although the owner did not physically enter the state, he would have sufficient minimum contacts via his directly reaching into the state to make contact with the woman in order to conduct business. Had the sale occurred entirely at the shop, and the owner merely followed up with a call, or the woman had a change of mind and called the owner, those situations would not have provided sufficient minimum contacts. However, the call here was made directly to the woman for the purpose of getting her to make a purchase. That is purposeful availment sufficient to establish personal jurisdiction.

How well did you know this?
1
Not at all
2
3
4
5
Perfectly
10
Q

A man was the victim of an attack at a truck stop in State B by the defendant who was motivated by his hatred of the man’s race. The man, a citizen of State A, filed suit in the U.S. District Court in State A alleging a federal claim against the defendant who was a citizen of State C. The defendant moved to change the venue to State B or, alternatively, to State C.

What outcome is likely?

(A) The action may be transferred to State B only.
(B) The action may be transferred to State B or State C.
(C) The action must be dismissed, because it was commenced in an improper forum.
(D) It is discretionary with the court whether to retain the action or transfer it to State B or State C.

A

(B) The action may be transferred to State B or State C.

How well did you know this?
1
Not at all
2
3
4
5
Perfectly
11
Q

Acme became involved in a labor dispute with the Steamrollers’ Union (the entity that ordinarily supplied workers for Acme’s plant). Acme commenced an action against the Steamrollers’ Union in the appropriate U.S. District Court, claiming $75,000 in damages as a consequence of the union’s conduct in harassing and intimidating non-union workers in violation of the National Labor Relations Act and applicable state law. Acme is an Indiana corporation, and the union (an unincorporated association) has members who are domiciled in every state except New York and New Jersey. The Steamrollers’ Union answered by denying Acme’s allegations and filing a $15,000 counterclaim, which asserted that Acme had deliberately made false accusations about the union to the local papers for the purpose of obtaining favorable press coverage. If the Steamrollers’ Union moves to dismiss for lack of subject-matter jurisdiction, should the court grant the motion?

(A) Yes, because Acme has not claimed monetary damages in excess of $75,000.
(B) Yes, because there is no diversity.
(C) No, because subject-matter jurisdiction is satisfied.
(D) No, because a state claim has been asserted in a federal court.

A

(C) No, because subject-matter jurisdiction is satisfied.

Discussion of correct answer: Because Acme is asserting a federal claim against the Steamrollers’ Union, subject-matter jurisdiction exists, even though the amount in controversy does not exceed $75,000 and without regard to diversity.

How well did you know this?
1
Not at all
2
3
4
5
Perfectly
12
Q

A State A man purchased a gift basket for himself from a State B corporation that shipped gift baskets and other foods through the mail. The man did not like the vanilla coffee that was included in the gift basket and claims that it was a defective product that gave him a stomachache. While he did not go to a hospital for the stomachache, he alleged that he had $300,000 in pain and suffering damages. He filed a product liability claim based in state law against the corporation in State A federal district court and sent a request for waiver of service of process by mail to the corporation. The general counsel of the corporation reviewed the man’s complaint and concluded that there was no court in the country that would award the man damages, much less enough damages to meet the amount in controversy requirement for diversity jurisdiction, and, based on this, determined that the corporation did not need to reply to the man’s request. 75 days after sending the original request and not having received a reply, the man returned to court requesting that the corporation be sanctioned for failing to reply.

Will the court impose sanctions on the corporation?

A No, because the corporation had good cause for failure to waive.
B No, because the corporation’s time to respond was not expired.
C Yes, because the corporation did not have good cause for failure to waive.
D Yes, because a corporation has a duty to waive service.

A

The correct answer is: Yes, because the corporation did not have good cause for failure to waive.

Discussion of correct answer: A court may impose sanctions on a party that fails to respond to a request for waiver, but will not do so when there was “good cause” for failing to respond, i.e., where the party did not receive the waiver. Belief that the court did not have jurisdiction, however, does not qualify as good cause, and therefore the general counsel’s reason for not responding will not prevent the court from imposing sanctions.

How well did you know this?
1
Not at all
2
3
4
5
Perfectly
13
Q

A woman who lived in State A sued a man for battery when she got hurt in a bar fight that he started. She is suing for $40,000 for her injuries. In the same suit, she also claimed $50,000 in punitive damages. She sued in federal court, claiming diversity jurisdiction. The relevant state law barred punitive damages in actions of this type.

Can the woman’s action be heard in federal court?

A No, because the amount of punitive damages that could be awarded is uncertain.
B No, because the claim is legally certain to be less than $75,000.
C Yes, because a federal court might award her over $35,000 in punitive damages, despite the state law.
D Yes, because the combined amount of damages she is claiming in good faith is $90,000.

A

The correct answer is: No, because the claim is legally certain to be less than $75,000.

Discussion of correct answer: The amount in controversy between the litigants must exceed $75,000, exclusive of the costs and expenses associated with trial. A court will dismiss an action based on diversity if it appears “to a legal certainty” that the claim is for less than $75,000. Here, because the relevant state law does not allow punitive damages, it is legally certain that the claim cannot be worth more than $40,000.

How well did you know this?
1
Not at all
2
3
4
5
Perfectly
14
Q

A man sued a corporation after he dined in one of their restaurants and suffered serious food poisoning which required that he be hospitalized for a month. The man, a State A citizen, filed suit in state court in State A seeking $100,000 in damages. The corporation sought removal of the case to federal court, claiming that it was a citizen of State B. The corporation has restaurants in 48 states, including State A. In addition, the corporation maintains its chief training center and the majority of its restaurants in State A. The corporation’s board of directors and all of its top management were based in State B, and the corporation was directed and controlled from there. The corporation will be considered a citizen of what state?

(A) State B, because the corporation is controlled from there.
(B) State B, because the board of directors and all of the top management are there.
(C) Any of the states in which it operates, because it likely has sufficient minimum contacts to face suit in any of them.
(D) State A, because the majority of its business is located in State A.

A

(A) State B, because the corporation is controlled from there.

How well did you know this?
1
Not at all
2
3
4
5
Perfectly
15
Q

Peter lives in Camden, New Jersey and recently went to dinner at Eatery Co., a restaurant in nearby Philadelphia, Pennsylvania. Eatery is incorporated in Pennsylvania and maintains its only place of business in Philadelphia. Eatery has been eager to attract customers from nearby Camden, and has thus placed advertisements on many billboards in the town. Seeing one of these advertisements, Peter decided to give the restaurant a try. Unfortunately, Peter became very sick after the meal, and subsequently brought a federal action in the district of New Jersey seeking damages.
Is the district of New Jersey a permissible venue?

(A) Yes, because Peter resides in the district.
(B) Yes, because Eatery is subject to personal jurisdiction in the district.
(C) No, because Eatery only resides in Pennsylvania.
(D) No, because advertising in a particular venue does not constitute a substantial occurrence.

A

(B) Yes, because Eatery is subject to personal jurisdiction in the district.

How well did you know this?
1
Not at all
2
3
4
5
Perfectly
16
Q

Citizens of State A were plaintiffs in an action brought in the United States District Court in State B against Firstcorp, a State C corporation, and Secondcorp, a State B corporation. Firstcorp’s office and plant were located in State C. At no time had Firstcorp had an office or salesmen in State B. Secondcorp’s sole place of business was in State B. The complaint alleged that each of the plaintiffs sustained serious personal injuries when a toaster oven exploded while a clerk was showing it to the plaintiffs in Secondcorp’s store in State B. Each plaintiff requested damages in the sum of $80,000.

The toaster oven had been manufactured by Firstcorp and shipped to Distributor in State A. Distributor had a contract with Firstcorp to act as the exclusive distributor of Firstcorp products in seven states, including State A and State B. Process was served personally on the president of Firstcorp at Firstcorp’s office in State C and on the president of Secondcorp at its office in State B. Thereafter, Firstcorp moved to dismiss the action on the ground that the court had no jurisdiction over it.

Under what circumstances can the U.S. District Court assert personal jurisdiction over Firstcorp?

(A) If State B has an appropriate long-arm statute and the assertion of personal jurisdiction would comport with due process.
(B) If the assertion of personal jurisdiction would comport with due process (there being nationwide service of process in actions commenced in federal court).
(C) If the amount in controversy exceeds $75,000, exclusive of interest and costs.
(D) If process was served upon Firstcorp in accordance with both federal and State B law.

A

(A) If State B has an appropriate long-arm statute and the assertion of personal jurisdiction would comport with due process.

How well did you know this?
1
Not at all
2
3
4
5
Perfectly
17
Q

A woman was hit by a car while walking her dog. The woman was a citizen of State A. The driver was a citizen of State B. The woman sued the driver in a State A state court, claiming $40,000 in damages based on her personal injuries. After a visit to a doctor revealed the need for more surgeries, the woman amended her complaint to add another $50,000 in damages. The driver filed a motion to remove the case to federal court.

Can the case be removed to federal court?

A Yes, after the woman amended her complaint to add $50,000 in damages.
B Yes, because the defendant made the request for removal.
C No, because the original claim was for $40,000 in damages.
D No, because the woman’s claim is for personal injuries.

A

The correct answer is: Yes, after the woman amended her complaint to add $50,000 in damages.

Discussion of correct answer: Removal to federal court is permissible only where at least one of the claims filed by the plaintiff would fall within the subject-matter jurisdiction of the federal courts. Removal is only available for actions filed in state court. Here, the action was filed in state court. The woman and the driver have complete diversity of citizenship, and once the woman added $50,000, the amount in controversy requirement was met.

How well did you know this?
1
Not at all
2
3
4
5
Perfectly
18
Q

A driver rented a car from a company in State B. The driver was a citizen of State A. The company was incorporated in State C, and had its principal place of business in State B. The victim was a citizen of State B. The driver was driving on a highway in State B when the accelerator pedal got stuck, and he hit the victim, who was also driving on the highway, causing severe injuries to the victim. The victim filed a lawsuit in federal district court in State B against the driver for $80,000 in damages. She served the summons and complaint on the driver three weeks later. In his answer, filed two days after receiving the summons and complaint, the driver cited the stuck accelerator pedal as the proximate cause of the accident. A week after receiving the driver’s answer, the victim amended her complaint to add the company as an additional defendant, seeking to hold the company liable for $80,000 in medical damages, plus $30,000 in damage to her car. Five days later, the victim served the amended complaint on the company and the driver. The company subsequently filed a motion to dismiss the complaint for lack of subject-matter jurisdiction.

Will the federal district court grant the company’s motion to dismiss?

A Yes, because the minimum amount in controversy required for federal court jurisdiction has not been met.
B Yes, because there is no longer diversity of citizenship among the parties.
C No, because diversity of citizenship was only required at the time that the lawsuit was initiated.
D No, because the victim’s amendment to her complaint was not timely.

A

The correct answer is: Yes, because there is no longer diversity of citizenship among the parties.

Discussion of correct answer: 28 U.S.C. Section 1367 allows supplemental jurisdiction over claims arising from the same case or controversy, but such jurisdiction is not available to a plaintiff in a diversity action who adds a party whose joinder destroys that diversity. Here, both the plaintiff and the newly added defendant are both citizens of State B. Because it is the plaintiff who has a claim against the defendant from her own state, the court must dismiss for lack of diversity.

How well did you know this?
1
Not at all
2
3
4
5
Perfectly
19
Q

A woman filed suit against a logger in a State A court claiming that he trespassed on her land and removed many of the trees without her permission. Both the woman and the logger were State A residents. Although it was unclear how many trees had been taken, the woman claimed $1 million in damages. A week later, the logger took a job in State B and moved there, establishing residency with an intent to permanently remain there. The logger then filed a motion to have the suit removed to federal court.

How should the court rule?

(A) Grant the motion, because the man is a citizen of State B.
(B) Grant the motion, because the suit involves natural resources.
(C) Deny the motion, because the amount in controversy requirement may not be met.
(D) Deny the motion, because the woman is a citizen of State A.

A

(D) Deny the motion, because the woman is a citizen of State A.

How well did you know this?
1
Not at all
2
3
4
5
Perfectly
20
Q

A visitor from Georgia was drunk after attending a college football game in Florida. As he was driving through Florida on his way home, he ran a red light and hit a pedestrian that was walking across the street. The pedestrian was seriously injured. After spending the night in jail for reckless driving, the distraught visitor returned to Georgia. Several weeks later, after receiving medical bills totaling over $80,000 for extensive physical therapy and medical treatments, the pedestrian filed an action against the visitor in Florida for her injuries.

Will the federal court in Florida have personal jurisdiction over the visitor?

(A) No, because the visitor is not domiciled in Florida.
(B) No, because the visitor was not personally served in Florida.
(C) Yes, because the visitor was present in Florida when the conduct that gave rise to the suit took place.
(D) Yes, because the pedestrian resides in Florida.

A

(C) Yes, because the visitor was present in Florida when the conduct that gave rise to the suit took place.

How well did you know this?
1
Not at all
2
3
4
5
Perfectly
21
Q

After losing a significant amount of money in the stock market, an investor sued her investment advisor in federal court for not investing her money properly and stated that the failure to invest in companies that earned at least 10% per year interest was fraudulent.

After discovery had concluded, the advisor and the investor each moved for summary judgment. In his motion, the advisor submitted the deposition testimony of an employee of the Securities and Exchange Commission who cited recent government data that suggests most investments earned less than 10% return on investment over the last three years.

How should the court rule?

(A) Deny both motions, because motions for summary judgment are not appropriate in fraud actions.
(B) Deny both motions, because there are genuine issues of material fact present.
(C) Grant the advisor’s motion, because there are no genuine issues of material fact present.
(D) Grant the investor’s motion, because the advisor’s claim for relief was based on hearsay that would otherwise be inadmissible at trial.

A

(B) Deny both motions, because there are genuine issues of material fact present.

How well did you know this?
1
Not at all
2
3
4
5
Perfectly
22
Q

A man bought a cold medicine from a store and had an allergic reaction to it which required him to be hospitalized for ten days. The man, a citizen of State X, brought an action in the United States District Court in State X against the store and the corporation that manufactured the cold medicine. The store was located in State Y and the corporation was a State Z corporation. The plaintiff requested damages in the sum of $80,000. Process was served personally on the owner of the store in State Y and on the president of the corporation at her office in State Z. Thereafter, the corporation filed a cross-claim against the store for $90,000 alleged to be due for merchandise previously sold by the corporation to the store.

How should the court rule on a motion to strike the corporation’s cross-claim against the store?

(A) The court should grant the motion, because the cross-claim is unrelated to the plaintiff’s action.
(B) The court should grant the motion, because there is no supplemental jurisdiction with respect to this action.
(C) The court should deny the motion, because the corporation’s action might confuse or divert the jury with respect to the original claim.
(D) The court should deny the motion, because there is diversity subject-matter jurisdiction with respect to the corporation’s cross-claim against the store.

A

(A) The court should grant the motion, because the cross-claim is unrelated to the plaintiff’s action.

How well did you know this?
1
Not at all
2
3
4
5
Perfectly
23
Q

Acme is a Georgia corporation that has its principal place of business in Alabama. Polly used to live in Alabama and work at Acme’s Alabama plant. However, she was recently terminated from her employment with Acme. She consulted an attorney, who advised her that her termination was wrongful under a law recently enacted by Congress. The lawyer also informed Polly that Georgia had the most liberal procedures for prosecuting this type of action. Polly moved to Georgia, where she is now working as a waitress, and commenced an action in the appropriate state court against Acme for wrongful termination under the applicable U.S. law and a similar Georgia statute. Polly seeks $80,000 in damages. Acme filed a notice of removal in the U.S. District Court in Georgia. Polly then filed a motion for remand back to state court. How should the court rule on Polly’s motion?

(A) The motion should be granted, because there is a lack of diversity.
(B) The motion should be granted, because Polly moved to Georgia only for the purpose of acquiring a more advantageous forum.
(C) The motion should be denied, because one of Polly’s actions is a federal claim.
(D) The motion should be denied, because there is no diversity of citizenship.

A

(C) The motion should be denied, because one of Polly’s actions is a federal claim.

Discussion of correct answer: Because Polly’s claim is based upon a federal claim, Acme may remove the case to the appropriate U.S. District Court. It is irrelevant that Acme is a citizen of Georgia because defendants who are citizens of the forum state are only prohibited from removing the suit if federal jurisdiction would be based on diversity. Here, federal jurisdiction would be based on federal question, not diversity.

How well did you know this?
1
Not at all
2
3
4
5
Perfectly
24
Q

A State A plaintiff sued a State C defendant in federal court in State A for personal injuries that occurred in State B which caused an alleged $100,000 in damages. The plaintiff argued that the defendant was subject to personal jurisdiction in State A because the defendant took the bar exam in State A several years earlier, and took several vacations to State A in the last 10 years. Additionally, on the latest vacation, the defendant rented a car. The defendant argued that the sum total of his time in State A was to take the bar (which he failed and never took again) and to vacation on two occasions.

Is the man subject to personal jurisdiction in State A?

A No, because although the man had several contacts, even in the aggregate, they are insufficient to establish personal jurisdiction over him.
B No, because the man has shown his intent not to remain in State A and has only entered at limited times and with a limited purpose each time.
C Yes, because renting a car is implied consent which subjects him to personal jurisdiction.
D Yes, because the man, by way of his taking the bar exam, has purposely availed himself of the jurisdiction.

A

The correct answer is: No, because although the man had several contacts, even in the aggregate, they are insufficient to establish personal jurisdiction over him.

Discussion of correct answer: While the number of times entered and purpose of each trip are useful in balancing the sufficient nature of contacts, it is much easier to look at the intentionality surrounding taking the bar exam. It is also important to note that the contacts are unrelated to the lawsuit. The defendant could actually have many, many more contacts with the state, but if they are unrelated to the lawsuit, they would still not be enough, because general jurisdiction is only available for natural persons who are residents of the state, or non-natural persons who are “essentially at home” in the state. While the man did take the bar exam in the state, he did not pass and never retook the exam. As such, this contact, even with the later vacations added to it, would not be sufficient for general personal jurisdiction.

How well did you know this?
1
Not at all
2
3
4
5
Perfectly
25
Q

A graduate student was injured when the escalator he was riding on malfunctioned and suddenly stopped. He sued an escalator manufacturing company for his injuries, alleging that the escalator manufacturing company had manufactured the escalator that had malfunctioned. The escalator manufacturing company denied that it had manufactured the faulty escalator and moved for summary judgment. The defendant’s motion was accompanied by an affidavit from a person with knowledge of the facts. In its brief opposing the defendant’s motion, the plaintiff stated that he planned to offer, as his proof that the defendant had manufactured the faulty escalator, the expert testimony of a psychic who had had a vision showing her that the defendant had indeed manufactured the escalator. The plaintiff did not offer any affidavits sworn to by the psychic. Should the court grant the defendant’s motion for summary judgment?

(A) Yes, because the plaintiff’s evidence is implausible.
(B) Yes, because the plaintiff has not presented any affidavits from the psychic.
(C) No, because the psychic’s testimony, while inadmissible at trial, can be used to defeat a motion for summary judgment.
(D) No, because there is an issue of material fact in dispute.

A

(A) Yes, because the plaintiff’s evidence is implausible.

How well did you know this?
1
Not at all
2
3
4
5
Perfectly
26
Q

A corporation specializing in the production of a synthetic, recreational drug hired a law firm, organized as a partnership, to lobby the federal government to continue to keep the drug legal. The law firm failed in its efforts and the federal government passed laws outlawing the sale and production of the drug. The corporation sued the law firm in California state court for malpractice in a complaint based entirely on state law claims, and sought $3 million in damages. The corporation is incorporated in Canada and its principal place of business is in California. The law firm is headquartered in Washington, D.C., and its partners all lived in Washington, D.C., Virginia, and Maryland, except for one partner who lived in California and worked there in a one-man office. Shortly after the corporation’s complaint was filed, but before any response to the complaint was filed, that partner closed down the California office and relocated to Washington, D.C. Soon after, the law firm removed the case to federal court.

If the corporation seeks to remand the case to state court, will it prevail?

A Yes, because the law firm is headquartered in Washington, D.C.
B Yes, because there was no diversity of citizenship at the time the complaint was filed.
C No, because the citizenship of the partnership includes the citizenship of all of its partners.
D No, because the case involves a federal question.

A

The correct answer is: Yes, because there was no diversity of citizenship at the time the complaint was filed.

Discussion of correct answer: For purposes of determining whether there is diversity jurisdiction, a corporation will be considered to be a citizen of the state or foreign country of its incorporation as well as of its principal place of business. A partnership will be considered to be a citizen of all the states in which its partners are citizens. This is determined at the time of the complaint. Thus, because the one partner was domiciled in California at the time of the complaint, there will be no diversity jurisdiction.

How well did you know this?
1
Not at all
2
3
4
5
Perfectly
27
Q

An attorney was hired to defend an energy company accused by a resident of a town serviced by the energy company of violating several federal environmental laws. The town resident brought an action in the appropriate federal court. After the attorney sought discovery of a large volume of documents from the town resident, the town resident’s lawyer advised the attorney that she objected to the discovery requests on the following grounds:
1. The documents sought were privileged.
2. The information was factual in nature and related to the town resident’s claims.
3. The documents sought were not calculated to lead to the discovery of admissible evidence.
4. The evidence sought was obtainable from a less expensive source.
Which, if any, of the foregoing objections might be valid?

(A) 1, 3, and 4.
(B) 1 and 2.
(C) 1 and 3.
(D) 2 and 4.

A

(C) 1 and 3.

How well did you know this?
1
Not at all
2
3
4
5
Perfectly
28
Q

A lawyer has a well-deserved reputation for being willing to win cases by any means necessary. He was involved in a high dollar personal injury case that was brought in the U.S. District Court through diversity jurisdiction. The other side’s attorney was angry at the manner that the lawyer was handling this case and the lawyer was confronted with the following allegations:
1. The lawyer’s examination of a witness was conducted in a manner designed to embarrass the deponent.
2. The lawyer failed to attend a deposition that he had scheduled for another party.
3. Although a party who was deposed by the lawyer was not represented by counsel, the lawyer nevertheless insisted upon carrying out the deposition.
4. The lawyer objected to almost every question asked of his client at a deposition, and all answers were made subject to the lawyer’s objections.
Which of the foregoing assertions could result in sanctions against the lawyer and his client?

(A) Allegations 1 and 2.
(B) Allegations 2, 3, and 4.
(C) Allegations 2 and 4.
(D) Allegation 2 only.

A

(A) Allegations 1 and 2.

How well did you know this?
1
Not at all
2
3
4
5
Perfectly
29
Q

Plaintiff, a citizen of Utah, was involved in an auto accident with Defendant, a citizen of Minnesota. The accident occurred in Utah. Pursuant to the applicable Utah long-arm statute, Plaintiff sued Defendant and Insureco, Defendant’s auto insurance company, in a Utah state court of general jurisdiction for $80,000. Insureco, which was joined with Defendant under an applicable state law, is incorporated in Delaware and has its principal place of business in Utah. Both defendants were served with process on the same day. Twenty-nine days after being served, Insureco and Defendant filed a joint notice of removal to the applicable U.S. District Court. Plaintiff then filed a motion for remand. Should the court grant or deny Plaintiff’s motion?

(A) The court should deny Plaintiff’s motion, because the amount-in- controversy requirement is satisfied.
(B) The court should deny Plaintiff’s motion, because Defendant is not a citizen of Utah.
(C) The court should grant Plaintiff’s motion, because the notice of removal was not filed in a timely fashion.
(D) The court should grant Plaintiff’s motion, because Insureco’s principal place of business is in Utah.

A

(D) The court should grant Plaintiff’s motion, because Insureco’s principal place of business is in Utah.

Discussion of correct answer: A corporation is deemed to be a citizen of the state in which it is incorporated and the jurisdiction in which it has its principal place of business. As a result of this rule, removal would not be possible for two reasons. The first reason is because Insureco’s presence in the dispute destroys complete diversity. The second reason is because defendants sued in their home states cannot remove if the basis for removal would be diversity of citizenship.

How well did you know this?
1
Not at all
2
3
4
5
Perfectly
30
Q

A teenager was out riding his motorcycle. He was wearing dark clothes and visibility was poor that night. A man was driving his car and crashed into the motorcycle causing an accident. Soon after another car driven by a woman hit the first car from behind and caused the first car to further damage the motorcycle. The teenager brought an action against the man in the U.S. District Court in the state in which the teenager was domiciled. The teenager sought to recover $76,000 for damages to his motorcycle. The man filed a counterclaim against the teenager, seeking $1,213 for damages to his car. The man also filed two claims against the woman, one seeking $76,000 in the event that the man was liable to the teenager, and the other seeking $1,213 for the damages caused to his car. The teenager and the man were both citizens of State X; the woman was a citizen of State Y. Judgment was entered for the teenager against the man, but all of the other claims were found to be without merit. The woman filed a lawsuit against the teenager and the man in the U.S. District Court in State X seeking $100,000 for personal injuries suffered in the earlier auto accident. The teenager and the man have each counterclaimed seeking $100,000 for their own personal injuries and have filed similar cross-claims against each other.
Which of the following statements is correct?

(A) The court in the initial action lacked subject-matter jurisdiction.
(B) Apart from any possible subject-matter jurisdiction problems in the prior suit, the woman could have filed any claim arising out of the accident that she had against the teenager in that initial lawsuit.
(C) Apart from any possible subject-matter jurisdiction problems in the prior suit, the teenager and the man may not assert their claims against each other in the present action.
(D) All of the above.

A

(D) All of the above.

How well did you know this?
1
Not at all
2
3
4
5
Perfectly
31
Q

The plaintiff, a citizen of State A, went on a business trip to State B and stayed in a hotel. While there, he received a chemical burn while using the hot tub. He filed a tort suit in State A state court against the hotel and the technician (an independent contractor) who serviced the hot tub and pool. The man alleged a violation of State B law and sought $100,000 in damages. The hotel and the technician are citizens of State B for the purposes of diversity jurisdiction.

Two weeks after the hotel and technician were served with process, the hotel’s lawyer sought to remove the case to federal court, but the technician did not wish to remove the action. Two months later, the plaintiff learned that the technician followed the maintenance instructions as provided by the manufacturer of the hot tub. The plaintiff then joined the manufacturer of the hot tub as a defendant. Several weeks after being served, the manufacturer, a resident of State A, wished to remove the case to federal court and persuaded the technician and hotel to join in a notice of removal. The manufacturer filed a notice of removal exactly four weeks after being served, and the hotel and technician filed a notice that same day joining in the removal.

May the suit be removed under these circumstances?

A No, because a case can only be removed that could have originally been filed in federal court.
B No, because the defendants waited too long to initiate their removal.
C Yes, because the defendants have the right to remove the action within 30 days after a reason for removal become apparent.
D Yes, because the plaintiff added the manufacturer to prevent the case from being removed.

A

The correct answer is: No, because a case can only be removed that could have originally been filed in federal court.

Discussion of correct answer: Removal is proper only when the plaintiff could have originally brought the case in federal court. Here, this case involves a sufficient amount in controversy, but because the plaintiff and the manufacturer are from the same state, there is not complete diversity.

How well did you know this?
1
Not at all
2
3
4
5
Perfectly
32
Q

A man from State B brought suit against his former employer in an appropriate U.S. District Court, claiming $100,000 in damages as a consequence of the employee’s termination in violation of the Civil Rights Act. The man, who is African American, claimed that he was terminated after he brought several safety violations to the attention of state inspectors when the employer’s facilities were being investigated for state health-code violations. The employer operates in States A, C, and D and is incorporated in State B. The man’s salary was $37,000 annually, and he had two years remaining on his employment contract.

If the employer moves to dismiss the case for lack of subject-matter jurisdiction, should the court grant the motion?

A No, because a federal court can hear violations of state employment contract disputes.
B No, because subject-matter jurisdiction is satisfied.
C Yes, because the federal court lacks diversity jurisdiction.
D Yes, because the man’s claim amounts to a breach of contract for termination stemming from whistleblowing to state inspectors.

A

The correct answer is: No, because subject-matter jurisdiction is satisfied.

Discussion of correct answer: Because the man is asserting a federal claim against the employer, subject-matter jurisdiction exists, even though the amount in controversy does not exceed $75,000 and without regard to diversity.

How well did you know this?
1
Not at all
2
3
4
5
Perfectly
33
Q

A prisoner in a large state prison received a credible threat that a group of inmates were going to do him harm. He repeatedly asked the guards for help, but they ignored him. A week later, the prisoner was attacked and savagely beaten by the group of inmates that he had warned the guards about. If the guards had taken action prior to the attack, then the prisoner would not have suffered his injuries. After being released from the prison hospital, the prisoner sued the prison in federal court, claiming a violation of his Eighth Amendment right against cruel and unusual punishment.”

Can the prisoner’s claim be heard in federal court?

A No, because the prisoner is being held in a state prison, and is consequently a ward of the state.
B No, because the prisoner is claiming damages based upon the inactions of state employees.
C Yes, because the prisoner is claiming relief for a violation of his Eighth Amendment rights.
D Yes, because the prisoner’s claim and the state prison’s likely defenses arise under federal laws prohibiting cruel and unusual punishment.

A

The correct answer is: Yes, because the prisoner is claiming relief for a violation of his Eighth Amendment rights.

Discussion of correct answer: Federal district courts have original jurisdiction over all civil actions “arising under the Constitution, laws, and treaties of the United States” [28 U.S.C. Sec. 1331]. Importantly, for a case to arise under federal question jurisdiction, the federal question must appear on a fair reading of a well-pleaded complaint. Here, the prisoner’s complaint states that he is suing for damages related to the prison’s violation of his Eighth Amendment rights, so the federal court has original jurisdiction.

How well did you know this?
1
Not at all
2
3
4
5
Perfectly
34
Q

A man from State B was injured while operating a machine that was manufactured by a State B corporation whose principal place of business was in State A. The man found his home state to be more favorable to his case, and so brought a state-law products liability action against the corporation in State B, claiming damages of $95,000. One week after being served, the corporation removed the case to federal court. The man now seeks to have the case remanded back to state court. How should the court rule?
(A) The case should be remanded, because the amount in controversy in a strict liability case in federal court must exceed $100,000.
(B) The case should be remanded, because a federal court lacks subject-matter jurisdiction to hear the matter.
(C) The case should remain in federal court, because the corporation removed within 30 days of being served.
(D) The case should remain in federal court, because the defendant is permitted to remove a case that could have originally been brought in federal court.

A

The case should be remanded, because a federal court lacks subject-matter jurisdiction to hear the matter.

Discussion of correct answer: The defendant cannot remove the action for two reasons. First, the suit could not have been filed in federal court originally, because there is no federal question and the parties are not completely diverse. Second, even if the parties were diverse, the defendant cannot remove a diversity action to federal court if it is a citizen of the state where the suit was filed. That is the case here, because the corporation is a citizen of both State A and State B. Choice (A) is incorrect, as there is no such rule. Choices (C) and (D) are incorrect because the court would not have subject-matter jurisdiction over the case.

How well did you know this?
1
Not at all
2
3
4
5
Perfectly
35
Q

A man, a citizen of State A, sued a movie company, incorporated in State B and with its primary place of business in State A, in a State A court alleging violations of both the U.S. Copyright Act and the State A Copyright Act. Both claims arose out of the alleged use of a book the man wrote as the basis of a movie without the man’s permission. The man claimed $100,000 in damages. The company filed notice to remove the action to federal court 10 days after the initial pleading.

Can the action be removed to federal court?

A No, because the company is a citizen of State A.
B No, because the man is making a State A law claim in a State A court.
C Yes, because removal is based on federal question jurisdiction.
D Yes, because the company filed notice of removal within 30 days of the initial pleading.

A

The correct answer is: Yes, because removal is based on federal question jurisdiction.

Discussion of correct answer: In an action based on federal-question jurisdiction, the action is removable without regard to the citizenship or residence of the parties. Here, one of the man’s claims is based on federal copyright law, over which the federal court has subject-matter jurisdiction. While the company is also a citizen of State A, the fact that the removal is based on federal question jurisdiction makes citizenship irrelevant.

How well did you know this?
1
Not at all
2
3
4
5
Perfectly
36
Q

Liz, a citizen of California, filed an action in the U.S District Court in Los Angeles against Watchco, a Texas corporation, and Monitinc, a New York corporation, alleging that they were engaged in a conspiracy to fix prices in violation of the Sherman Act, and that her actual damages from the conduct of the defendants was $73,000. Liz further alleged in a second cause of action that Watchco was wrongfully withholding $1,000 that Liz paid to them as a deposit for a computer that was not delivered.

Which of the following statements is true?

A The action should be dismissed by the district court, because the value of the aggregated claims does not meet the amount-in-controversy requirement.
B The district court may exercise supplemental jurisdiction over the second cause of action, but only if it arises from a common nucleus of operative facts with the first cause of action.
C The federal court has subject-matter jurisdiction, because there is complete diversity of citizenship between all plaintiffs and defendants.
D The federal court lacks subject-matter jurisdiction, because Sherman Act violations must be tried in the state court where such violations allegedly occurred.

A

The correct answer is: The district court may exercise supplemental jurisdiction over the second cause of action, but only if it arises from a common nucleus of operative facts with the first cause of action.

Discussion of correct answer: Under the doctrine of supplemental jurisdiction, a U.S. District Court may, if a federal claim exists, permit the plaintiff to append any state claims arising from a “common nucleus of operative facts” with the federal claim [United Mineworkers v. Gibbs, 383 U.S. 715 (1966)].

How well did you know this?
1
Not at all
2
3
4
5
Perfectly
37
Q

A man moved to State B to open up a restaurant. During the first six months the restaurant struggled to attract customers. The man wasn’t sure how long he would stay, but he was willing to close the restaurant and return to State A if business did not improve. The next day he was hit by a driver from State B while walking his dog. He suffered over $100,000 in damages. He sued in federal court, claiming diversity jurisdiction based on the fact the amount in controversy was over $75,000, the man was a citizen of State B, and the car owner that hit him was a citizen of State B.

Can the man sue in federal court based on diversity jurisdiction?

A Yes, because the man is domiciled in State A.
B Yes, because the man intends to return to State A if his restaurant in State B does not succeed.
C No, because the man is domiciled in State B.
D No, because this is a state law claim involving state roads.

A

The correct answer is: No, because the man is domiciled in State B.

Discussion of correct answer: In order for diversity to exist, the dispute must involve citizens of different states. A shared citizenship between any plaintiff and any defendant will destroy diversity jurisdiction. A person will be considered a citizen of his or her state of domicile. Domicile requires the person to have both physical presence in the state and the intent to remain in the state indefinitely. Here, since the man has the intent to remain in the state indefinitely (there is no definite time he will be leaving the state, and in fact he may never leave the state), he is considered domiciled in State B and thus, a citizen of State B.

How well did you know this?
1
Not at all
2
3
4
5
Perfectly
38
Q

A woman sued a man who was a notorious recluse and whose sole passion was writing nonfiction books about recent historical events. The woman sent the man a letter stating that the man had won the World Writing Award, which would be given to him at a banquet in his honor in her state. The man accepted the invitation to attend. When the man arrived in the state, the woman told him that there was no such thing as the World Writing Award and served the man with process.

Does the court in the woman’s state have personal jurisdiction over the man in this case?

A No, because the man was not impossible to reach under normal circumstances.
B No, because there was no World Writing Award.
C Yes, because of the doctrine of quasi in personam jurisdiction.
D Yes, because the man was personally served with process in that state.

A

The correct answer is: No, because there was no World Writing Award.

Discussion of correct answer: A defendant who is served with process while in the forum state is likely to be subject to the court’s personal jurisdiction even if his or her presence in the state is temporary and even if his or her presence is entirely unrelated to the lawsuit. However, if the defendant’s presence in the forum state is solely due to force, fraud, or participation in another judicial proceeding, service may not establish personal jurisdiction. Here, the man’s presence in the state is due to fraud, in that he was lured there with promises of receiving the World Writing Award, which does not exist.

How well did you know this?
1
Not at all
2
3
4
5
Perfectly
39
Q

A State A man was injured while vacationing at a resort located in State B. He was injured while taking surfing lessons with one of the resort’s instructors. The resort advertised in several states on the East Coast, including States A and C. As part of his claim, the man also sued the manufacturer of the surf board he was riding on, who was a citizen of State A. All of the witnesses to the accident were in State B. Because he felt the relevant law was more in his favor in State C, the man filed suit against the resort in a federal court there.

From the options below, what would be the most appropriate action for the State C federal court to take?

A Dismiss the action.
B Defer to the plaintiff’s choice of forum and hear the action in State C.
C Transfer the action to a federal court in State B.
D Hear the action in State C, but follow State B law.

A

The correct answer is: Dismiss the action.

Discussion of correct answer: A court must have both subject-matter jurisdiction over an action and personal jurisdiction over the parties to it. Here, the State C federal district court has no personal jurisdiction over the defendants because none have any minimum contacts with State C sufficient to support the exercise of long-arm jurisdiction. In addition, the State C federal district court does not have subject-matter jurisdiction because there is neither a federal question nor complete diversity of parties. Consequently, the federal court must dismiss the action.

How well did you know this?
1
Not at all
2
3
4
5
Perfectly
40
Q

Paul, a citizen of Idaho, and Greg, a citizen of Oregon, sued Darryl, a citizen of Montana, for breach of contract occurring in Montana. The action was filed in the U.S. District Court in Oregon, and Darryl was improperly served. Upon learning of the suit, Darryl made a successful motion to transfer the case to the U.S. District Court in Montana.

Which of the following is true?

A The federal court had the authority to transfer the action to Montana.
B The federal court in Oregon should have dismissed the action, because venue in the U.S. District Court in Oregon was improper.
C The federal court in Oregon should have denied Darryl’s motion to transfer the case, because he was improperly served.
D The federal court improperly transferred the action to Montana.

A

The correct answer is: The federal court had the authority to transfer the action to Montana.

Discussion of correct answer: The action was not properly commenced in the U.S. District Court in Oregon, because the defendant does not reside in Oregon, and the cause of action did not arise there. However, under the applicable law, the action may be transferred to the U.S. District Court in Montana (where the sole defendant was domiciled).

How well did you know this?
1
Not at all
2
3
4
5
Perfectly
41
Q

A plaintiff and a defendant entered into a contract. The contract stated that any litigation concerning the contract could only be brought in the courts of State A. A dispute about the contract arose. The plaintiff used diversity jurisdiction to bring suit in federal court in its home state, State B, where the state courts refused to enforce forum selection clauses. The defendant filed a motion to transfer the case to the United States District Court for the District of State A because of the forum selection clause. The court denied the motion because it felt obliged in a diversity case to apply the state law in State B that refused to enforce forum selection clauses. Did the court err in denying the motion to transfer on this basis?

(A) Yes, because venue is a procedural issue rather than a substantive issue.
(B) Yes, because a federal procedural statute governs the circumstances under which a federal trial court may transfer venue.
(C) No, because a federal court sitting in diversity must apply state law.
(D) No, because the federal transfer of venue statute does not mention forum selection clauses.

A

(B) Yes, because a federal procedural statute governs the circumstances under which a federal trial court may transfer venue.

How well did you know this?
1
Not at all
2
3
4
5
Perfectly
42
Q

A doctor, who was a citizen of State A, was injured when a student negligently hit him while skiing. The student was a citizen of State B. The accident occurred in State A. Pursuant to the applicable State A long-arm statute, the doctor sued the student in a State A state court of general jurisdiction for $80,000. The doctor also named the student’s insurance company as a defendant. Pursuant to State A law, this type of action was permissible. The insurance company was incorporated in State D and had its principal place of business in State A. The student filed a notice to remove the case to the applicable U.S. District Court.

Is removal proper?

(A) Yes, because the amount-in-controversy requirement is satisfied.
(B) Yes, because the student is not a citizen of State A.
(C) No, because the insurance company did not join in the notice.
(D) No, because the insurance company’s principal place of business is in State A.

A

(D) No, because the insurance company’s principal place of business is in State A.

How well did you know this?
1
Not at all
2
3
4
5
Perfectly
43
Q

A woman who was a State A citizen was injured when her van was hit by an underage bartender from State A. He was illegally working at a bar in State B that was owned by a State B citizen. The woman brought an action in the only U.S. District Court in State A against the owner, alleging negligence and breach of a statutory duty. The owner answered and, 30 days later, he moved for leave to file a third-party claim (i.e., an impleader action) against the bartender based on the bartender’s: (1) unpaid bar bill; and (2) failure to pay for a used car that the owner had sold to him.

Which of the following is true?

(A) The motion will be denied, because the owner’s claim does not assert that the bartender may be liable to the owner on all or part of the woman’s claim.
(B) If the court finds that the owner may properly implead the bartender, the owner will probably be allowed to assert any claim he has against the bartender.
(C) The bartender cannot be made a party, because that would destroy diversity jurisdiction.
(D) The motion will be granted, because it was properly filed.

A

(A) The motion will be denied, because the owner’s claim does not assert that the bartender may be liable to the owner on all or part of the woman’s claim.

How well did you know this?
1
Not at all
2
3
4
5
Perfectly
44
Q

A plaintiff car driver from State A sued a defendant truck driver from State B for $150,000 in damages following a collision. The suit was filed in the United States District Court in State B. The defendant sought to remove to state court in State B, where the judges were well known to him.

Can the defendant remove to state court?

(A) No, because there is no federal statute that allows removal from federal court to state court.
(B) No, because the defendant is in the “home state” of the state that he wishes to remove to.
(C) Yes, because removal must be unanimous, and there is only one defendant.
(D) Yes, because the suit is based in diversity.

A

(A) No, because there is no federal statute that allows removal from federal court to state court.

How well did you know this?
1
Not at all
2
3
4
5
Perfectly
45
Q

An elderly couple, through a Washington State travel agent, purchased passage for a seven-day cruise. The couple paid the fare to the agent, who forwarded the payment to cruise line’s headquarters in Miami, Florida, where the cruise line was incorporated and conducted all of its business operations. The cruise line then prepared and sent the tickets to the couple. The back of the tickets stated, “It is agreed by the passenger and carrier that all disputes in connection with this contract shall be litigated before a court in the State of Florida.” The couple boarded the ship in Los Angeles, California. While the ship was in international waters off the Mexican coast, the couple was injured when a faulty light fixture fell on their bed. The couple sued the cruise line in Washington, claiming that their injuries were caused by negligence. The cruise line moved to dismiss, arguing it was not subject to the Washington court’s jurisdiction.

How should the court rule?

A Grant the motion, based on the forum selection clause.
B Grant the motion, because the suit was based on a contract formed in Florida.
C Grant the motion, because the cruise line lacked sufficient contacts with Washington.
D Deny the motion.

A

The correct answer is: Grant the motion, based on the forum selection clause.

Discussion of correct answer: If a contract is otherwise valid and enforceable, a person may consent to personal jurisdiction within a state. Here, the contract made between the couple and the cruise line stated, “All disputes in connection with this contract shall be litigated before a court in the State of Florida.” Consequently, Florida, not Washington, had jurisdiction over this claim [Carnival Cruise Lines Inc. v. Shute, 499 U.S. 585 (1991)].

How well did you know this?
1
Not at all
2
3
4
5
Perfectly
46
Q

An architect who is a citizen of State B has a dispute on a $2,000,000 contract with a client who is a citizen of State A. The client wants to file in federal district court in State A. When he finds out from the architect’s assistant that the architect is about to take a vacation to State A to attend a family reunion, the client arranges for a professional process server to serve the architect while she is at her family reunion.

Assuming that the contract does not specify where a contract claim is to be heard, will a federal district court in State A have jurisdiction over the architect?

(A) Yes, because the architect’s trip to State A means that she has substantial contacts with the state.
(B) Yes, because the architect was served with process while in the forum state.
(C) No, because the architect does not intend to remain in State A indefinitely.
(D) No, because the architect was not a State A resident at the time of service.

A

(B) Yes, because the architect was served with process while in the forum state.

How well did you know this?
1
Not at all
2
3
4
5
Perfectly
47
Q

A tourist, who was a citizen of State A, brought a products liability action in federal court against a supermarket that sold him a contaminated box of cereal. The tourist became ill and had to spend 10 days in the hospital. The supermarket was a State B corporation. The supermarket impleaded the manufacturer of the cereal, alleging that it would have a right to indemnity from the manufacturer as a consequence of the latter’s improper manufacture of the cereal in question. The manufacturer was a State A corporation. The tourist was granted leave to file an amended complaint alleging negligent manufacture against the manufacturer. Thereafter, the manufacturer moved to dismiss the tourist’s action against it for lack of subject-matter jurisdiction.

Which of the following is true?

A The motion should be denied, under the supplemental jurisdiction doctrine.
B The motion should be denied, under the pendent jurisdiction doctrine.
C The motion should be granted, because diversity is lacking.
D The motion should be denied, because there is diversity between the tourist and the supermarket.

A

The correct answer is: The motion should be granted, because diversity is lacking.

Discussion of correct answer: Diversity jurisdiction requires complete diversity as between all plaintiffs and all defendants. Since the tourist and the manufacturer are both citizens of State A, with the addition of the manufacturer as a defendant and the amendment of the complaint to allow plaintiff to bring a claim directly against this same-state defendant, complete diversity was destroyed. 28 U.S.C Section 1367 does not allow supplemental jurisdiction over claims by the plaintiff in a diversity action where allowing those claims would destroy diversity.

How well did you know this?
1
Not at all
2
3
4
5
Perfectly
48
Q

A witness was on vacation visiting her family in Alabama. While there, she saw a crane collapse and injure several people. Months later, when she was back at her home in Florida, she received a summons to testify about the accident in court in Alabama. Although she hadn’t planned to see her family again so soon, she decided to turn the court appearance into another trip to see her family. Consequently, she testified in Alabama court on a Friday, then spent the rest of the weekend to visit and attend a family reunion. At the family reunion on Sunday, the witness got into a fight with her mother and vowed never to set foot in the state again. On Monday, as the witness was returning to the airport, a man approached her and served her with a summons and complaint from her estranged cousin. The witness read over the complaint and realized that it claimed Alabama had jurisdiction over the matter. The witness told the man, “I vowed never to come back to Alabama!” The man told her, “That’s too bad. Since I served you here, the court has jurisdiction over you. You should have gotten on the plane quicker.” Is the witness required to appear in Alabama court because she was served with process while in the state?

(A) Yes, because she was physically served in Alabama.
(B) Yes, because she was in Alabama on Monday on personal business.
(C) No, because she was in Alabama as the result of her participation in another judicial proceeding.
(D) No, because she did not have the necessary minimum contacts with Alabama.

A

(C) No, because she was in Alabama as the result of her participation in another judicial proceeding.

How well did you know this?
1
Not at all
2
3
4
5
Perfectly
49
Q

Several young journalists worked together to investigate rumors of illegal activities at a large corporation’s operating facilities in State X. The journalists, who all resided thousands of miles away in State A, conducted most of their reporting from State A as they had limited funds to travel to and spend time in State X. They conducted interviews by phone and researched over the Internet. Once the journalists had enough evidence to continue their investigations, they traveled by car to State X and entered the corporation’s facilities. They were arrested for trespassing but the local prosecutor dropped the charges. On their way back to State A, they traveled through State Y and were served with process by the corporation, which had filed a tort case based on the trespassing incident against the journalists in a federal district court in State Y. The court determined that it had personal jurisdiction over the journalists as well as subject-matter jurisdiction over the case. The journalists argued that the case should be dismissed on the grounds of forum non conveniens, and have established that a suitable alternative forum exists in State A where they live.

Which of the following factors is the court least likely to consider in determining whether the case should be dismissed on these grounds?

A The corporation’s activities and the journalists’ efforts to report on them have little to no impact on the community representing the jury pool in State Y.
B It would be extremely burdensome on the journalists to be away from their work and have to pay for lodging during a trial in State Y.
C The jury pool for the State Y federal court is far more biased in favor of corporations than a jury pool would be in State A.
D The journalists would not be able to afford counsel in State Y and would be relying on the court to appoint them counsel.

A

The correct answer is: The jury pool for the State Y federal court is far more biased in favor of corporations than a jury pool would be in State A.

Discussion of correct answer: When analyzing whether a case should be dismissed under the doctrine of forum non conveniens, a court will balance a number of private and public interests, which generally go to the inconvenience and expense that would be incurred by the parties, the court, and the community in conducting a trial in the forum. Whether a jury might be biased is not one of the factors generally considered in a forum non conveniens analysis.

How well did you know this?
1
Not at all
2
3
4
5
Perfectly
50
Q

A teacher, who was a citizen of State A, filed an action in the U.S District Court in State A against a stockbroker, a State B citizen, and his firm, a State C corporation, alleging that they violated SEC federal regulations and perpetrated a fraud during the sale of securities to the teacher. The teacher claimed that his total damages from the fraud were $66,000. The teacher further alleged in a second cause of action that the stockbroker did not repay an $8,000 loan that the teacher made to him.

Which of the following is true?

(A) The action should be dismissed by the District Court, because the value of the aggregated claims does not meet the amount-in-controversy requirement.
(B) The District Court may exercise supplemental jurisdiction over the second cause of action, but only if it arises from a common nucleus of operative facts with the first cause of action.
(C) The federal court has subject matter jurisdiction, because there is complete diversity of citizenship between all plaintiffs and defendants.
(D) The federal court lacks subject matter jurisdiction, because the violation of SEC federal regulations must be tried in the state court where such violations allegedly occurred.

A

(B) The District Court may exercise supplemental jurisdiction over the second cause of action, but only if it arises from a common nucleus of operative facts with the first cause of action.

How well did you know this?
1
Not at all
2
3
4
5
Perfectly
51
Q

An athlete from State A was drafted out of high school to play for a professional sports team in his hometown and signed to a five-year contract. During that time he became both a local hero and a national celebrity. At the end of his contract, the athlete negotiated with another team in State B to play for them at triple the salary he was earning in State A. In a press conference, the athlete announced he was going to go play in State B and that he was very happy to be leaving State A. The citizens of State B were overjoyed, while the citizens of State A were heartbroken and angry. Soon, however, the owner of the State B team realized he did not have enough money to pay the athlete the agreed upon salary and sought to have the contract rescinded by filing suit in federal court against the athlete on the grounds that the athlete had violated the terms of the contract. The owner filed the action in State A federal district court, believing the court and jury would be more receptive to his arguments. Soon after the complaint was filed, the athlete moved to State B, partially on account of the fact that the vast majority of people in State A had come to despise him, and sought to transfer the case to federal district court in State B.

Can the athlete successfully transfer the case to State B federal district court?

A Yes, if he can show that the State B court is an adequate forum, and that the State A court was significantly inappropriate.
B Yes, because he is now a resident of State B.
C No, because he was a resident of State A at the time the action was filed.
D No, because the case was brought in a proper venue.

A

The correct answer is: Yes, if he can show that the State B court is an adequate forum, and that the State A court was significantly inappropriate.

Discussion of correct answer: A federal court may transfer an action even where it was brought in an appropriate forum, but the burden is on the defendant to show that an adequate alternative forum exists and that the original forum was significantly inappropriate. If the athlete can show that here, then the action can be transferred to State B.

How well did you know this?
1
Not at all
2
3
4
5
Perfectly
52
Q

A man is a citizen of State A and was hospitalized when a car that was manufactured by a company crashed after its steering wheel did not work while the man was driving the car on an interstate highway. The company was incorporated in State B, and had its principal place of business in State C. The man filed suit against the company in federal court on June 15, claiming $100,000 in damages. On July 1, the man moved to State C, intending to stay there permanently.

Does the federal court have jurisdiction over the suit?

A Yes, because the man was a State A citizen when he filed the lawsuit.
B Yes, because the accident occurred on an interstate highway.
C No, because the man moved to State C before his claim was heard by the federal court.
D No, because the man moved to State C within 30 days of filing his lawsuit in federal court.

A

The correct answer is: Yes, because the man was a State A citizen when he filed the lawsuit.

Discussion of correct answer: In order for a federal court to have diversity jurisdiction over a claim, the dispute must involve citizens of different states. A shared citizenship between any plaintiff and any defendant will destroy diversity jurisdiction. However, diversity must be met only at the time the suit is filed. Consequently, even though the man moved to State C and became a citizen of State C, the federal court will retain jurisdiction.

How well did you know this?
1
Not at all
2
3
4
5
Perfectly
53
Q

A man sued a company that owned a national television talk show in State A federal court claiming the company violated his federal copyright rights by using one of his songs as the theme song for the show. The man was a citizen of State B, while the company was incorporated in State C and had its main office in State A. However, the company did have an office in State A where many decisions involving the show were made. The man claimed $60,000 in actual damages, and $100,000 in “reputational harm.” A recent State A Supreme Court decision found that the maximum amount of damages that could be claimed for “reputational harm” were $5,000, although the federal courts had established no such limitation.

Can the action be heard in federal court?

(A) Yes, because the amount in controversy is over $75,000.
(B) Yes, because the man’s claim is based on federal copyright.
(C) No, because the Erie doctrine would limit possible damages to $65,000.
(D) No, because there is no diversity of citizenship between the parties.

A

(B) Yes, because the man’s claim is based on federal copyright.

How well did you know this?
1
Not at all
2
3
4
5
Perfectly
54
Q

A brother and sister lived in separate parts of the same large house that they had inherited from their parents. The brother and sister were both published poets. The brother found out one day that the sister had been traveling the country, speaking at universities and literary societies where she had been reading from and selling copies of the brother’s poems, but indicating they were hers. The brother filed suit against the sister in federal district court, alleging federal copyright violations. The brother was very angry with the sister, but he did meet with her at a local coffee shop and handed her a summons and copy of the complaint. This was the first the sister had heard of the suit, and she apologized profusely to the brother and asked him to drop the case, but he refused. She filed a motion to dismiss with the single argument that the brother had failed to state a claim upon which relief could be granted.

If the sister later files an answer containing the objection that there was insufficient service of process, will she prevail?

(A) Yes, because she was not served with process while at her residence.
(B) Yes, because service of process may not be effected by a person who is a party to the suit.
(C) No, because she clearly indicated that she was aware of the suit.
(D) No, because she failed to include the objection in the motion to dismiss.

A

(D) No, because she failed to include the objection in the motion to dismiss.

How well did you know this?
1
Not at all
2
3
4
5
Perfectly
55
Q

A man, a citizen of State A, bought a life insurance policy from an insurance company located in State C. The company only sold policies to customers in State A, State C, State D, and State E and had no customers or business interests in any other states.

Several years later, the man moved to State B. The man retained the policy, and the insurance company continued to accept premiums and keep the policy in effect although it had no other customers in State B. The man then sued the company in State B court for fraud and other violations regarding his policy.

Does the State B court have personal jurisdiction over the company?

(A) Yes, because the policy remained in effect, and so sufficient minimum contacts exist.
(B) Yes, because the company consented to jurisdiction in State B by accepting premium payments.
(C) No, because the company did not have sufficient minimum contacts with State B.
(D) No, because it was not reasonably foreseeable that the company would be sued in State B.

A

(A) Yes, because the policy remained in effect, and so sufficient minimum contacts exist.

How well did you know this?
1
Not at all
2
3
4
5
Perfectly
56
Q

The Drug Enforcement Administration (DEA), a federal agency, seized a warehouse that a criminal used to store illegal drugs. The DEA gave notice to the criminal by certified mail and sold the warehouse to a developer who was going to use the warehouse to open up the biggest dance club in the city. The criminal filed a state law claim in state court to quiet title against the developer, claiming that the developer’s title was invalid because federal law required the DEA to give the criminal notice by personal service. The relevant statute expressly gave plaintiffs a right to sue for damages.

Can the developer remove the action to federal court?

A Yes, because the DEA is a federal agency and closely regulated by the federal government and its courts.
B Yes, because the criminal could only reclaim the property by proving the notice was insufficient under federal law.
C No, because issues of property are matters reserved to the states.
D No, because the criminal’s claim is a state law claim to quiet title.

A

The correct answer is: Yes, because the criminal could only reclaim the property by proving the notice was insufficient under federal law.

Discussion of correct answer: A state question that involves a question of federal law may be sufficient to establish jurisdiction in a federal court, provided that the federal law’s impact on the state question is substantial. Here, since it is necessary for the criminal’s case to prove the embedded federal question of whether the DEA followed federal law in giving notice to the criminal, the federal question is substantial and the developer can remove the action to federal court.

How well did you know this?
1
Not at all
2
3
4
5
Perfectly
57
Q

The plaintiff is an Alabama corporation that markets computer products of the defendant, a nationwide computer manufacturer with its headquarters in New York. The marketing agreement contained a forum-selection clause providing that any disputes arising out of the contract could only be brought in a court located in the Southern District of New York. After business relations took a turn for the worse, the plaintiff sued the defendant in the Northern District of Alabama. The complaint alleged that the defendant had breached the dealership agreement, causing more than $1 million in damages. The defendant moved the district court to transfer the case to the Southern District of New York. The Alabama court refused to do so, finding that transfer was controlled by Alabama law and that Alabama did not view contractual forum-selection clauses as enforceable. Specifically, the court found that Alabama disfavored such clauses in order to protect state court jurisdiction.

Was the Alabama court’s ruling correct?

A Yes, because the Alabama court based its ruling on Alabama law.
B Yes, because such forum-selection clauses are invalid.
C No, because the clause was evidence of what the parties believed was convenient.
D No, because the defendant was a resident of New York.

A

The correct answer is: No, because the clause was evidence of what the parties believed was convenient.

Discussion of correct answer: Venue is considered appropriate where either party consents to the venue. Importantly, contractual forum-selection clauses are evidence of what the parties believed would be a convenient forum. Here, the contract that served as the basis of the suit had a forum-selection clause stating that all claims had to be brought in the Southern District of New York. While the clause was not dispositive, the court would take it into account with other factors regarding appropriate transfer, such as the convenience of witnesses, ease of access to evidence, the location of the claim, and local interest of the community.

How well did you know this?
1
Not at all
2
3
4
5
Perfectly
58
Q

An accountant sued a flight attendant and an airline company in federal court, alleging damages in the amount of $100,000 arising out of the negligence of both defendants. At the time the accountant filed the lawsuit, she was a citizen of State A, while both the airline company and the flight attendant were citizens of State B. Sometime after the suit was filed, the flight attendant became a citizen of State A. The airline company moved to dismiss the suit, arguing that the court lacked jurisdiction.

Will the airline company’s motion be successful?

(A) Yes, because at the time the lawsuit was filed, it involved neither diversity of citizenship nor a federal question.
(B) Yes, because the flight attendant’s new citizenship destroyed the court’s diversity jurisdiction.
(C) No, because at the time the lawsuit was filed, there was diversity of citizenship among the parties.
(D) No, because the accountant has alleged a tort.

A

(C) No, because at the time the lawsuit was filed, there was diversity of citizenship among the parties.

How well did you know this?
1
Not at all
2
3
4
5
Perfectly
59
Q

A cellist accidentally stepped on a violinist’s valuable violin, damaging the violin beyond repair. The violinist sued the cellist in federal court for negligence, alleging damages worth $120,000. The cellist filed a counterclaim against the violinist for federal trademark infringement, alleging that the violinist had infringed a trademark the cellist owned for a line of music-themed tote bags, causing damages in the amount of $50,000. The violinist is a resident of State A, while the cellist is a resident of State B. The violinist moved to dismiss the counterclaim, arguing that the court lacked jurisdiction to hear it.

Should the court allow the counterclaim?

(A) No, because the counterclaim does not arise out of the same facts and circumstances as the original claim.
(B) No, because the initial claim qualifies for jurisdiction on the basis of diversity jurisdiction, whereas the counterclaim would not qualify for jurisdiction on this basis.
(C) Yes, because the plaintiff and defendant are from different states.
(D) Yes, because the counterclaim involves a question of federal law.

A

(D) Yes, because the counterclaim involves a question of federal law.

How well did you know this?
1
Not at all
2
3
4
5
Perfectly
60
Q

A driver, a citizen of State A, traveled to State B. While in State B, the driver rented an RV from a rental company incorporated and doing business in State B. The driver drove the RV in State B and State C. While in State C, the accelerator on the RV stuck, and the driver got into a single vehicle accident, sustaining major injuries and totaling the RV. The driver was taken to a hospital in State C. When the driver got out of the hospital, he sued the company in federal district court in State B for renting a faulty RV to him, and asked for $300,000 in medical expenses and pain and suffering. The company counterclaimed for $70,000, the cost of the RV that was totaled in the accident. The driver filed a motion to dismiss the company’s counterclaim.

Will the federal district court grant the driver’s motion to dismiss?

(A) Yes, because the counterclaim is permissive, and therefore the court does not have jurisdiction over the company’s claim against the driver.
(B) Yes, because the claim arises out of an occurrence that happened in State C, so the case was filed in the incorrect venue.
(C) No, because the counterclaim is compulsory, and therefore the court has supplemental jurisdiction over the company’s claim against the driver.
(D) No, because the driver is a citizen of State A who chose to bring the lawsuit in federal district court located in State B, and therefore the company, a citizen of State B, may bring its claim for the cost of the RV.

A

(C) No, because the counterclaim is compulsory, and therefore the court has supplemental jurisdiction over the company’s claim against the driver.

How well did you know this?
1
Not at all
2
3
4
5
Perfectly
61
Q

A lawyer in Nevada contracted with a surgeon to perform over $80,000 worth of plastic surgery. When the lawyer failed to pay, the surgeon sued. However, because the surgeon moved to California after the service was completed, he filed the action in California. The lawyer’s contacts with California are minimal, and the surgeon is unsure whether they will suffice for personal jurisdiction. On a previous occasion, the lawyer drove through California to attend a conference in the state capitol, and he paid for a hotel room and other expenses there. The lawyer had also considered moving there at one time, and he had submitted his resume online to some employers in California.

Did the lawyer implicitly consent to personal jurisdiction in California?

(A) Yes, because he operated a motor vehicle in California.
(B) Yes, because applying for employment within California showed his intent to be domiciled there.
(C) No, because his contacts with California were minimal.
(D) No, because this suit does not arise out of his contacts with California.

A

(D) No, because this suit does not arise out of his contacts with California.

How well did you know this?
1
Not at all
2
3
4
5
Perfectly
62
Q

The plaintiff sued the defendant in federal court and requested waiver of service of process. The defendant waived service of process and filed an answer 40 days after the plaintiff sent the request for waiver of service of process. In the answer, the defendant asserted a defense of lack of venue.

Has the defendant waived the defense of lack of venue?

(A) Yes, because the defendant did not file an answer within twenty-one days of receipt of the request for waiver of service.
(B) Yes, because the defendant did not raise the defense of lack of venue in a motion to dismiss.
(C) No, because the lack of venue defense was raised in a proper and timely manner.
(D) No, because the lack of venue defense can never be waived.

A

(B) Yes, because the defendant did not raise the defense of lack of venue in a motion to dismiss.

How well did you know this?
1
Not at all
2
3
4
5
Perfectly
63
Q

A Hawaiian plaintiff sued a California defendant for personal injuries which resulted in $100,000 in damages after the defendant negligently pushed her down some stairs while in Hawaii.

The defendant argued that he was not subject to personal jurisdiction in Hawaii, because the sum total of his time in Hawaii was to take the bar (in which he immediately went inactive and only paid yearly dues), to vacation there on two occasions (which were paid for by his company, as he was awarded attorney of the year both of those years), and to rent a car while he was on vacation.

Is the defendant subject to personal jurisdiction in Hawaii?

A No, because although the defendant had several contacts, even in the aggregate, they are insufficient to establish personal jurisdiction over him.
B No, because the defendant has shown his intent not to remain in Hawaii and has only entered at limited times and with a limited purpose each time.
C Yes, because the cause of action arose out of the defendant’s contacts with the jurisdiction.
D Yes, because entering a contract with a hotel for lodging and with a rental car company is implied consent which subjects him to personal jurisdiction.

A

The correct answer is: Yes, because the cause of action arose out of the defendant’s contacts with the jurisdiction.

Discussion of correct answer: If a defendant harms a plaintiff in the forum state, and the lawsuit is about that harm, that satisfies the constitutional standard of minimum contacts for specific personal jurisdiction, without any other contacts needed. Because the defendant injured the plaintiff while in Hawaii, Hawaii will have personal jurisdiction over the defendant for this specific cause of action.

How well did you know this?
1
Not at all
2
3
4
5
Perfectly
64
Q

A State X athlete traveled to State Y to play with his soccer team and was injured when the elevator in the hotel he was staying at closed on his foot as he tried to hold the door open for a pregnant woman who was exiting the elevator. The athlete sued both the hotel and his soccer team in the state court of State X, asserting state law tort claims against both and seeking $100,000 in damages to compensate him for his injuries from the incident. The soccer team is a resident of State X and the hotel is a resident of State Y. The athlete served the summons and complaint on both defendants on May 1. After being heavily criticized by his teammates and fans of the team for suing his own team, the athlete amended the complaint so that the only defendant was the hotel (although the alleged damages remained the same) and served the amended complaint on the hotel on May 13. On June 9, the hotel filed a notice to remove the case to federal court in the State X district in which the athlete’s complaint was filed.

Can the hotel successfully remove the case to federal court?

A Yes, because there is complete diversity of parties and the amount in controversy is met.
B Yes, because the soccer team was fraudulently joined as a party.
C No, because the hotel failed to file the notice of removal in a timely manner.
D No, because diversity of parties is determined at the time the original complaint is filed.

A

The correct answer is: Yes, because there is complete diversity of parties and the amount in controversy is met.

Discussion of correct answer: Typically, a defendant has 30 days from the initial pleading to file a notice of removal to federal court. But where a case is not removable when the initial pleading is filed, and the complaint is amended in a way that now makes the case removable, a notice of removal must be made within 30 days of service of the amended pleading. This case was not originally removable because there was not complete diversity of parties. When the amended complaint made the hotel the only defendant, however, there did exist complete diversity, as well as a sufficient amount in controversy, which then made the case removable within 30 days of May 13.

How well did you know this?
1
Not at all
2
3
4
5
Perfectly
65
Q

Tax Inc. franchised thousands of tax preparation offices nationwide. Ten years ago, a franchisee signed a franchise agreement with Tax Inc. granting him the right to operate Tax Inc. franchises in Charleston, West Virginia. Believing that the franchisee breached his original agreement and was using his franchise to further a rival tax preparation business, Tax Inc. filed a request for a permanent injunction in federal court. In its complaint, Tax Inc. correctly identified the franchisee as a citizen of West Virginia and itself as a citizen of Delaware, where it was incorporated, and New York, where it had its primary place of business. In determining the cost of the injunction, Tax Inc. focused on the alleged improper use of the franchise, and valued the stopping of that improper use at $80,000. The franchisee calculated the injunction’s cost to be $25,000. He derived this figure from the remaining five years on his franchise agreement, arguing that he had made $5,000 per year in profit for the previous 10 years of the agreement and was likely to continue to do so for the remaining five years. The federal district court then dismissed Tax Inc.’s complaint for lack of subject-matter jurisdiction, finding that it failed to meet the $75,000 amount-in-controversy requirement for diversity jurisdiction. Tax Inc. appealed.

Did Tax Inc.’s complaint meet the necessary amount in controversy to be heard in federal court?

A Yes, because the value of an injunction is determined by the plaintiff.
B Yes, because the value of an injunction is determined by the larger of the injunction’s worth to the plaintiff or its cost to the defendant.
C No, because lost profits is the correct way to determine the value of an injunction.
D No, because Tax Inc.’s $80,000 figure was too speculative.

A

The correct answer is: Yes, because the value of an injunction is determined by the larger of the injunction’s worth to the plaintiff or its cost to the defendant.

Discussion of correct answer: For diversity jurisdiction, the amount in controversy must be more than $75,000 [28 U.S.C. Sec. 1332]. To determine the value of an injunction for amount in controversy purposes, courts look at the larger of two figures: (1) the injunction’s worth to the plaintiff; or (2) its cost to the defendant [JTH Tax, Inc. v. Frashier, 624 F.3d 635 (4th Cir. 2010)]. Here, Tax Inc. determined that the value of shutting down the franchisee’s office was $80,000, which meets the $75,000 requirement.

How well did you know this?
1
Not at all
2
3
4
5
Perfectly
66
Q

Arnold and Bates, citizens of Florida, are plaintiffs in an action brought in the United States District Court in Georgia against Manco, a Mississippi corporation, and Storeco, a Georgia corporation. Manco’s office and plant are located in Mississippi. At no time has Manco had an office or salesperson in Georgia. Storeco’s sole place of business is in Georgia. The complaint alleges that each of the plaintiffs sustained serious personal injuries when a blade broke on an electric lawnmower while the equipment was being demonstrated by a clerk in Storeco’s store in Georgia. Each plaintiff has requested damages in the sum of $80,000. The mower had been manufactured by Manco and shipped to Roe in Florida. Roe had a contract with Manco to act as exclusive distributor of Manco products in 11 states, including Georgia and Florida. Process was served personally on the president of Manco at Manco’s office in Mississippi and on the president of Storeco at its office in Georgia. Thereafter, Manco moved to dismiss the action on the grounds that the court had no jurisdiction over it.

Storeco also filed a counterclaim against Arnold for $74,000 which Storeco alleges to be due for merchandise previously sold to Arnold. Manco then filed a cross-claim against Storeco for $12,000, which it alleges is due for merchandise previously sold by Manco to Storeco. Under what circumstances can the U.S. District Court assert personal jurisdiction over Manco?
(A) Georgia has an appropriate long-arm statute and the assertion of personal jurisdiction would comport with due process.
(B) The assertion of personal jurisdiction would comport with due process (there being nationwide service of process in actions commenced in federal court).
(C) The amount in controversy exceeds $75,000, exclusive of interest and costs.
(D) Process was served upon Manco in accordance with both federal and Georgia law

A

(A) Georgia has an appropriate long-arm statute and the assertion of personal jurisdiction would comport with due process.

How well did you know this?
1
Not at all
2
3
4
5
Perfectly
67
Q

A company made screws in State A. The company sold the screws to the corporation, incorporated in State B. The corporation found that the screws were defective. The defective screws have caused $65,000 in damages. The corporation also discovered that the company did not send the proper number of screws, in violation of the contract between the companies. The corporation paid $100,000 for 1,000 screws, but only received 800 screws, causing $20,000 in damages. The corporation sued the company in federal district court for damages. The company filed a motion to dismiss the case for lack of subject matter jurisdiction.

Will the federal district court grant the company’s motion to dismiss?

A Yes, because the amount in controversy is insufficient to maintain an action in federal court.
B Yes, because the company conveying the remaining 200 screws to the corporation can resolve a portion of the amount in controversy.
C Yes, because federal courts do not hear breach of contract claims.
D No, because the amount in controversy meets the requirement for federal court jurisdiction.

A

The correct answer is: No, because the amount in controversy meets the requirement for federal court jurisdiction.

Discussion of correct answer: Where a plaintiff sues a defendant for multiple claims based on the same case or controversy, the separate damages of each claim can be aggregated together. Here, there are two claims regarding the screws. The first is that the screws were defective and caused damage. The second is that the plaintiff did not receive the proper amount of screws in the first place. If the plaintiff only sued on one claim or the other, the amount in controversy–$75,000–would not be met. However, because there are two claims against the defendant, the amount of each claim can be added together. Once the two claims are aggregated, the total amount claimed–$85,000–would meet the amount-in-controversy threshold of $75,000.

How well did you know this?
1
Not at all
2
3
4
5
Perfectly
68
Q

HoagieWorld franchised thousands of sandwich shops nationwide. Ten years ago, a franchisee signed a franchise agreement with HoagieWorld granting him the right to operate five franchises in State A. Believing that the franchisee breached his original agreement requiring him to purchase all of his meat, condiments, and bread from approved vendors, HoagieWorld filed a request for a permanent injunction in federal court. In its complaint, HoagieWorld correctly identified the franchisee as a citizen of State A and itself as a citizen of State B, where it was incorporated, and State C, where it had its primary place of business.

In determining the cost of the injunction, HoagieWorld focused on the alleged improper use of the franchise and reputational damage suffered by the brand, and valued the stopping of that improper use at $90,000. The franchisee calculated the injunction’s cost to be $5,000. He derived this figure from the increased yearly costs of using approved vendors, rather than local, less expensive vendors. The franchisee filed a motion to dismiss the case, claiming that the federal court lacked subject-matter jurisdiction to hear the case.

How should the court rule on the motion?

A Deny the motion, because federal courts have exclusive jurisdiction to hear cases involving copyright violations.
B Deny the motion, because there is a sufficient amount in controversy.
C Grant the motion, because the case deals with breach of contract, not a federal question.
D Grant the motion, because HoagieWorld’s $90,000 figure associated with reputational damage was too speculative.

A

The correct answer is: Deny the motion, because there is a sufficient amount in controversy.

Discussion of correct answer: For diversity jurisdiction, the amount in controversy must be more than $75,000 [28 U.S.C. Sec. 1332]. To determine the value of an injunction for amount-in-controversy purposes, courts look at the larger of two figures: (1) the injunction’s worth to the plaintiff; or (2) its cost to the defendant [JTH Tax, Inc. v. Frashier, 624 F.3d 635 (4th Cir. 2010)]. Here, HoagieWorld determined that the value of the injunction was $90,000, which meets the $75,000 requirement.

How well did you know this?
1
Not at all
2
3
4
5
Perfectly
69
Q

A man is a citizen of State A and filed a defamation action in federal court against a reporter and an editor, both of whom were citizens of State B. The claims against the two defendants, the reporter and the editor, arose from the same events. Plaintiff alleged that the amount in controversy with respect to each defendant was $100,000. State law caps damages in this type of case against defendants like the editor at $25,000. The editor filed a motion to dismiss for lack of subject matter jurisdiction.

Should the court grant the motion to dismiss for lack of subject matter jurisdiction?

(A) Yes, because there is an insufficient amount in controversy between the man and the editor.
(B) Yes, because the reporter and the editor are from the same state.
(C) No, because there is supplemental jurisdiction over the claim against the editor.
(D) No, because the Court looks to the man’s allegations to determine the amount in controversy.

A

(A) Yes, because there is an insufficient amount in controversy between the man and the editor.

How well did you know this?
1
Not at all
2
3
4
5
Perfectly
70
Q

A CEO brought a federal court action in diversity for fraudulent misrepresentation of the qualities of a yacht that he bought from a dealer. The court found for the CEO and awarded him $600,000 in compensatory damages. However, when the CEO’s lawyer received the judgment, she saw that it stated that the award was $670,000.

Which of the following would be the best way for the CEO’s lawyer to correct the problem under the Federal Rules?

(A) She could contact the boat dealer’s lawyer to let the boat dealer’s lawyer know that she was aware of the mistake and would accept $600,000 in full payment.
(B) She could contact the court to ask the court to reissue the judgment.
(C) She could file a motion for relief from a judgment or order.
(D) She could wait for the boat dealer’s lawyer to discover the mistake.

A

(C) She could file a motion for relief from a judgment or order.

How well did you know this?
1
Not at all
2
3
4
5
Perfectly
71
Q

Z filed an action against B, C, and D asserting: (1) infringement of copyright; and (2) breach of contract. Copyright actions are exclusively within the subject-matter jurisdiction of federal courts. All of the claims arose from B, C and D having produced a movie about three bank-robbing lawyers that Z had written for them. Z sought to recover $75,000 apiece from each defendant. All of the parties are citizens of State V.

Which of the following is true?

(A) If this action was filed in a State V state court, the defendants could not have it removed to federal court.
(B) If this action was filed in federal court, dismissal for lack of subject-matter jurisdiction would be required.
(C) If this case goes to trial in a state court and judgment is entered against Z, Z will have waived any lack of subject-matter jurisdiction by failing to assert it before or during trial.
(D) If this action were filed in a state court, the defendants would have the right to remove the action to federal court.

A

(D) If this action were filed in a state court, the defendants would have the right to remove the action to federal court.

How well did you know this?
1
Not at all
2
3
4
5
Perfectly
72
Q

A collector in State B bought a rare car from a local showroom in State C for $260,000. The parties executed a two-year warranty at the time of sale. The warranty was silent as to choice of forum in case of dispute. One year later, the collector relocated to State A. When the collector subsequently needed major repairs to the car, the State C showroom insisted that the warranty was void because unapproved mechanics had worked on the car. The collector sued the showroom in federal district court in State A for $83,000. The showroom filed a motion to dismiss for lack of jurisdiction.

Should the court grant the showroom’s motion?

(A) Yes, because the parties did not have a choice of forum clause that allowed either of them to sue in State A.
(B) Yes, because the showroom would not reasonably have expected to have been sued in State A.
(C) No, because it was not unforeseeable that the collector would move to another state.
(D) No, because State A’s long-arm statute could apply.

A

(C) No, because it was not unforeseeable that the collector would move to another state.

How well did you know this?
1
Not at all
2
3
4
5
Perfectly
73
Q

A man from State A rented a car and drove to State B for a work meeting. On his way back to State A, the man ran into the back of a State B driver’s car, injuring the State B driver. The State B driver then brought suit against the State A man in State B federal court claiming $100,000 in damages. The State A man argued that the case should be dismissed because the State B federal court lacked sufficient jurisdiction over him.

Which of the following statements is most accurate?

A The State B court should dismiss the case, because driving the car in State B does not provide sufficient minimum contact for the court to have jurisdiction over the State A man.
B The State B court should dismiss the case, because the rental of the car occurred in State A.
C The State B court should hear the case, because the man impliedly consented to being sued in State B by driving a car in the jurisdiction.
D The State B court should hear the case, because diversity is complete and the amount in controversy is over $75,000.

A

The correct answer is: The State B court should hear the case, because the man impliedly consented to being sued in State B by driving a car in the jurisdiction.

Discussion of correct answer: Consent can be implied, and one of the most common forms of implied consent is by driving on the roads of a state. Courts consider persons to have given implied consent to the laws regulating roads, and thus, if a defendant has a car accident on the road in that state, a court has personal jurisdiction over the defendant in an action arising from that accident.

How well did you know this?
1
Not at all
2
3
4
5
Perfectly
74
Q

The plaintiff sued the defendant in federal court. Subject matter jurisdiction existed. After discovery, the defendant filed a motion for summary judgment and argued that there is no genuine dispute as to any material fact and that the defendant is entitled to judgment as a matter of law. The court denied the motion, and the plaintiff won at trial. The defendant did not make a motion for judgment as a matter of law at trial. The court entered judgment on the jury’s verdict. The defendant filed a “renewed motion for judgment as a matter of law” 20 days after the court entered judgment for the plaintiff. Must the court deny the “renewed motion for judgment as a matter of law”?

(A) Yes, because the motion was not filed within 10 days of entry of judgment.
(B) Yes, because the defendant did not make a motion for judgment as a matter of law at trial.
(C) No, because the defendant filed a motion for summary judgment.
(D) No, because the defendant filed the motion within 28 days of the entry of judgment.

A

(B) Yes, because the defendant did not make a motion for judgment as a matter of law at trial.

How well did you know this?
1
Not at all
2
3
4
5
Perfectly
75
Q

An engineer was a citizen of State C and filed a case against a corporation in state court in State A. The corporation was incorporated in State B and had its principal place of business in State A. The complaint alleged that the corporation harmed the engineer in two unrelated transactions and that the plaintiff was harmed in each transaction in the amount of $50,000. The first claim was defamation and the second claim was based on a breach of contract. The corporation removed the case to federal court in order to assert a defense under federal law.

Was the removal to federal court proper?

(A) Yes, because the corporation asserted a federal defense.
(B) Yes, because the plaintiff could have filed the case originally in federal court.
(C) No, because the corporation had its principal place of business in State A.
(D) No, because there was an insufficient amount in controversy.

A

(C) No, because the corporation had its principal place of business in State A.

How well did you know this?
1
Not at all
2
3
4
5
Perfectly
76
Q

A plaintiff wanted to sue two defendants for damages stemming from the defendants’ reprinting of her novel in violation of copyright laws. The defendants reprinted and sold the novel in the Northern District of Illinois. One defendant resides in the Eastern District of Michigan, while the other resides in the Western District of Illinois. The plaintiff resides in the Northern District of Iowa.

Where would venue be proper for this action?

A The Northern District of Illinois.
B The Western District of Illinois or the Eastern District of Michigan.
C The Northern District of Illinois, the Western District of Illinois, or the Eastern District of Michigan.
D The Northern District of Illinois, the Western District of Illinois, the Northern District of Iowa, or the Eastern District of Michigan.

A

The correct answer is: The Northern District of Illinois.

Discussion of correct answer: Pursuant to 28 U.S.C Section 1391, venue is appropriate: (1) where any defendant resides, provided that all defendants are within the same state; (2) in the district where a substantial portion of events occurred; or (3) in the district where any defendant can be found, if there is no other district in which the action could be brought. Here, the action was based on copyright, a federal question. Venue is appropriate in the Northern District of Illinois because that is where the defendants reprinted and sold the novel. Because the defendants live in different states, the plaintiff cannot take advantage of the rule allowing a plaintiff to sue in any district where a defendant resides.

How well did you know this?
1
Not at all
2
3
4
5
Perfectly
77
Q

A plaintiff car driver from State A sued a defendant truck driver from State B for $150,000 in damages following a collision. The suit was filed in the United States District Court in State B. The defendant sought to remove to state court in State B, where the judges were well known to him.

Can the defendant remove to state court?

(A) No, because there is no federal statute that allows removal from federal court to state court.
(B) No, because the defendant is in the “home state” of the state that he wishes to remove to.
(C) Yes, because removal must be unanimous, and there is only one defendant.
(D) Yes, because the suit is based in diversity.

A

(A) No, because there is no federal statute that allows removal from federal court to state court.

How well did you know this?
1
Not at all
2
3
4
5
Perfectly
78
Q

A woman, a citizen of State A, was involved in a car accident where she negligently injured the driver and passenger of the other car. Both the other driver and passenger are residents of State B. The other driver filed a lawsuit against the woman alleging $30,000 in damages, while the passenger filed a lawsuit against the woman alleging $50,000 in damages. The plaintiffs wish to bring their action in federal court.

Does the federal court have jurisdiction?

A No, because one plaintiff is only alleging $50,000 in damages and the other is only alleging $30,000.
B No, because the accident occurred in State B.
C Yes, because the federal court has diversity jurisdiction over the matter.
D Yes, because the federal court has federal-question jurisdiction over the matter.

A

The correct answer is: No, because one plaintiff is only alleging $50,000 in damages and the other is only alleging $30,000.

Discussion of correct answer: For diversity jurisdiction, the amount in controversy must be more than $75,000 [28 U.S.C. Sec. 1332]. In order to meet the $75,000 threshold for diversity jurisdiction, a claimant may combine each amount in controversy against a single defendant. However, more than one claimant cannot add their claims together. Here, there are two separate claimants, each with claims less than $75,000. If one of them had made a claim over $75,000, there might have been subject matter jurisdiction.

How well did you know this?
1
Not at all
2
3
4
5
Perfectly
79
Q

A wife and her husband in State A were both writers. The husband fell in love with another woman in State B, and divorced the wife and moved to State B. A year later, the husband learned that the wife had just sold a novel to a publishing company which he believed was based on his writing ideas that she had improperly used. He filed suit against her in State B federal district court. The husband delivered the proper papers to a process server, but the process server failed to serve the wife. Months later, the husband called the wife only to find out that she had never heard of his lawsuit. The husband then sent her a written request for waiver of service of process, which met the requirements for such a waiver, and the wife immediately responded by waiving service. The wife then filed a motion to dismiss, arguing that she had no contacts with State B and thus the court did not have personal jurisdiction over her.

If the husband now argues that the wife may not assert the argument that the court does not have personal jurisdiction over her, will he prevail?

A No, because the husband was responsible for any insufficiency in the service of process.
B No, because the wife did not waive her objections to the court’s jurisdiction.
C Yes, because the wife waived her objections to the court’s jurisdiction.
D Yes, because waiver of service automatically subjects a party to a court’s jurisdiction.

A

The correct answer is: No, because the wife did not waive her objections to the court’s jurisdiction.

Discussion of correct answer: When a defendant waives service, she does not waive her objections to the court’s subject matter jurisdiction or personal jurisdiction. Therefore, despite the fact the wife waived the husband’s service of process, she may still argue that the court does not have jurisdiction over her.

How well did you know this?
1
Not at all
2
3
4
5
Perfectly
80
Q

A driver rented a car from Rental Company in State B. The driver was a citizen of State A. Rental Company was incorporated in State C, and had places of business in State B and State C. The driver was driving on a highway in State B when the accelerator pedal got stuck, and he hit the victim, a resident of State B, who was also driving on the highway, causing severe injuries to the victim. The victim filed a lawsuit in federal district court in State B against the driver for $90,000 in damages. She served the summons and complaint on the driver two weeks later. The driver, citing the stuck accelerator pedal, filed a third-party complaint against Rental Company, seeking indemnification for the $90,000 the victim sought against the driver, as well as additional damages in the amount of $50,000. Rental Company filed a motion to dismiss the third-party complaint for lack of subject-matter jurisdiction.

Will the federal district court grant Rental Company’s motion to dismiss?

A Yes, because there is no longer complete diversity of citizenship amongst the parties in the lawsuit as a whole.
B Yes, because the amount in controversy the driver seeks against Rental Company is insufficient for federal court diversity jurisdiction.
C No, because the court can exercise supplemental jurisdiction over the driver’s claim against Rental Company.
D No, because there is complete diversity of citizenship between the driver and Rental Company.

A

The correct answer is: No, because the court can exercise supplemental jurisdiction over the driver’s claim against Rental Company.

Discussion of correct answer: Under 28 U.S.C. Sec. 1367, a federal court can exercise supplemental jurisdiction over additional claims that arise from the same case or controversy, if the court already has original jurisdiction over the main claim. Here, the third-party complaint arises out of the same accident. The court has diversity jurisdiction over the original claim because there is complete diversity of the original plaintiff and defendant, and the amount in controversy meets the mandatory minimum required. Therefore, the federal district court is entitled to exercise supplemental jurisdiction over the third-party complaint, whatever the citizenship of the third-party defendant and regardless of the contested amount between the third-party plaintiff and the third-party defendant.

How well did you know this?
1
Not at all
2
3
4
5
Perfectly
81
Q

A fashion designer and a pattern maker entered into a services contract. The fashion designer’s place of business was located in Somerset County, which is in State A. The contract contained a provision stating that any dispute arising out of the contract would be litigated either in a state court located in Somerset County, or in the federal district court located in Somerset County. The pattern maker was a resident of State B, and the services that were the subject of the contract were to be performed in State B. A few months after the contract was signed, the fashion designer sued the pattern maker, alleging breach of the contract and citing damages in the amount of $150,000. The fashion designer brought the suit in the federal district court in Somerset County. The pattern maker argued that the venue was improper, and that the suit should have been brought in State B.

Will the court accept the pattern maker’s argument?

(A) Yes, because the services that were the subject matter of the contract were to be performed in State B.
(B) Yes, because a forum selection clause cannot override federal law regarding venue.
(C) No, because the pattern maker consented to the forum.
(D) No, because one of the litigants is located in the forum state.

A

(A) Yes, because the services that were the subject matter of the contract were to be performed in State B.

How well did you know this?
1
Not at all
2
3
4
5
Perfectly
82
Q

An actress sued a taxicab company in federal court. In the weeks leading up to the trial, she sent the taxicab company four separate interrogatories. The taxicab company answered the first three interrogatories but refused to answer the fourth one. May the taxicab company properly refuse to answer the fourth interrogatory?

(A) Yes, because interrogatories may only be sent to witnesses who are not parties to the litigation.
(B) Yes, because the plaintiff is limited to three interrogatories.
(C) No, because the defendant is a party to the litigation.
(D) No, because, having answered the first three interrogatories, the taxicab company is estopped from refusing to answer the fourth.

A

(C) No, because the defendant is a party to the litigation.

83
Q

In what situation would a motion to dismiss under Fed. R. Civ. P. 12(b)(6) be appropriate?

(A) A bartender filed a lawsuit against a man for assault but failed to join another man involved in making the threats.
(B) A pedestrian filed a lawsuit against a driver but failed to serve the driver within 90 days of filing the complaint.
(C) A woman filed a lawsuit against her neighbor alleging that he had engaged in harassment to try and drive her to move out but failed to state any specific facts.
(D) An employee filed a lawsuit against his employer but failed to engage in any discovery or otherwise move the case forward in 18 months.

A

(C) A woman filed a lawsuit against her neighbor alleging that he had engaged in harassment to try and drive her to move out but failed to state any specific facts.

84
Q

A State A toymaker developed a robot that responded to children’s thoughts and took it from his home in the Eastern District of State A to a toy convention taking place in the Northern District of State A. There he showed it to two manufacturers, one from the Eastern District of State A, and one from the Western District of State B, the latter of which was headquartered and incorporated in State B, although it had a retail store in the Western District of State A. The two manufacturers colluded to steal the toymaker’s designs and repurpose them for their own benefit.

If the toymaker now seeks to file a federal trademark claim against both manufacturers in federal court, in what venue(s) can he file his claim?

A The Northern District of State A.
B The Eastern District of State A.
C The Northern and Eastern District of State A, and the Western District of State B.
D The Northern, Eastern, and Western District of State A, and the Western District of State B.

A

The correct answer is: The Northern District of State A.

Discussion of correct answer: According to 28 U.S.C. Section 1391, venue is proper: (1) in any district in which a defendant resides, if all defendants are residents of the State in which the district is located; (2) in a juridical district in which a substantial part of the events or omissions giving rise to the claim occurred, or a substantial part of property that is the subject of the action is situated; or (3) if there is no district in which an action may otherwise be brought as provided, in any judicial district in which any defendant is subject to personal jurisdiction. Because the defendants reside in two different states, venue would not be proper in any judicial district in which any defendant resides. Instead, venue would be proper where a substantial part of the events giving rise to the claim occurred, which here would be the Northern District of State A.

85
Q

A plaintiff who was a citizen of State A was riding on a bus when the bus collided with a truck. The plaintiff lost his sight. He sued the bus company, which was incorporated in State B, in federal district court for $1,000,000 in damages. The jury found the bus company partially liable and returned a verdict in favor of the plaintiff in the amount of $500,000. The jury also found that the corporation that owned the truck would be separately liable for $500,000 of damages, for a total of $1,000,000, but the truck corporation was not a defendant. Assume that this jurisdiction does not apply joint and several liability. Subsequently, the plaintiff filed a lawsuit against the owner of the bus company, seeking the remaining $500,000. The owner of the bus company filed a motion to dismiss.

Will the federal district court grant the motion to dismiss?

A Yes, because the truck corporation was the other entity that was also liable for the injuries.
B Yes, because the new lawsuit is barred by claim preclusion.
C No, because the owner of the bus company is liable for damages arising from use of the buses in the company.
D No, because the jury found that the plaintiff would be entitled to the $1,000,000 in damages.

A

The correct answer is: Yes, because the new lawsuit is barred by claim preclusion.

Discussion of correct answer: The owner of the bus company is the same party as the bus company itself. Claim preclusion bars a new lawsuit against the bus company, its owner, and any other individual or company that is the same party as the bus company or is in privity with the bus company.

86
Q

A driver, a citizen of State A, traveled to State B by airplane. While in State B, the driver saw a motorcycle shop, and stopped in to rent a motorcycle from a rental company incorporated and with two business locations, both also located in State B. All of the property the company owned was located in State B. The company advertised in local newspapers and television stations reaching States B and C. The driver rode the motorcycle in State B and State C. While in State C, the clutch on the motorcycle failed, and the driver got into a single vehicle accident, sustaining major injuries and totaling the motorcycle. The driver was taken to a hospital in State B. When the driver got out of the hospital, he sued the rental company in federal district court located in State A for renting him a faulty motorcycle, and asked for $250,000 in medical expenses and pain and suffering. The company filed a motion to dismiss for lack of personal jurisdiction over the company.

Will the federal district court grant the company’s motion to dismiss for lack of personal jurisdiction over the company?

A Yes, because the company is neither incorporated in nor has a place of business in State A, and has not otherwise consented to personal jurisdiction in State A.
B Yes, because the accident that forms the basis of the cause of action in this case occurred in State C, not State A.
C No, because the company appeared in court, and therefore became subject to the court’s jurisdiction.
D No, because the company is reachable by a long-arm statute.

A

The correct answer is: Yes, because the company is neither incorporated in nor has a place of business in State A, and has not otherwise consented to personal jurisdiction in State A.

Discussion of correct answer: Corporations may have citizenship in both the state in which it is incorporated and in the state where it has its principal place of business. The company is therefore a citizen of, and subject to personal jurisdiction in, State B. The company does not have a business location and is not incorporated in State A. The driver happened to see the shop while on vacation and stopped in. However, the company does not advertise to State A’s citizens or otherwise seek to do business in such a way as to implicate State A’s interest in regulation. Generally, the company has done nothing to avail itself of the benefits of State A (the forum state) or to otherwise create an expectation that it would be haled into State A’s courts. Therefore, there are not even minimum contacts between the company and State A to establish any sort of jurisdiction.

87
Q

A large fast food company with many franchises throughout the state has been accused of systematically forcing many of its employees to work overtime without receiving compensation, in violation of federal law. A group of these employees hired a large law firm to sue the company and the franchises accused of violating this federal law. The firm does its best to send out notices to all of the possible members of the class action, but the population is large, and they cannot reach everyone.

Which of the following is true regarding personal jurisdiction in this class action lawsuit?

(A) The court must have personal jurisdiction over the absent members of the plaintiff class, who must receive adequate notice of the pendency of the action.
(B) The court must have personal jurisdiction over the absent members of the plaintiff class, who must be afforded the opportunity to opt out of the class.
(C) The court must have personal jurisdiction over the absent members of the plaintiff classes, who are not required to receive notice or an opportunity to opt out of the class.
(D) The court must have personal jurisdiction over each of the defendants named in the action.

A

(D) The court must have personal jurisdiction over each of the defendants named in the action.

88
Q

A State A resident received a job offer on the other side of the country. He decided to take the offer and move to State C. He went to his local Brand Z car dealer in State A to shop for a new Brand Z. He asked the dealer for advice, stating that he was thinking about driving across the country. Based on the dealer’s assertions that the new Brand Z Model X “was perfect for all kinds of weather,” the man bought the Model X. On the third day of his trip, some 1,000 miles from either State A or State C, he was involved in an accident in State B. The car slid out of control and smashed into a fence. Due to the nature of his injuries, the man never made it to State C or took the job. The man sued the State A car dealer for $100,000 in a State B state court, alleging a design defect in the car.

Does the State B court have jurisdiction over the car dealer?

A No, because the car dealer does not have sufficient minimum contacts with State B.
B No, because the car was sold in State A to a resident of State A and the car dealer is a resident of State A.
C Yes, because the car dealer knew the vehicle was likely to travel all around the country.
D Yes, because as a commercial seller, the car dealer purposefully availed itself of interstate commerce, and so being sued in State B would not offend traditional notions of fair play and substantial justice.

A

The correct answer is: No, because the car dealer does not have sufficient minimum contacts with State B.

Discussion of correct answer: Where a party is not a citizen of a state, the court may still establish personal jurisdiction over that party if he has a minimum number of contacts with the state. A minimum contacts analysis looks at the nature of the defendant’s relationship with the forum state and whether the defendant would have reasonably expected to be called into court in the forum state. If the defendant purposely involves himself in transactions within the state, then he receives the benefits and privileges of that state’s laws, and so it is fair that he is exposed to the jurisdiction of that forum state as well. Importantly, if a defendant purposely availed himself of the benefits of the forum state, sufficient minimum contacts for personal jurisdiction would exist. Here, the car was the only connection between the dealer and State B. While the dealer may have had some abstract knowledge that the car could travel the country, he did not know that the car was going to State B, and importantly, he did not purposefully direct any of his actions toward State B.

89
Q

A plaintiff sued a defendant in federal court in State A under diversity jurisdiction. The defendant was a citizen of State B and did not derive at least $500,000 in annual revenue from sales in State A. The plaintiff alleged that the defendant caused injury to the plaintiff by an act in State B that harmed the plaintiff in State A. The long-arm statute of State A empowered the courts in State A to reach out-of-state defendants who caused injury in the state by an act or omission outside the state only if the defendant derived at least $500,000 in annual revenue from sales in State A. The defendant filed a timely motion to dismiss for lack of personal jurisdiction. The court granted the motion.

Was the court correct to grant the motion to dismiss?

(A) Yes, because the long-arm statute of the forum state did not reach the defendant.
(B) Yes, because the act that allegedly gave rise to liability occurred outside the forum state.
(C) No, because it would have been constitutional for State A to assert personal jurisdiction over the defendant under these circumstances.
(D) No, because the state long-arm statute was not relevant in federal court.

A

(A) Yes, because the long-arm statute of the forum state did not reach the defendant.

90
Q

A man sued a company that owned a national television talk show in State A federal court claiming the company violated his federal copyright rights by using one of his songs as the theme song for the show. The man was a citizen of State B, while the company was incorporated in State C and had its main office in State A. However, the company did have an office in State A where many decisions involving the show were made. The man claimed $60,000 in actual damages, and $100,000 in “reputational harm.” A recent State A Supreme Court decision found that the maximum amount of damages that could be claimed for “reputational harm” were $5,000, although the federal courts had established no such limitation.

Can the action be heard in federal court?

(A) Yes, because the amount in controversy is over $75,000.
(B) Yes, because the man’s claim is based on federal copyright.
(C) No, because the Erie doctrine would limit possible damages to $65,000.
(D) No, because there is no diversity of citizenship between the parties.

A

(B) Yes, because the man’s claim is based on federal copyright.

91
Q

A driver rented a car from Rental Company in State B. The driver was a citizen of State A. Rental Company was incorporated in State C, and had places of business in State B and State C. The driver was driving on a highway in State B when the accelerator pedal got stuck, and he hit the victim, a resident of State B, who was also driving on the highway, causing severe injuries to the victim. The victim filed a lawsuit in federal district court in State B against the driver for $90,000 in damages. She served the summons and complaint on the driver two weeks later. The driver, citing the stuck accelerator pedal, filed a third-party complaint against Rental Company, seeking indemnification for the $90,000 the victim sought against the driver, as well as additional damages in the amount of $50,000. Rental Company filed a motion to dismiss the third-party complaint for lack of subject-matter jurisdiction.

Will the federal district court grant Rental Company’s motion to dismiss?

(A) Yes, because there is no longer complete diversity of citizenship amongst the parties in the lawsuit as a whole.
(B) Yes, because the amount in controversy the driver seeks against Rental Company is insufficient for federal court diversity jurisdiction.
(C) No, because the court can exercise supplemental jurisdiction over the driver’s claim against Rental Company.
(D) No, because there is complete diversity of citizenship between the driver and Rental Company.

A

(C) No, because the court can exercise supplemental jurisdiction over the driver’s claim against Rental Company.

92
Q

A client hired an architect to design a house, but a dispute soon arose. The client, a citizen of state A, believes that the architect, a citizen of State B, breached their contract, causing the client $200,000 in damages. The client filed suit in federal district court in State A. When he found out that the architect was about to take a vacation to State A in order to attend a family reunion, the client had a professional process server serve the architect while she was at her family reunion.
Assuming that the contract does not specify where a contract claim is to be heard, will the federal district court in State A have jurisdiction over the architect?

(A) Yes, because the architect’s trip to State A means that she has substantial contacts with the state.
(B) Yes, because the architect was served with process while in the forum state.
(C) No, because the architect does not intend to remain in State A indefinitely.
(D) No, because the architect was not a resident of State A at the time of service.

A

(B) Yes, because the architect was served with process while in the forum state.

93
Q

On January 1, a State A plaintiff sued a State B defendant and a State A defendant in a State A state court. The plaintiff’s claim against each defendant exceeded $100,000. The case went through a series of pretrial conferences, and after a long, drawn-out discovery process, the plaintiff settled her case against the State A defendant on January 20 of the following year. The case has now been certified to proceed to trial.

At this point, what recourse does the State B defendant have?

A He could remove the case to federal court, because he has 30 days from when the grounds for removal become apparent.
B He can remove because the plaintiff, by settling with the State A defendant, provided complete diversity.
C He cannot remove at this point, because he is time barred.
D He cannot remove the case, because removal can only occur before the close of discovery.

A

The correct answer is: He cannot remove at this point, because he is time barred.

Discussion of correct answer: It is true that a defendant can remove within 30 days of when the grounds for removal become apparent. However, in diversity cases, removal cannot occur more than one year after the date when the suit was filed. Here, more than one year has passed. Therefore, the defendant can no longer remove the case to federal court.

94
Q

A State X school district hired a State X contractor to do environmental clean-up work and construction on an area of land that it planned to use as athletic fields for its state champion soccer teams. As part of the contract, the contractor agreed to reduce levels of contaminants to below federally mandated minimums, as set by EPA regulations. The EPA regulations do not provide for a private cause of action, but do provide for government-imposed penalties against any party in violation of the regulations. The contractor completed the work, certifying that it had done so within EPA regulations, but when an EPA inspector inspected the land, she determined that the level of contaminants on the property were still in violation of the EPA regulations. The school district filed suit in federal district court, arguing that the contractor had breached the contract under State X contract law because the contractor failed to reduce the contaminant levels to below federal minimums. The school alleged $500,000 in damages. The contractor argued that it did the work properly, and that the EPA inspector used faulty measurements. The EPA has not yet penalized the school district, and has given the school district one year to remedy the level of contaminants.

Does the federal district court have jurisdiction over the case?

A Yes, because the amount in controversy is $500,000.
B Yes, because it has federal question jurisdiction.
C No, because there is neither diversity jurisdiction nor a federal question.
D No, because the EPA has not yet penalized the school district.

A

The correct answer is: No, because there is neither diversity jurisdiction nor a federal question.

Discussion of correct answer: A federal court may only hear a case where it has subject-matter jurisdiction, based on either diversity jurisdiction or a federal question. Here, the parties are from the same state, so there is no diversity jurisdiction. Although the case does reference a federal regulation, there is no federal question, because the case deals with whether the contractor performed its duties under the contract. Put another way, the cause of action arises under state contract law, not federal law, and there is no disputed area of federal law.

95
Q

After an automobile accident, the plaintiff brought suit against the defendant in the United States District Court for State X, where venue was proper. After being served, the defendant consulted his attorney and they felt that a change of venue would be better. The defendant then filed a motion to transfer venue to the United States District Court for the District of State Y. Both parties agreed that venue would have been proper there, and that it would be more convenient for the parties and witnesses. The court scheduled a pretrial conference to settle the issue.

Two weeks later at the pretrial conference, the plaintiff and defendant informed the court that they had agreed that the case should be transferred to the United States District Court for the District of State Z, which all parties agreed was not a proper venue. The plaintiff and defendant each thought that a State Z jury pool would be more sympathetic to their case.

May the court transfer the case to the United States District Court for the District of State Z?

A No, because that court is not a court where the case might have originally been brought.
B No, because venue is proper in the district where the case was originally filed.
C Yes, because all parties have agreed that the court may do so.
D Yes, if the court could be convinced that the transfer was also for the convenience of the parties and witnesses, in the interest of justice.

A

The correct answer is: Yes, if the court could be convinced that the transfer was also for the convenience of the parties and witnesses, in the interest of justice.

Discussion of correct answer: This question asks whether a change in venue is proper. Where the case is initially brought in a judicial district where venue is proper, a court may transfer the case to any other district in which it might have been brought (either initially or with the consent of all parties), “for the convenience of parties and witnesses, in the interest of justice” [28 U.S.C. Sec. 1404(a)]. Here, the facts clearly establish the consent of the parties to a transfer to a District Court in State Z. However, the facts do not clearly establish whether such a transfer would be in the interest of justice because it would be convenient for not only the parties but also the witnesses. Here, the only motivation given for the parties’ desire to move to State Z is the belief that the jury pool would be more sympathetic Thus, the court may transfer this case to the District Court in State Z only if it would be convenient for the parties and witnesses.

96
Q

A supermarket employee filed an employment discrimination suit in federal court against the supermarket she worked for. Her suit claimed that her manager had asked her out on a date and then gave a promotion she deserved to someone else when she spurned his advances. The supermarket responded that the manager was justified in not giving the employee the promotion because she was more interested in dating the customers than she was in working. During discovery, the supermarket sought to discover detailed information about the employee’s sexual history, including the names of every person she had sex with in the past 10 years. The employee refused to answer deposition questions related to this subject, and the supermarket brought a motion to compel. The court denied the motion to compel and granted the employee’s cross-motion for a protective order.

Was the court correct in granting the protective order?

(A) Yes, because of the unreasonable intrusion on the employee’s privacy.
(B) Yes, because information of this nature is never appropriate for discovery.
(C) No, because the information is evidence of habit.
(D) No, because the information is evidence of motive.

A

(A) Yes, because of the unreasonable intrusion on the employee’s privacy.

97
Q

A pharmaceutical company sued two engineers for patent infringement, a federal law claim. The pharmaceutical company brought the lawsuit in State C, in the judicial district in which one of the engineers lived. The other engineer also resided in State C. The alleged patent infringement took place in State B, which is the state in which the pharmaceutical company is incorporated and has its principal place of business. The pharmaceutical company alleges that it had suffered damages in the amount of $60,000 arising from the alleged infringement. One of the engineers moved to dismiss the lawsuit on the basis of improper venue.

Will the engineer’s motion to dismiss be successful?

(A) No, because the plaintiff is not a resident of the same state as the defendants.
(B) No, because both engineers are domiciled in the forum state.
You Answered
(C) Yes, because the alleged infringement took place outside of the forum.
A pharmaceutical company sued two engineers for patent infringement, a federal law claim. The pharmaceutical company brought the lawsuit in State C, in the judicial district in which one of the engineers lived. The other engineer also resided in State C. The alleged patent infringement took place in State B, which is the state in which the pharmaceutical company is incorporated and has its principal place of business. The pharmaceutical company alleges that it had suffered damages in the amount of $60,000 arising from the alleged infringement. One of the engineers moved to dismiss the lawsuit on the basis of improper venue.

Will the engineer’s motion to dismiss be successful?

(A) No, because the plaintiff is not a resident of the same state as the defendants.
(B) No, because both engineers are domiciled in the forum state.
(C) Yes, because the alleged infringement took place outside of the forum.
(D) Yes, because the amount in controversy is only $60,000.

A

(B) No, because both engineers are domiciled in the forum state.

98
Q

A shareholder of a corporation brought a lawsuit against the corporation in federal court, alleging that several of the corporation’s board members had breached their duty of loyalty to the corporation. Prior to the trial, the shareholder asked the corporation to hand over the minutes of board meetings that had taken place during the time of the alleged breaches. The corporation refused to hand over the minutes, and in its objection did not state a reason for its refusal. The shareholder brought a motion to compel disclosure of the minutes. Should the court grant the motion?

(A) No, because the evidence is not a mandatory disclosure.
(B) No, because a motion to compel is not the appropriate means of obtaining the evidence.
(C) Yes, because the evidence is relevant.
(D) Yes, because the defendant did not cite a reason for its objection.

A

(C) Yes, because the evidence is relevant.

99
Q

A woman filed suit against a logger in a State A court claiming that he trespassed on her land and removed many of the trees without her permission. Both the woman and the logger were State A residents. Although it was unclear how many trees had been taken, the woman claimed $1 million in damages. A week later, the logger took a job in State B and moved there, establishing residency with an intent to permanently remain there. The logger then filed a motion to have the suit removed to federal court.

How should the court rule?

(A) Grant the motion, because the man is a citizen of State B.
(B) Grant the motion, because the suit involves natural resources.
(C) Deny the motion, because the amount in controversy requirement may not be met.
(D) Deny the motion, because the woman is a citizen of State A.

A

(D) Deny the motion, because the woman is a citizen of State A.

100
Q

ABC Corporation was incorporated and had its place of business in State A. It made widgets. The tor, a key component of the widget, was made by BCD Company incorporated and with its place of business in State B. ABC sued BCD in federal court for damages from defective tors in the amount of $500,000. During discovery, ABC sent 10 requests for admission to BCD. One stated that BCD made the tors that were used in making the widgets at issue. BCD made the admission. Just before discovery ended, BCD found out that ABC bought tors from a different company in the past. During trial, BCD denied that it made the tors that caused the widgets to break, and sought to litigate the issue of who made the tors. May BCD raise this issue at trial?

(A) Yes, because ABC previously bought tors from a different company.
(B) Yes, because BCD did not know that ABC had bought tors from a different company.
(C) No, because BCD did not file a motion to bring in the other company that had provided tors to ABC.
(D) No, because BCD agreed to ABC’s request for admission that BCD had made the tors at issue in the case.

A

(D) No, because BCD agreed to ABC’s request for admission that BCD had made the tors at issue in the case.

101
Q

Paula and Pete were hit by a truck driven by an employee of the D Corporation. The accident occurred in California, where Paula and Pete were living at the time. D Corporation is incorporated in Delaware, but its headquarters are in Florida and its principal place of business is in Michigan. After the accident, Paula and Pete moved to Miami, Florida, where they planned to retire. After moving, they contacted a lawyer, who filed suit against D Corporation in the appropriate U.S. District Court where Paula and Pete lived, alleging diversity jurisdiction. The suit alleged that each plaintiff had suffered personal injuries in excess of $75,000. D Corporation moved to dismiss the case for lack of subject-matter jurisdiction. Should D Corporation’s motion be granted by the district court?

(A) Yes, because D Corporation’s headquarters are in Florida.
(B) Yes, because the plaintiffs were citizens of California when they were injured.
(C) No, because diversity subject-matter jurisdiction exists.
(D) No, because D Corporation is a citizen of Delaware only.

A

(C) No, because diversity subject-matter jurisdiction exists.

Discussion of correct answer: The requirements for diversity subject-matter jurisdiction are both satisfied based on these facts: (1) the plaintiffs are citizens of Florida, and D Corporation is a citizen of Delaware and Michigan (a corporation is deemed to be a citizen of the states in which it is incorporated and in which the officers control the corporation); and (2) each plaintiff is claiming damages in excess of $75,000. Note that it is irrelevant that the corporation’s principal manufacturing plant is in Florida. Although the diversity statute links corporate citizenship to the corporation’s principal place of business, the Supreme Court has interpreted that phrase to mean the corporation’s “nerve center”–i.e., the place where the corporation is controlled.

102
Q

A plaintiff sued a defendant in state court in State A about an accident that occurred entirely within State A. The plaintiff and the defendant were of diverse citizenship, and the amount in controversy exceeded $75,000. The defendant, a citizen of State B, removed the case to the United States District Court for the District of State A. The defendant then made a motion to transfer the case, for the convenience of parties and witnesses, in the interest of justice, to the United States District Court for the District of State B. That motion was granted. At a pretrial conference, the federal court in State B ruled that it would apply the substantive law of State B to the case because it would be more convenient to do so. Was the court correct to rule that the law of State B would apply to the case?

(A) Yes, because a federal court sitting in diversity must apply the substantive law of the state where it sits.
(B) Yes, because the court determined that it would be more convenient to apply the law of State B in State B.
(C) No, because federal courts apply federal substantive law.
(D) No, because the law that is applicable to a case follows the case when it is transferred as a matter of convenience.

A

(D) No, because the law that is applicable to a case follows the case when it is transferred as a matter of convenience.

103
Q

A teacher, who was a citizen of State A, filed an action in the U.S District Court in State A against a stockbroker, a State B citizen, and his firm, a State C corporation, alleging that they violated SEC federal regulations and perpetrated a fraud during the sale of securities to the teacher. The teacher claimed that his total damages from the fraud were $66,000. The teacher further alleged in a second cause of action that the stockbroker did not repay an $8,000 loan that the teacher made to him.

Which of the following is true?

(A) The action should be dismissed by the District Court, because the value of the aggregated claims does not meet the amount-in-controversy requirement.
(B) The District Court may exercise supplemental jurisdiction over the second cause of action, but only if it arises from a common nucleus of operative facts with the first cause of action.
(C) The federal court has subject matter jurisdiction, because there is complete diversity of citizenship between all plaintiffs and defendants.
(D) The federal court lacks subject matter jurisdiction, because the violation of SEC federal regulations must be tried in the state court where such violations allegedly occurred.

A

(B) The District Court may exercise supplemental jurisdiction over the second cause of action, but only if it arises from a common nucleus of operative facts with the first cause of action.

104
Q

A plaintiff wanted to sue two defendants for damages stemming from the defendants’ reprinting of her novel in violation of federal copyright laws. The defendants reprinted and sold the novel in the Northern District of State A. One defendant resided in the Eastern District of State A, while the other resided in the Western District of State A. The plaintiff resided in the Northern District of State B.

Where would venue be proper for this action?

A The Northern District of State A only.
B The Western District of State A or the Eastern District of State A.
C The Northern District of State A, the Western District of State A, or the Eastern District of State A.
D The Northern District of State A, the Western District of State A, the Northern District of State A, or the Eastern District of State B.

A

The correct answer is: The Northern District of State A, the Western District of State A, or the Eastern District of State A.

Discussion of correct answer: Pursuant to 28 U.S.C. Section 1391, venue is appropriate: (1) where any defendant resides, provided that all defendants are within the same state; (2) in the district where a substantial portion of events occurred; or (3) in the district where any defendant can be found, if there is no other district in which the action could be brought. Here, the action was based on a federal copyright violation, a federal question. Venue would be appropriate in the Northern District of State A because that is where the defendants reprinted and sold the novel. Also, venue would be proper in the Western and Eastern Districts of State A because those are the districts where the defendants live.

105
Q

A plaintiff brought a civil action in federal court against a corporation that owned a coal mine for violating federal labor law. The corporation filed a motion for summary judgment and cited to depositions in the record to show that the plaintiff would not be able to produce admissible evidence to support a finding of fact without which the plaintiff could not prevail. The plaintiff responded, but did not point out anywhere in the depositions or otherwise in the summary judgment record anything to show that the plaintiff could establish the crucial fact. In reviewing the summary judgment record, the court noticed deposition testimony not cited by either party that showed that the plaintiff could produce evidence of the crucial fact.

May the court grant the motion for summary judgment?

(A) No, because the corporation had the burden to disprove at least one element of the plaintiff’s case.
(B) No, because the court must take into account the entire record before it.
(C) Yes, because the court does not have the power to consider materials in the record that the parties do not cite.
(D) Yes, because the court has the discretion, but not the duty, to consider materials in the record that the parties do not cite.

A

(D) Yes, because the court has the discretion, but not the duty, to consider materials in the record that the parties do not cite.

106
Q

Two plaintiffs sued a defendant in federal court for a state law tort cause of action for which there was no statutory cap on damages. The plaintiffs each alleged that more than $75,000 was in controversy as to his claim, and demanded a judgment for more than $75,000, but neither pled these damages with particularity. The first plaintiff was domiciled in State B but moved before the suit was filed to State A. This first plaintiff resided in State A at the time the suit was filed but at all times intended to return to State B upon the completion of a temporary work assignment. The second plaintiff was domiciled at all times in State A. The defendant was a corporation incorporated under the laws of State C with its principal place of business in State B. Just before trial, after two years of pleadings, discovery, and a final pretrial conference, the defendant filed a motion to dismiss for lack of subject-matter jurisdiction. The court granted the motion.

Was the trial court correct to grant the motion to dismiss for lack of subject-matter jurisdiction?

(A) Yes, because there was incomplete diversity of citizenship.
(B) Yes, because the plaintiffs did not plead the amount in controversy with particularity.
(C) No, because the motion was filed too late.
(D) No, because the first plaintiff resided in State A at the time the suit was filed.

A

(A) Yes, because there was incomplete diversity of citizenship.

107
Q

A State A citizen was injured in State B when the brakes on the truck that the citizen had rented from a State B rental corporation failed. The citizen sued the corporation in federal district court for $125,000 in damages. The corporation filed an answer generally denying liability. In its motion for summary judgment, the corporation claimed that it was not liable for the full amount of damages because the citizen was contributorily negligent. Will the court allow the corporation to raise contributory negligence?

(A) Yes, because the defense of contributory negligence may be raised in a motion for summary judgment.
(B) Yes, because the citizen’s contributory negligence centrally informs the corporation’s liability.
(C) No, because whether the citizen was contributorily negligent is not relevant to whether the corporation is liable as a general matter.
(D) No, because the corporation waived its defense of contributory negligence.

A

(D) No, because the corporation waived its defense of contributory negligence.

108
Q

A plaintiff filed a case against a defendant in federal court. The parties were not diverse, but federal question jurisdiction existed over the plaintiff’s claim. The defendant brought a state-law compulsory counterclaim against the plaintiff for damages that were alleged to total $100,000. The plaintiff then brought in a third-party and claimed that the third-party was liable under state law to indemnify the plaintiff if the plaintiff were found to be liable to the defendant on the defendant’s compulsory counterclaim for $100,000. The third-party defendant was incorporated in State A and had its principal place of business in State B. The plaintiff was a citizen of State A. The third-party defendant moved to dismiss the third-party claim for lack of subject-matter jurisdiction. The court granted the motion.

Was the court correct to grant the motion to dismiss the third-party claim for lack of subject-matter jurisdiction?

(A) Yes, because the third-party claim was not sufficiently related to the other claims to be within the court’s supplemental jurisdiction.
(B) Yes, because supplemental jurisdiction did not extend to a third-party claim made by the plaintiff.
(C) No, because diversity jurisdiction existed between the plaintiff and the third-party defendant.
(D) No, because supplemental jurisdiction existed over the third-party claim.

A

(D) No, because supplemental jurisdiction existed over the third-party claim.

109
Q

Company Y sued Company Z in federal court on a claim that Company Z had infringed its trademark by selling clothing with an almost imperceptibly different logo. After Company Y, at a bench trial, concluded its case-in-chief, Company Z believed that Company Y had not proven its case. Company Z filed a motion for judgment as a matter of law (JMOL).

Did Company Z file an appropriate motion?

(A) Yes, because it believed that Company Y had not proven its case.
(B) Yes, because filing a JMOL would preserve its rights to file a renewed JMOL if necessary.
(C) No, because a JMOL is only appropriate in a jury trial.
(D) No, because it should have waited until after Company Z’s case-in-chief.

A

(C) No, because a JMOL is only appropriate in a jury trial.

110
Q

An engineer was a citizen of State C and filed a case against a corporation in state court in State A. The corporation was incorporated in State B and had its principal place of business in State A. The complaint alleged that the corporation harmed the engineer in two unrelated transactions and that the plaintiff was harmed in each transaction in the amount of $50,000. The first claim was defamation and the second claim was based on a breach of contract. The corporation removed the case to federal court in order to assert a defense under federal law.

Was the removal to federal court proper?

(A) Yes, because the corporation asserted a federal defense.
(B) Yes, because the plaintiff could have filed the case originally in federal court.
(C) No, because the corporation had its principal place of business in State A.
(D) No, because there was an insufficient amount in controversy.

A

(C) No, because the corporation had its principal place of business in State A.

111
Q

A lawyer, a citizen of State A, brought an action against a doctor in the U.S. District Court in State B. The doctor was a citizen of State B. The action alleges $200,000 in damages and was based on an alleged malpractice by the doctor when he performed surgery on the lawyer. Subsequently, the doctor brought an action against the lawyer for $80,000 in a State A court of general jurisdiction for breach of contract when the lawyer did not purchase a luxury automobile from the doctor after signing an agreement to do so.

Which of the following is true?

(A) The state court cannot hear the doctor’s breach of contract claim.
(B) The doctor could not have added his breach of contract claim to the malpractice action.
(C) The doctor could, but was not required to, add his breach of contract claim to the malpractice action.
(D) The doctor was required to add his breach of contract claim to the malpractice action.

A

(C) The doctor could, but was not required to, add his breach of contract claim to the malpractice action.

112
Q

A supermarket was accused by a civil rights group of discriminatory hiring and promotion practices. The situation escalated when there was a violent incident at the protest rally being held by the civil rights group outside of the supermarket. The supermarket commenced an action against the civil rights group in the appropriate U.S. District Court, claiming $95,000 in damages as a consequence of the civil rights group’s conduct and violent actions of some of its members during the protest in violation of the National Labor Relations Act and applicable state law. The supermarket was a State A corporation, and the civil rights group was an unincorporated association which has members who were domiciled in every state. The civil rights group answered by denying the supermarket’s allegations and filing a $90,000 counter-claim of conversion asserting that some of the supermarket’s security guards burned down the headquarters of the civil rights group.

If the supermarket moves to dismiss civil rights group’s counterclaim for lack of subject-matter jurisdiction, is the court likely to grant the motion?

(A) Yes, because the supermarket has asserted a state cause of action.
(B) Yes, because diversity does not exist.
(C) No, if the civil rights group’s counterclaim is compulsory in nature.
(D) No, if the civil rights group’s counterclaim is permissive in nature.

A

(C) No, if the civil rights group’s counterclaim is compulsory in nature.

113
Q

After the pleadings had closed, a defendant filed a motion for judgment on the pleadings and asked for dismissal for failure to join an indispensable party. The plaintiff admitted that the absent party had an interest in the subject matter of the litigation and that as a practical matter, the absent party’s interest could be impaired by the disposition of the action. Joining the absent party would destroy diversity, the only possible basis for subject-matter jurisdiction over the case. All existing parties to the case and the absent party could be joined in one case in the state court system. The trial court granted the motion for judgment on the pleadings.

Did the trial court err when it granted the motion for judgment on the pleadings because of the failure to join the absent party?

(A) No, because the absent party could not be joined.
(B) No, because the plaintiff had an adequate remedy if it brought the case in state court after the judgment on the pleadings.
(C) Yes, because the defendant waived the defense by not making it in a motion to dismiss in response to the complaint.
(D) Yes, because the judgment on the pleadings would operate as an adjudication upon the merits.

A

(B) No, because the plaintiff had an adequate remedy if it brought the case in state court after the judgment on the pleadings.

114
Q

A man from State B was injured while driving a vehicle that he purchased from a manufacturer of hybrid technology cars whose principal place of business and place of incorporation were in State A. The man brought a state-law products liability action against the manufacturer in State B, claiming damages of $100,000. Two weeks after being served, the manufacturer removed the case to federal court. The man now seeks to have the case remanded back to state court.

How should the court rule?

A The case should be remanded, because it revolves exclusively around state law.
B The case should be remanded, because a federal court lacks subject-matter jurisdiction to hear the matter.
C The case should remain in federal court, because removal is the right of the defendant.
D The case should remain in federal court, because the defendant is permitted to remove a case that could have originally been brought in federal court.

A

The correct answer is: The case should remain in federal court, because the defendant is permitted to remove a case that could have originally been brought in federal court.

Discussion of correct answer: While a plaintiff is the master of his complaint (and decides the forum in which he will file a lawsuit), the defendant is not without any say in the matter. The procedure for removal allows a defendant to remove certain cases filed in state court to federal court. A notice of removal must be filed within 30 days after the defendant’s receipt of the initial pleading (or an amended pleading, if the pleading as amended gives rise to the grounds for removal) “through service or otherwise,” or within 30 days after service of the summons on the defendant, if the initial pleading is not required to be served on the defendant, whichever period is shorter. Here, the plaintiff could have sued in federal court, because there is complete diversity and the amount-in-controversy requirement is satisfied. Therefore, the defendant is allowed to remove within 30 days, which the facts indicate occurred here.

115
Q

A man, who lived in the Northern District of State B, sued a doctor, who lived in the Northern District of State C, and a lawyer, who lived in the Northern District of State A, for an accident that occurred in Canada. The man filed his suit in the Northern District of State A based on diversity jurisdiction. All parties agree to personal and subject-matter jurisdiction.

Is venue proper in this action?

(A) Yes, because the venue for foreign accidents is State A federal court.
(B) Yes, because the lawyer lives in the Northern District of State A.
(C) No, because the defendants are not residents of the same state.
(D) No, because a substantial portion of events occurred in Canada.

A

(B) Yes, because the lawyer lives in the Northern District of State A.

116
Q

Citizens of State A were plaintiffs in an action brought in the United States District Court in State B against Firstcorp, a State C corporation, and Secondcorp, a State B corporation. Firstcorp’s office and plant were located in State C. At no time had Firstcorp had an office or salesmen in State B. Secondcorp’s sole place of business was in State B. The complaint alleged that each of the plaintiffs sustained serious personal injuries when a toaster oven exploded while a clerk was showing it to the plaintiffs in Secondcorp’s store in State B. Each plaintiff requested damages in the sum of $80,000.

The toaster oven had been manufactured by Firstcorp and shipped to Distributor in State A. Distributor had a contract with Firstcorp to act as the exclusive distributor of Firstcorp products in seven states, including State A and State B. Process was served personally on the president of Firstcorp at Firstcorp’s office in State C and on the president of Secondcorp at its office in State B. Thereafter, Firstcorp moved to dismiss the action on the ground that the court had no jurisdiction over it.

Under what circumstances can the U.S. District Court assert personal jurisdiction over Firstcorp?

(A) If State B has an appropriate long-arm statute and the assertion of personal jurisdiction would comport with due process.
(B) If the assertion of personal jurisdiction would comport with due process (there being nationwide service of process in actions commenced in federal court).
(C) If the amount in controversy exceeds $75,000, exclusive of interest and costs.
(D) If process was served upon Firstcorp in accordance with both federal and State B law.

A

(A) If State B has an appropriate long-arm statute and the assertion of personal jurisdiction would comport with due process.

117
Q

A man is a citizen of State A and filed a defamation action in federal court against a reporter and an editor, both of whom were citizens of State B. The claims against the two defendants, the reporter and the editor, arose from the same events. Plaintiff alleged that the amount in controversy with respect to each defendant was $100,000. State law caps damages in this type of case against defendants like the editor at $25,000. The editor filed a motion to dismiss for lack of subject matter jurisdiction.

Should the court grant the motion to dismiss for lack of subject matter jurisdiction?

(A) Yes, because there is an insufficient amount in controversy between the man and the editor.
(B) Yes, because the reporter and the editor are from the same state.
(C) No, because there is supplemental jurisdiction over the claim against the editor.
(D) No, because the Court looks to the man’s allegations to determine the amount in controversy.

A

(A) Yes, because there is an insufficient amount in controversy between the man and the editor.

118
Q

LLC, a limited liability corporation incorporated and doing business in State A, sued CO, a company incorporated and doing business in State B, in federal district court for $150,000 in damages to LLC’s products as a result of faulty snow-making equipment. LLC served the summons and complaint on CO. In its complaint, LLC enumerated each product that was damaged, how it was damaged, and when. In its answer, CO denied each allegation. After engaging in discovery practice, including taking two depositions and reviewing several documents, CO filed a motion for summary judgment. LLC filed its response, pointing to all of the specific information delineated in its complaint. Will the federal district court grant CO’s motion for summary judgment?

(A) Yes, because CO has provided evidence to prove its case through summary judgment motion practice.
(B) Yes, because LLC cannot rely on allegations in the complaint, even if the damages were specifically delineated therein.
(C) No, because LLC gave specific information in its complaint regarding each product that was damaged by CO’s faulty snow-making equipment.
(D) No, because LLC raised an issue of fact in its response to the motion for summary judgment.

A

(B) Yes, because LLC cannot rely on allegations in the complaint, even if the damages were specifically delineated therein.

119
Q

A buyer filed an action against a seller corporation for breach of contract and misrepresentation in the sale of a classic race car. An agent of the seller allegedly represented to the buyer that the vehicle was in excellent condition and ready to be driven. However, the buyer found that: (1) the steering was defective; and (2) there was a problem with the transmission. Prior to filing the action, the buyer’s attorney consulted a former race car driver concerning the buyer’s damages. The attorney planned to call the race car driver to testify as an expert witness during the trial. Which of the following is true regarding the potential discovery request of the former race car driver?

(A) The opinion of an expert who will be called to testify is discoverable.
(B) Discovery of the former race car driver’s opinion cannot be obtained because it constitutes work product.
(C) The opinion of an expert is discoverable only if it is solicited or obtained prior to the commencement of litigation.
(D) Discovery of an expert’s opinion is privileged.

A

(A) The opinion of an expert who will be called to testify is discoverable.

120
Q

An employee worked for a knife factory in State A. The factory was owned by a State B corporation with its principle place of business and headquarters located in State A. The machine operated by the employee began to make loud noises and smoke. The employee was injured when the machine exploded. He brought suit against the corporation in federal district court in State A. Service of process was sent to the residence of the corporation’s president. Service was accepted by the president’s 18-year-old son. The corporation filed two motions. The first motion, made one week after process was served, was made under Fed. R. Civ. P. 12(b)(1) for lack of subject-matter jurisdiction. The second motion, filed a day later, was brought under Rule 12(b)(5) for insufficient service of process. The court denied the second motion, stating that the corporation had waived its right to complain about service of process. Was the court’s action proper?

(A) No, because service of process must be made to the agent for service of process.
(B) No, because the motion was made within 20 days of service of process.
(C) Yes, because the son was 18 years old and had the capacity to accept service of process.
(D) Yes, because the corporation waived its right to object to service of process.

A

(D) Yes, because the corporation waived its right to object to service of process.

121
Q

A woman filed suit in federal court against a stockbroker alleging common law claims of fraud in connection with an investment the woman made with the stockbroker. The woman anticipated that the stockbroker would claim that compliance with the federal laws that govern the type of investment the woman made with him provided a defense to the woman’s claims. The woman alleged in the complaint that this argument was based upon a misunderstanding of the federal law. There was no diversity of citizenship between the parties. The stockbroker filed a motion to dismiss for lack of subject-matter jurisdiction.

Should the court grant the motion to dismiss for lack of subject-matter jurisdiction?

(A) Yes, because the stockbroker has not raised a federal defense.
(B) Yes, because the federal issue in the complaint is not a necessary element of a cause of action for fraud.
(C) No, because the woman alleged the federal issue in the complaint.
(D) No, because the interpretation of the federal law will be important to the outcome of the case.

A

(B) Yes, because the federal issue in the complaint is not a necessary element of a cause of action for fraud.

122
Q

In what circumstances would a writ of habeas corpus be inappropriate?

A A prisoner’s claim that his attorney slept during the trial.
B A prisoner’s claim that his confession at trial was given in violation of his Miranda rights.
C A prisoner’s claim that his solitary confinement is cruel and unusual.
D A prisoner’s claim that a search of his home was made in violation of his Fourth Amendment rights.
Explanation
The correct answer is: A prisoner’s claim that a search of his home was made in violation of his Fourth Amendment rights.

Discussion of correct answer: Federal courts have subject-matter jurisdiction to review convictions of prisoners in state court through a writ of habeas corpus [28 U.S.C. Sec. 2254(a)]. A writ of habeas corpus is a civil cause of action brought against the jailor or custodian of a person placed in confinement. It does not operate to appeal or continue the criminal case against the prisoner in any manner, nor does it operate to make a determination of guilt or innocence as to the prisoner. The only question is whether the confinement of a person is in violation of his or her constitutional rights under the Fifth, Eighth, or Fourteenth Amendments to the U.S. Constitution. Here, the prisoner’s claim of a Fourth Amendment violation does not create an appropriate ground for a writ of habeas corpus.

A

The correct answer is: A prisoner’s claim that a search of his home was made in violation of his Fourth Amendment rights.

Discussion of correct answer: Federal courts have subject-matter jurisdiction to review convictions of prisoners in state court through a writ of habeas corpus [28 U.S.C. Sec. 2254(a)]. A writ of habeas corpus is a civil cause of action brought against the jailor or custodian of a person placed in confinement. It does not operate to appeal or continue the criminal case against the prisoner in any manner, nor does it operate to make a determination of guilt or innocence as to the prisoner. The only question is whether the confinement of a person is in violation of his or her constitutional rights under the Fifth, Eighth, or Fourteenth Amendments to the U.S. Constitution. Here, the prisoner’s claim of a Fourth Amendment violation does not create an appropriate ground for a writ of habeas corpus.

123
Q

Railroad Corporation has brought a class action in the U.S. District Court for the Eastern District of Ohio against the members of the United Railroad Firemen’s Union (“Union”). The class action complaint names the four officers of the Union as the class representatives. Railroad is a Pennsylvania corporation which has its principal place of business in New York. The Union is an unincorporated association which has its headquarters in Ohio, which is also the place where the major portion of the Union’s administrative work is performed. All of the officers of the Union are citizens of Pennsylvania. The membership of the Union includes railroad employees who reside in Michigan, Pennsylvania, and Ohio. The action asserts that members of the Union destroyed railroad property in Pennsylvania during a strike. Which of the following is most accurate?

(A) Venue is proper only in Ohio, because that is where the Union is doing business, and would therefore be considered its residence.
(B) The action cannot be brought in a U.S. District Court unless it is based upon a federal claim.
(C) The action is outside the subject-matter jurisdiction of federal courts, because it is an action involving an unincorporated labor organization.
(D) Venue is proper only in Pennsylvania, because all of the officers of the Union are citizens of Pennsylvania.

A

(B) The action cannot be brought in a U.S. District Court unless it is based upon a federal claim.

Discussion of correct answer: The citizenship of an unincorporated association is the citizenship of all the members of the association. As such, the Union’s citizenship would be Michigan, Pennsylvania, and Ohio. Because the plaintiff and the defendant are citizens of the same state (Pennsylvania), diversity does not exist. Thus, the plaintiff’s action would have to be predicated upon a federal claim to avoid dismissal. If, however, the plaintiff’s claim were based upon a federal statute, no diversity requirements would exist.

124
Q

A woman who is a citizen of State A was dating a man who was a citizen of State B for two years. They broke up and she sued the man when he would not return some jewelry that she claimed was hers. The woman mailed the service and complaint to the man at his residence in State B, along with a request for waiver of the service. The man chose not to agree to the woman’s request for waiver of service.

Assuming the federal court has jurisdiction over the matter, may the man choose not to agree to the waiver of service?

A Yes, but he may be liable for expenses and attorneys’ fees.
B Yes, because the Federal Rules of Civil Procedure guarantee his right to service in accordance with federal law.
C No, because mail is a reasonable means of apprizing the man of the pending action.
D No, because the man is an individual and not a corporation, partnership, or association.

A

The correct answer is: Yes, but he may be liable for expenses and attorneys’ fees.

Discussion of correct answer: If a defendant located within the United States fails without good cause to sign and return a waiver requested by a plaintiff located within the United States, the court may impose on the defendant: (a) expenses incurred in making the service; and (b) reasonable expenses, including attorneys’ fees, of any motion required to collect service expenses. Since the question does not indicate the man had a good cause to refuse waiver, Answer (A) is the best answer.

125
Q

A man sued a trucker for negligence. The man took a summons and a copy of the complaint and walked into a bar where he knew the trucker was drinking. The man handed the summons and a copy of the complaint directly to the trucker as soon as he saw him in the bar.

Assuming the case was properly filed in federal court, was service of process proper?

A Yes, because the man gave the trucker a summons and copy of the complaint by hand.
B Yes, because the plaintiff may serve process by any means reasonably calculated to apprize interested parties of the pendency of the action.
C No, because the man is a party to the lawsuit.
D No, because there is no indication the summons was presented to the clerk for signature and seal.

A

The correct answer is: No, because the man is a party to the lawsuit.

Discussion of correct answer: Service may be properly made by any person who is: (A) not a party to the suit and (B) over the age of 18 years old. Here, since the man is a party to the suit, he may not personally deliver the summons and complaint to the trucker.

126
Q

On March 1, an employee filed a complaint against her employer in state court, claiming that the employer did not pay all of the required contributions into her health benefit plan. The summons and complaint were served on the employer on March 14. The employee was a member of a union. The union had a contract with the employer that provided for health benefit fund contributions to be made on behalf of the employer’s employees. A fund is the entity that administered the health benefit plan for the employees of the employer. The health benefit plan and the fund were created and administered in accordance with the terms of ERISA, a federal statute. This federal statute created causes of action and described standing for those who had claims associated with such plans.

On March 8, the fund filed a complaint against the employer in federal court to collect delinquent contributions owed pursuant to the contract between the union and the employer. This lawsuit was brought in accordance with the terms of ERISA. Contributions owed on behalf of the employee to the health benefit plan were included in the fund’s lawsuit against the employer. On April 10, the employer filed a notice of removal of the employee’s case to federal district court.

Is the removal proper?

A Yes, because the employee’s case is based on federal-question subject-matter jurisdiction.
B Yes, because judicial efficiency is best served by having both cases against the employer be heard in federal court.
C No, because the employer filed its notice to remove outside the permitted time limits.
D No, because the employee is entitled to bring a lawsuit to enforce her rights.

A

The correct answer is: Yes, because the employee’s case is based on federal-question subject-matter jurisdiction.

Discussion of correct answer: The employee’s case could have originally been brought in federal court. Further, the employer filed its notice of removal in a timely fashion. Therefore, removal was proper.

127
Q

A plaintiff brought an action in federal court for assault and battery against a diverse defendant claiming $15,000 in actual damages, $12,500 in pain and suffering, $12,500 in lost wages, and $36,000 in punitive damages. The defendant made a motion to remand the case to state court.

How should the court proceed?

A Deny the request, because the amount in controversy is satisfied.
B Deny the request, because a plaintiff can aggregate multiple claims against a single defendant.
C Grant the request, because punitive damages cannot be used towards the amount in controversy for a case arising out of diversity.
D Grant the request, because although punitive damages are counted towards the amount in controversy, they cannot account for the majority of the total amount.

A

The correct answer is: Deny the request, because the amount in controversy is satisfied.

Discussion of correct answer: Punitive damages can be used in calculating the amount in controversy provided that they are available for the particular claim being asserted. As punitive damages are allowed in most tort cases, this answer choice is correct.

128
Q

A State A man was abandoned by his wife who left their home to move to State B. She filed for divorce against the man in State B, but she did not serve process on him. The man had never been to State B and had no contacts with the state. Meanwhile, the man’s father received a letter that led him to believe he had won a million dollar lottery and that he needed to go to State B to claim his winnings. The man accompanied his father to State B to claim the winnings, although the man did not believe the father had actually won anything. After they arrived in State B, the father found out that he had not won the lottery, and the company that had sent him the letter was later shut down for sending letters fraudulently enticing people to come to State B. While in State B, however, the man was served by his wife with a summons related to the State B divorce proceedings.

If the man argues that he is not subject to personal jurisdiction in State B, will he prevail?

A No, because he was served while physically present in State B.
B No, because he consented to personal jurisdiction in State B.
C Yes, because his presence in State B was procured by fraud.
D Yes, because he did not have minimum contacts with State B.

A

The correct answer is: No, because he was served while physically present in State B.

Discussion of correct answer: A state has personal jurisdiction over a party when that person is served while physically present in the state, even if the person is only there temporarily. This does not apply when the plaintiff has fraudulently enticed the party to enter the state. While the lottery company may have committed fraud which caused the man to come to State B, the wife played no part in this fraud and therefore there is personal jurisdiction over the man.

129
Q

A man was injured while riding a bus. The bus company operates nationwide and was concerned that their reputation would be tarnished if the case was litigated, so they settled with the man out of court. In the settlement, the man was given free bus rides anywhere in the country for life.

Several years after the settlement was reached, the National Transportation Safety Board passed a regulation precluding nationwide passenger carriers from providing free transportation. The ruling was retroactive and covered airlines, cruise ships, trains, and bus services. The defendant then notified the man that it could no longer allow him to ride the bus for free as originally agreed upon. The man subsequently brought suit in federal court against the company. The defendant moved that the case should be dismissed because the federal court lacked subject-matter jurisdiction.

Which of the following is true?

A The motion should be denied, because the matter involves interstate commerce, which is an exclusively federal power, and therefore should be heard in federal court.
B The motion should be denied, because a central issue in the case is the federal statute.
C The motion should be granted, because there is no federal question presented.
D The motion should be granted, because defendants can remand a case to state court if they choose.

A

The correct answer is: The motion should be granted, because there is no federal question presented.

Discussion of correct answer: A suit arises under the United States Constitution and laws of the United States only when the plaintiff’s statement of his own cause of action shows that it is based upon those laws or the Constitution. It is not enough for a plaintiff to allege an anticipated defense to his cause of action and assert that the defense is invalidated by some provision of the United States Constitution. These facts are similar to the Mottley decision [Louisville & Nashville Railroad Company v. Mottley, 211 U.S. 149 (1908)]. In that case, the court found that the defendant’s allegations showed that in the course of litigation a question under the United States Constitution would arise, but this did not satisfy the court’s requirement for federal-question jurisdiction that the plaintiff’s original cause of action arise under the Constitution.

130
Q

A retirement fund filed a suit in federal district court against a corporation under ERISA, a federal statute, for contributions the corporation owed to the fund on behalf of three corporation employees. The corporation took depositions of the three non-party employees. According to the employees’ testimony, the corporation did not owe any contributions. The corporation filed a motion for summary judgment, using the deposition transcripts of the three employees. In its motion, the corporation stated that, based on the employees’ testimony, under the law, the corporation did not owe any contributions. The fund responded to the motion with additional evidence showing that the corporation did, in fact, owe contributions on behalf of the three employees. Will the federal district court grant the corporation’s motion for summary judgment?

(A) Yes, because the three employees for whom contributions are at issue testified at their depositions that no contributions were owed.
(B) Yes, because according to the law, the corporation does not owe any contributions.
(C) No, because the motion for summary judgment and response bring up an issue of fact that cannot be resolved through a summary judgment motion.
(D) No, because the employees’ deposition testimony was insufficient to prove that the corporation did not owe any contributions.

A

(C) No, because the motion for summary judgment and response bring up an issue of fact that cannot be resolved through a summary judgment motion.

131
Q

A freelance graphic designer brought a lawsuit in federal court for breach of contract against a graphic design firm. The graphic design firm responded with a counterclaim for tort damages arising out of the designer’s alleged negligence in leaving a window at the firm’s office open during a winter storm, resulting in water damage to the firm’s equipment. Prior to the commencement of the trial, the graphic designer moved for summary judgment as to the design firm’s counterclaim. May the court rule on the graphic designer’s motion for summary judgment?

(A) Yes, because the counterclaim constitutes an issue on which the court may make a summary judgment ruling.
(B) Yes, because the counterclaim is a permissive counterclaim.
(C) No, because the motion is being made prior to the commencement of the trial.
(D) No, because the motion is not being made with respect to the principal cause of action.

A

(A) Yes, because the counterclaim constitutes an issue on which the court may make a summary judgment ruling.

132
Q

A citizen of State A filed a strict liability claim for $30,000 against a State B corporation in a State A state court. The citizen alleged that the corporation manufactured a car with defective breaks which caused the citizen to crash while driving the car. A year later, she amended her complaint to add a state negligence claim for $50,000. Twenty-two days after the amended pleading was filed, the corporation filed notice of removal in federal court based on diversity jurisdiction.

Can the corporation have the action removed to federal court?

A No, because over a year has passed since the claim was commenced.
B No, because the notice of removal was filed more than 20 days after the amended pleading was filed.
C Yes, because the amount in controversy now totals over $75,000.
D Yes, because the federal court has subject-matter jurisdiction over the matter.

A

The correct answer is: No, because over a year has passed since the claim was commenced.

Discussion of correct answer: A case removable on the basis of diversity jurisdiction may not be removed more than one year after the action was commenced [28 U.S.C. Sec. 1446]. Here, the claim could be removed based on complete diversity of citizenship and an amount in controversy over $75,000, but over one year has passed since the citizen filed her claim.

133
Q

An architect who is a citizen of State B has a dispute on a $2,000,000 contract with a client who is a citizen of State A. The client wants to file in federal district court in State A. When he finds out from the architect’s assistant that the architect is about to take a vacation to State A to attend a family reunion, the client arranges for a professional process server to serve the architect while she is at her family reunion.

Assuming that the contract does not specify where a contract claim is to be heard, will a federal district court in State A have jurisdiction over the architect?

(A) Yes, because the architect’s trip to State A means that she has substantial contacts with the state.
(B) Yes, because the architect was served with process while in the forum state.
(C) No, because the architect does not intend to remain in State A indefinitely.
(D) No, because the architect was not a State A resident at the time of service.

A

(B) Yes, because the architect was served with process while in the forum state.

134
Q

Two plaintiffs sued a defendant in federal court for a state law tort cause of action for which there was no statutory cap on damages. The plaintiffs each alleged that more than $75,000 was in controversy as to his claim, and demanded a judgment for more than $75,000, but neither pled these damages with particularity. The first plaintiff was domiciled in State B but moved before the suit was filed to State A. This first plaintiff resided in State A at the time the suit was filed but at all times intended to return to State B upon the completion of a temporary work assignment. The second plaintiff was domiciled at all times in State A. The defendant was a corporation incorporated under the laws of State C with its principal place of business in State B. Just before trial, after two years of pleadings, discovery, and a final pretrial conference, the defendant filed a motion to dismiss for lack of subject-matter jurisdiction. The court granted the motion.

Was the trial court correct to grant the motion to dismiss for lack of subject-matter jurisdiction?

(A) Yes, because there was incomplete diversity of citizenship.
(B) Yes, because the plaintiffs did not plead the amount in controversy with particularity.
(C) No, because the motion was filed too late.
(D) No, because the first plaintiff resided in State A at the time the suit was filed.

A

(A) Yes, because there was incomplete diversity of citizenship.

135
Q

An opera singer sued a theater company in federal court for breach of contract. Feeling that the opera singer’s lawsuit was meritless, the theater company made a motion for summary judgment immediately after the parties’ initial discovery conference. In opposing the motion, the opera singer’s lawyer argued that the motion was not timely filed. The court in which the lawsuit was filed follows the Federal Rules of Civil Procedure. Is the motion untimely?

(A) No, because the motion was made prior to the close of discovery.
(B) No, because the motion may be made at any time.
(C) Yes, because there is not enough evidence on which to base a summary judgment ruling.
(D) Yes, because 30 days have not elapsed since the discovery conference.

A

(A) No, because the motion was made prior to the close of discovery.

136
Q

A golf pro at a private prep school was fired. The golf pro believed that he was being discriminated against because of his age, and filed suit against the school for unlawful termination based on a state law prohibiting age discrimination. The golf pro, whose salary was $35,000 a year, requested damages of $350,000, arguing that he planned to work for another 10 years. Associates of the golf pro were prepared to testify that he had planned to stay at the job for only another two years before leaving to complete a long-deferred graduate degree. The golf pro filed suit in the state court of State X, where all parties were domiciled. The school planned to defend the suit by arguing that it fired the golf pro based on a new federal regulation prohibiting schools from employing anyone who had been convicted of multiple federal drug possession crimes, as the golf pro has been.

Can the school remove the case to federal court?

A Yes, because there is a federal question presented.
B Yes, because the golf pro has successfully shown the amount in controversy.
C No, because the golf pro has failed to show the amount in controversy.
D No, because there is no federal question presented.

A

The correct answer is: No, because there is no federal question presented.

Discussion of correct answer: For a federal court to have jurisdiction, there must be either a federal question or diversity jurisdiction. Here, the parties are not diverse. Although the defendants plan on arguing that federal law was the reason that the plaintiff was fired, federal question jurisdiction requires that the federal question appear on a fair reading of the complaint. Because the plaintiff did not invoke federal law, there will be no federal question jurisdiction.

137
Q

A man and woman attended a local art show. The art show featured a world-renowned photographer who was famous for taking pictures of people he saw on his daily walk around the city. The artist would then paint slogans on the photos. As the man and woman walked around the show, they discovered that the artist had taken a picture of them while they were dressed up in matching American flag shirts. Below the couple was painted the slogan “War Pigs.” The woman became so upset that she fainted, injuring herself when she hit the ground. The man and woman joined as plaintiffs under Rule 20 and sued the artist in federal court. The man was a citizen of Connecticut, the woman was a citizen of Rhode Island, and the artist was a citizen of New York. The man’s claim was for $10,000 under the Federal Misuse of Likeness Act, which made it a federal crime to use another person’s image for monetary gain. The woman’s claim was for $20,000 under state laws for intentional infliction of emotional distress. The artist moved to dismiss the woman’s claim for lack of subject-matter jurisdiction. How should the court rule?

(A) The court should grant the motion, because the amount in controversy was only $20,000.
(B) The court should grant the motion, because the woman’s claim was based on state law.
(C) The court should deny the motion, because the woman’s claim arose from the transaction or occurrence underlying the man’s claim.
(D) The court should deny the motion, because the woman’s claim, in effect, alleged a violation of a federal law.

A

(C) The court should deny the motion, because the woman’s claim arose from the transaction or occurrence underlying the man’s claim.

138
Q

Tax Inc. franchised thousands of tax preparation offices nationwide. Ten years ago, a franchisee signed a franchise agreement with Tax Inc. granting him the right to operate Tax Inc. franchises in Charleston, West Virginia. Believing that the franchisee breached his original agreement and was using his franchise to further a rival tax preparation business, Tax Inc. filed a request for a permanent injunction in federal court. In its complaint, Tax Inc. correctly identified the franchisee as a citizen of West Virginia and itself as a citizen of Delaware, where it was incorporated, and New York, where it had its primary place of business. In determining the cost of the injunction, Tax Inc. focused on the alleged improper use of the franchise, and valued the stopping of that improper use at $80,000. The franchisee calculated the injunction’s cost to be $25,000. He derived this figure from the remaining five years on his franchise agreement, arguing that he had made $5,000 per year in profit for the previous 10 years of the agreement and was likely to continue to do so for the remaining five years. The federal district court then dismissed Tax Inc.’s complaint for lack of subject-matter jurisdiction, finding that it failed to meet the $75,000 amount-in-controversy requirement for diversity jurisdiction. Tax Inc. appealed. Did Tax Inc.’s complaint meet the necessary amount in controversy to be heard in federal court?

a. Yes, because the value of an injunction is determined by the plaintiff.
b. Yes, because the value of an injunction is determined by the larger of the injunction’s worth to the plaintiff or its cost to the defendant.
c. No, because lost profits is the correct way to determine the value of an injunction.
d. No, because Tax Inc.’s $80,000 figure was too speculative.

A

b. Yes, because the value of an injunction is determined by the larger of the injunction’s worth to the plaintiff or its cost to the defendant.

139
Q

A woman filed suit in federal court against a stockbroker alleging common law claims of fraud in connection with an investment the woman made with the stockbroker. The woman anticipated that the stockbroker would claim that compliance with the federal laws that govern the type of investment the woman made with him provided a defense to the woman’s claims. The woman alleged in the complaint that this argument was based upon a misunderstanding of the federal law. There was no diversity of citizenship between the parties. The stockbroker filed a motion to dismiss for lack of subject-matter jurisdiction.

Should the court grant the motion to dismiss for lack of subject-matter jurisdiction?

(A) Yes, because the stockbroker has not raised a federal defense.
(B) Yes, because the federal issue in the complaint is not a necessary element of a cause of action for fraud.
(C) No, because the woman alleged the federal issue in the complaint.
(D) No, because the interpretation of the federal law will be important to the outcome of the case.

A

(B) Yes, because the federal issue in the complaint is not a necessary element of a cause of action for fraud.

140
Q

A citizen of State A was injured in an auto accident by a citizen of State B. The accident occurred in State A. Pursuant to the applicable State A long-arm statute, the plaintiff sued the defendant and the defendant’s auto insurance company in a State A state court of general jurisdiction for $80,000. The defendant’s auto insurance company, which was joined with the defendant under an applicable state law, is incorporated in State D and has its principal place of business in State A. Both defendants were served with process on the same day. Twenty-nine days after being served, both defendants filed a joint notice of removal to the applicable U.S. District Court. The plaintiff argued that a federal court did not have jurisdiction and then filed a motion for remand.

How should the court rule on the plaintiff’s motion?

A The court should deny the plaintiff’s motion, because the amount-in-controversy requirement is satisfied.
B The court should deny the plaintiff’s motion, because there is complete diversity and the amount in controversy is satisfied.
C The court should grant the plaintiff’s motion, because the notice of removal was not filed in a timely fashion.
D The court should grant the plaintiff’s motion, because the defendant auto insurance company’s principal place of business is in State A.

A

The correct answer is: The court should grant the plaintiff’s motion, because the defendant auto insurance company’s principal place of business is in State A.

Discussion of correct answer: A corporation is deemed to be a citizen of the state in which it is incorporated and the jurisdiction in which it has its principal place of business. As a result of this rule, removal would not be possible for two reasons. The first reason is because the auto insurance company’s presence in the dispute destroys complete diversity. The second reason is because defendants sued in their home state cannot remove if the basis for removal would be diversity of citizenship.

141
Q

A man sued a public relations firm that is owned equally by three brothers as a partnership. The man is claiming that the public relations firm cost the man’s company over $100,000 by making defamatory remarks in a press release. The man and his business were citizens of State A. The main offices of the partnership were in State B, but the three partners’ homes were split between State A, State B, and State C.

Can the man bring his lawsuit in federal court based on diversity jurisdiction?

A Yes, because the amount in controversy is over $75,000.
B Yes, because the partnership is a citizen of State B, where its primary place of business is located.
C No, because one of the partners lives in State A.
D No, because the loss to the business occurred in State A.

A

The correct answer is: No, because one of the partners lives in State A.

Discussion of correct answer: The citizenship of an unincorporated association, such as a partnership, limited partnership, or limited liability company, is the citizenship of all of the members of the association. Consequently, since one of the partner’s lives in State A, this will defeat complete diversity of the parties.

142
Q

The plaintiff, a citizen of State A, went on a business trip to State B and stayed in a hotel. While there, he was injured while using an exercise machine in the hotel’s gym. He filed a tort suit in State A state court against the hotel and the technician (an independent contractor) who serviced the gym equipment. The man alleged a violation of State B law and sought $100,000 in damages. The hotel and the technician are citizens of State B for the purposes of diversity jurisdiction.
One week after the hotel and technician were served with process, the hotel’s lawyer sought to remove the case to federal court, but the technician did not wish to remove the action. Two months later, the plaintiff joined the manufacturer of the exercise machine as a defendant. Several weeks after being served, the manufacturer, a resident of State C, wished to remove the case to federal court and persuaded the technician and hotel to join in a notice of removal. The manufacturer filed a notice of removal exactly four weeks after being served, and the hotel and technician filed a notice that same day joining in the removal. May the suit be removed under these circumstances?

a. Yes, because the defendants have the right to remove the action at any time prior to trial.
b. Yes, because the manufacturer’s removal was timely and the hotel and technician were free to join in manufacturer’s removal.
c. No, because the manufacturer waited too long to initiate its removal.
d. No, because the manufacturer waited too long to initiate its removal.

A

Yes, because the manufacturer’s removal was timely and the hotel and technician were free to join in manufacturer’s removal.

Discussion of correct answer: Notice of removal must be filed within 30 days of when the grounds for removal become apparent. Normally, this is when process is served, but it could be later, such as when an amendment is made. If a defendant declines to remove within this 30-day window, he may not initiate a removal at a later time. However, he may join in a removal initiated by a defendant that is served at a later time. Here, removal did not occur within the 30-day window applicable to the hotel or technician based on their service. (While the hotel desired to remove, all defendants must agree to removal for removal to occur.) However, when the manufacturer was served and elected to remove within the 30-day window applicable to it, the hotel and technician were free to join in the removal within that same 30-day window. Thus, removal is permissible here.

143
Q

A man was the victim of an attack at a truck stop in State B by the defendant who was motivated by his hatred of the man’s race. The man, a citizen of State A, filed suit in the U.S. District Court in State A alleging a federal claim against the defendant who was a citizen of State C. The defendant moved to change the venue to State B or, alternatively, to State C.

What outcome is likely?

A The action may be transferred to State B only.
B The action may be transferred to State B or State C.
C The action must be dismissed, because it was commenced in an improper forum.
D It is discretionary with the court whether to retain the action or transfer it to State B or State C.

A

The correct answer is: The action may be transferred to State B or State C.

Discussion of correct answer: Because the action could have been commenced in either State C (where the defendant resides) or State B (where the racial attack took place), it may be transferred to the proper federal judicial district in either of those states.

144
Q

A company sued one of its former employees in state court for defamation because of the scathing reviews of the company’s services that she wrote in her blog. The former employee defended by arguing that the company violated federal law when it fired her because it only did so to put a younger person in her position. The relevant statute expressly created a right for employees to sue for damages for acts of age discrimination. The former employee sought to remove the case to federal court.

May the case be heard in federal court?

A No, because the company first sued the employee for defamation.
B No, because the discrimination claim is not closely linked to the defamation claim.
C Yes, because the employee is claiming discrimination as part of her defense.
D Yes, because the employee’s defense is based on a federal right.

A

The correct answer is: No, because the company first sued the employee for defamation.

Discussion of correct answer: To be heard in federal court, the federal question must appear in a fair reading of a well-pleaded complaint. Here, the plaintiff (the company) is simply suing the employee on a state defamation claim. Consequently, the complaint does not create a federal question.

145
Q

A teacher, who was a citizen of State A, filed an action in the U.S District Court in State A against a stockbroker, a State B citizen, and his firm, a State C corporation, alleging that they violated SEC federal regulations and perpetrated a fraud during the sale of securities to the teacher. The teacher claimed that his total damages from the fraud were $66,000. The teacher further alleged in a second cause of action that the stockbroker did not repay an $8,000 loan that the teacher made to him.

Which of the following is true?

(A) The action should be dismissed by the District Court, because the value of the aggregated claims does not meet the amount-in-controversy requirement.
(B) The District Court may exercise supplemental jurisdiction over the second cause of action, but only if it arises from a common nucleus of operative facts with the first cause of action.
(C) The federal court has subject matter jurisdiction, because there is complete diversity of citizenship between all plaintiffs and defendants.
(D) The federal court lacks subject matter jurisdiction, because the violation of SEC federal regulations must be tried in the state court where such violations allegedly occurred.

A

(B) The District Court may exercise supplemental jurisdiction over the second cause of action, but only if it arises from a common nucleus of operative facts with the first cause of action.

146
Q

A woman is walking her dog when the woman was struck by a car driven by an employee of a delivery company that is owned by a businessman. The woman sued both the employee and the businessman for her injuries. The woman was a citizen of State A. The employee was a citizen of State B and the businessman was a citizen of State C. The woman claimed over $100,000 in damages. The woman filed the lawsuit in State C state court. Twenty-eight days after service of the initial pleading, the employee and the businessman filed notice of removal in federal court.

Can the action be removed to federal court?

A Yes, because the federal court has subject-matter jurisdiction.
B Yes, because the notice of removal was filed within 30 days of service of the initial pleading.
C No, because the businessman is a citizen of State C.
D No, because the notice of removal was not filed within 25 days of service of the initial pleading.

A

The correct answer is: No, because the businessman is a citizen of State C.

Discussion of correct answer: An action in state court may be removed by the defendant to federal court if the case originally could have been brought by the plaintiff in federal court [28 U.S.C. Sec. 1441]. However, there is an exception to this rule where the request for removal is on diversity jurisdiction grounds. A request for removal on grounds of diversity jurisdiction will not be granted if any of the defendants in the case is a citizen of the forum state. In that case, the case is removal-proof and will remain in state court. Here, the request for removal would be based on diversity jurisdiction since the woman is making a state tort claim, there is complete diversity of citizenship between the parties, and the amount in controversy is over $75,000. However, since the businessman is a citizen of the forum state, the action cannot be removed to federal court.

147
Q

A lawyer in Nevada contracted with a surgeon to perform over $80,000 worth of plastic surgery. When the lawyer failed to pay, the surgeon sued. However, because the surgeon moved to California after the service was completed, he filed the action in California. The lawyer’s contacts with California are minimal, and the surgeon is unsure whether they will suffice for personal jurisdiction. On a previous occasion, the lawyer drove through California to attend a conference in the state capitol, and he paid for a hotel room and other expenses there. The lawyer had also considered moving there at one time, and he had submitted his resume online to some employers in California.

Did the lawyer implicitly consent to personal jurisdiction in California?

(A) Yes, because he operated a motor vehicle in California.
(B) Yes, because applying for employment within California showed his intent to be domiciled there.
(C) No, because his contacts with California were minimal.
(D) No, because this suit does not arise out of his contacts with California.

A

(D) No, because this suit does not arise out of his contacts with California.

148
Q

Denny, a citizen of Texas, leased an apartment from Polly, a citizen of California, under a one-year rental agreement at $500 per month. The rental agreement stated that Denny could sublet the premises to any person who is not of Oriental ancestry. With six months remaining on the lease, Denny sublet the apartment to a Chinese person. Polly sued Denny in the appropriate U.S. District Court for breach of contract, seeking to evict Denny and recover the remaining six months’ rent. The complaint stated that Denny violated the terms of the rental agreement and asserts that the clause concerning subletting does not violate the U.S. Constitution. Denny moves to dismiss the case for lack of subject-matter jurisdiction, asserting a lack of diversity jurisdiction and the absence of a federal claim. How should the court rule on Denny’s motion to dismiss?

(A) Denny’s motion will be granted, because a federal claim is lacking.
(B) Denny’s motion will be granted, because diversity is lacking.
(C) Denny’s motion will be granted, because there is neither a federal claim nor diversity jurisdiction
(D) Denny’s motion will be denied.

A

(C) Denny’s motion will be granted, because there is neither a federal claim nor diversity jurisdiction

Discussion of correct answer: A state cannot exercise jurisdiction over a nonresident defendant if that party has only sporadic and inadvertent contacts with the state. Nonresidents must be protected from the burden of defending a suit in a distant and inconvenient court. In this case, the defendant bank had no office nor transacted business in State C. This cause of action did not arise from acts or transactions in State C. Choice (C) is incorrect as there was nothing the State B bank did to reach out to State C, or even the woman, as they were unaware of her having moved. Choice (D) is incorrect because more is required than just foreseeability of being sued, rather, it is about the foreseeability that the defendant would be haled into court in State C. Based on these facts, it was not foreseeable.

149
Q

A woman from State A had worked for a law school in neighboring State B for 30 years. Each week, she would deposit her check into the savings account she maintained at a State B bank. Upon her retirement, the woman moved to State C, where she died five years later. Her daughter, named as executrix of her estate, discovered that the account with the State B bank still existed. This was a surprise to the State B bank as well, because the account statements sent to the woman at her original State A address had come back as undeliverable for the past five years. The bank never learned that the woman moved to State C until the woman’s daughter came forward to notify it of the woman’s death.

The daughter believes that the bank failed to pay proper interest on the account, and has since sued the bank in State C. The bank claims that it is not subject to personal jurisdiction in State C, as it has no branches there (its only branches are in States A and B) and was not aware that the woman had moved or died. If the bank’s assertions are correct, how should the court rule?

(A) For the bank, because it only has branches in State A and State B.
(B) For the bank, because its contacts with the state are insufficient to support personal jurisdiction.
(C) Against the bank, because the woman maintained her State B account after she moved to State C.
(D) Against the bank, because it is foreseeable that a customer would move to another jurisdiction.

A

(B) For the bank, because its contacts with the state are insufficient to support personal jurisdiction.

150
Q

A man was stabbed in a bar during an argument with another customer. He was taken to the hospital where he was given a drug to help alleviate the pain. The drug was defective and he sustained further injuries. The man sued a company that he believed manufactured the drug in federal court. Diversity jurisdiction existed. The company was not the only manufacturer of the drug, and under applicable tort law the man had the burden to show that the company caused his injuries. In its answer, the company denied that it manufactured the drug that caused the injuries. After extensive discovery, the company filed a motion for summary judgment and pointed out, correctly, that the discovery record was devoid of evidence about who manufactured the drug in question.

Should the court grant the motion for summary judgment?

(A) No, because the company has not supported its motion with evidence that it did not manufacture the drug that caused the injury.
(B) No, because who manufactured the drug that caused the injuries is a question of fact for the jury.
(C) Yes, because the company denied in its answer that it manufactured the drug that caused the injuries.
(D) Yes, because the record shows that the man cannot sustain his burden of proof.

A

(D) Yes, because the record shows that the man cannot sustain his burden of proof.

151
Q

The plaintiff, a man from State A, was injured when the defendant, a citizen of State B, ran a red light in State A, causing $40,000 in damages. During an informal interview before the any suit was filed, the defendant told a newspaper reporter that the plaintiff was “a drunk who should not be allowed to drive a car, let alone practice medicine.” The plaintiff believed that he suffered $50,000 in damages to his reputation as a result of the newspaper article. The plaintiff filed suit against the defendant in a State A federal court. The defendant objects, claiming that the federal court does not have jurisdiction to hear the case. May the federal court hear the case?

(A) Yes, because defamation actions are properly heard in federal court.
(B) Yes, because the elements for diversity jurisdiction are met.
(C) No, because even though the plaintiff’s damages arose out of the same case or controversy, both claims arose from violations of state law.
(D) No, because the plaintiff cannot aggregate unrelated claims.

A

(B) Yes, because the elements for diversity jurisdiction are met.

Discussion of correct answer: The plaintiff can bring his case in federal court if there is a federal question or diversity of citizenship. Here, there is no federal question involved. However, the plaintiff and the defendant are from different states, and the amount in controversy exceeds $75,000. The amount in controversy can be aggregated even if the claims are unrelated, as long as they are asserted by the same plaintiff against the same defendant. Here, both claims are brought by the same plaintiff against the same defendant. Therefore, aggregation is proper.

152
Q

An employee was a citizen of State A. She worked for a corporation, incorporated and doing business from three locations in State B and two locations and its main office in State A. The corporation had contracts to do work for a local government agency in State B. The employee created a flyer with statements and graphics to let people know that the corporation was a bad employer. The employee made copies of the flyer, and distributed the flyers near the corporation’s three locations in State B. The corporation subsequently lost its government contract and sued the employee in state court in State B for defamation and $10,000 in damages due to lost buyer traffic and $60,000 in lost revenue due to advertisers dropping the corporation’s ads, and $10,000 in physical damage caused by graffiti spray painted on the company’s locations in State B, which repeated accusations made in the employee’s flyers. The employee filed an answer asserting that creating and distributing the flyers falls under the protection of the First Amendment. She filed a notice of removal to federal district court in State B.

Is removal proper?

A Yes, because the employee’s defense is based on the First Amendment, and so the federal district court has federal-question jurisdiction.
B Yes, because there is diversity of citizenship, the federal district court has diversity jurisdiction.
C No, because the amount in controversy is insufficient to invoke the federal district court’s diversity jurisdiction.
D No, because the employee’s First Amendment defense is insufficient to invoke federal-question jurisdiction.

A

The correct answer is: No, because the employee’s First Amendment defense is insufficient to invoke federal-question jurisdiction.

Discussion of correct answer: In order to remove an action to federal court, an original claim filed by the plaintiff must fall within the subject-matter jurisdiction of the federal court. The plaintiff did not file a claim based on a federal question. There is also no federal court jurisdiction based on diversity of citizenship. The corporation has its principle place of business in the same state of which the employee is a citizen.

153
Q

A large fast food company with many franchises throughout the state has been accused of systematically forcing many of its employees to work overtime without receiving compensation, in violation of federal law. A group of these employees hired a large law firm to sue the company and the franchises accused of violating this federal law. The firm does its best to send out notices to all of the possible members of the class action, but the population is large, and they cannot reach everyone.

Which of the following is true regarding personal jurisdiction in this class action lawsuit?

(A) The court must have personal jurisdiction over the absent members of the plaintiff class, who must receive adequate notice of the pendency of the action.
(B) The court must have personal jurisdiction over the absent members of the plaintiff class, who must be afforded the opportunity to opt out of the class.
(C) The court must have personal jurisdiction over the absent members of the plaintiff classes, who are not required to receive notice or an opportunity to opt out of the class.
(D) The court must have personal jurisdiction over each of the defendants named in the action.

A

(D) The court must have personal jurisdiction over each of the defendants named in the action.

154
Q

In March 2016, the plaintiff filed a claim against a supermarket chain for injuries that resulted from a slip and fall. As part of discovery, the plaintiff has requested: “All communications and reports submitted to the supermarket’s insurance company prior January 2016.” The supermarket disclosed all the copies of statements and accident reports that were submitted to the insurance company but refused to disclose any other documents asserting that the documents were protected work product.

If the plaintiff decides to file a motion to compel, what is the plaintiff’s best argument for requiring disclosure of the documents?

(A) The documents are not work product, because they were not communications with the supermarket’s or insurance company’s attorney.
(B) The documents are not work product, because they were not prepared in anticipation of litigation.
(C) The documents are not work product, because they do not contain an attorney’s mental impressions, conclusions, opinions, or legal theories.
(D) The documents are not work product, because the plaintiff cannot obtain the substantial equivalent of the documents without undue hardship.

A

(B) The documents are not work product, because they were not prepared in anticipation of litigation.

155
Q

A local politician and his girlfriend attended a local art show. The art show featured a world-renowned photographer who was famous for taking pictures of people he saw on his daily walks around the city. The artist would then paint slogans on the photos. As the politician and his girlfriend walked around the show, they discovered that the artist had taken a picture of them while standing in front of a foreign consulate. Below the couple was painted the slogan “Commie Pigs.” The girlfriend became so upset that she fainted, injuring herself when she hit the ground. The politician and girlfriend sued the artist in federal court. The politician was a citizen of State C, the girlfriend was a citizen of State B, and the artist was a citizen of State A. The politician’s claim was for libel in the amount of $70,000. The girlfriend’s claim was for $20,000 under state laws for intentional infliction of emotional distress. The artist moved to dismiss the claims for lack of subject-matter jurisdiction.

Which of the following is most accurate?

A The court should deny the motion, because a federal court must hear the case as it involves defamation of a public figure.
B The court should deny the motion, because there is complete diversity and the couple can aggregate their claims against the artist.
C The court should grant the motion, because the federal court does not have jurisdiction over either claim.
D The court should grant the motion as to the girlfriend’s claim, because she is not a public figure.

A

The correct answer is: The court should grant the motion, because the federal court does not have jurisdiction over either claim.

Discussion of correct answer: A federal court must have subject-matter jurisdiction over each cause of action alleged in the complaint. While it is true that the politician is a public figure, there must be proper subject-matter jurisdiction for a federal court to hear the case. There is no federal question presented simply by way of the politician being a public figure. While there is complete diversity, these two individuals cannot aggregate their claims. Multiple plaintiffs cannot aggregate their claims to meet the amount-controversy requirement.

156
Q

A man, a citizen of State A, sued a furniture company in State A federal court after the man was hurt when a chair he was sitting on broke in half. The man claimed over $1 million in damages. The chair was a huge hit and sold over a million chairs in State A. The furniture company was incorporated in State B and had all of its offices in State B. However, because of the great number of lawsuits based on a design defect of the chair, the company was on the brink of insolvency. Therefore, the company argued it could not afford to defend itself in State A.

Does the State A federal court have personal jurisdiction over the company?

A No, because the company is on the brink of insolvency.
B No, because the company only sold goods in State A.
C Yes, because the company marketed and sold the chairs in State A.
D Yes, because the man was injured by the chair in State A.

A

The correct answer is: No, because the company is on the brink of insolvency.

Discussion of correct answer: Based on the contacts found between the forum state and the defendant, the court must find that exercising personal jurisdiction over the defendant would not offend traditional notions of fair play and substantial justice. This is a balancing test used by the courts, weighing the following factors: (1) the plaintiff’s interest in trying the case in the forum state and the availability of other forums; (2) the defendant’s burden in trying the case in the forum state as opposed to the defendant’s state of residence; and (3) the forum state’s interest in the case. Here, because the company is on the brink of insolvency, a court will likely find the defendant’s burden in defending in State A (when its offices are in State B) so high as to offend traditional notions of fair play and substantial justice.

157
Q

A student, a citizen of State A, had five alcoholic drinks before deciding to drive home. He crashed into a ladder that was left in the middle of the road after it fell off a truck driven and owned by a contractor. The accident occurred in State A. The contractor was a citizen of State B. The student sustained over $80,000 in medical bills. The student sued the contractor in federal court in State A. The student wishes to have a jury trial. The federal court, following federal rules, grants a jury trial in such a case, while State A law would not. Is the student entitled to a trial by jury?

(A) No, because the state law is outcome determinative.
(B) No, because forum shopping is against public policy.
(C) Yes, because the federal court has proper jurisdiction.
(D) Yes, because the federal rule must be followed to determine whether there is a right to a jury trial.

A

(D) Yes, because the federal rule must be followed to determine whether there is a right to a jury trial.

158
Q

An individual is a citizen of State A. He works for an employer incorporated and doing business from five locations in State A. The individual decides that he and the other workers at the employer need a union. Collective action and discussing the terms and conditions of employment are covered by the NLRA, a federal statute. The statute also contains protections for employers. The individual starts talking to other workers about how little they are paid, and that they face bad working conditions, and, generally, the need for collective action by the employees of the employer. The employer sues the individual in state court for defamation for talking to the other workers about their working conditions. The individual files an answer, stating that his actions are protected by the NLRA, and that he is therefore not liable for defamation. The individual also files a notice to remove the action to federal court, claiming federal-question jurisdiction.

Is removal proper?

A Yes, because the individual’s main defense is based on a federal statute.
B Yes, because the subject has been completely preempted by federal law.
C No, because a defense based on a federal statute is insufficient grounds for removal.
D No, because the complaint is based on a state law defamation claim.

A

The correct answer is: Yes, because the subject has been completely preempted by federal law.

Discussion of correct answer: The employer’s attempt to avoid federal court by invoking a state law question (defamation) is insufficient to defeat the underlying federal-question subject-matter jurisdiction upon which the employer’s claim is based. The question of defamation is seated in whether the individual was acting in accordance with the National Labor Relations Act (NLRA), a federal statute that preempts the field of labor union formation. Therefore, removal is proper in this instance.

159
Q

The plaintiff sued the defendant in federal court for damages under the federal antitrust laws. The plaintiff served a request for the production of documents. The information requested was relevant to the plaintiff’s claim and was not privileged. It included certain electronically stored information. The defendant did not object to the request but was unable to produce the electronically stored information because the defendant’s electronic information system had destroyed the information prior to the defendant becoming aware of any possible lawsuit, as part of its routine operation. The plaintiff filed a motion for sanctions, and the court declined to sanction the defendant for failing to produce the documents.

Was the court correct to decline to sanction the defendant?

(A) Yes, because the routine destruction of the documents occurred before the duty to preserve arose.
(B) Yes, because the destruction of the documents resulted from the routine operation of an electronic information system.
(C) No, because the documents were relevant to the plaintiff’s claim and were not privileged.
(D) No, because the defendant’s electronic information system routinely destroyed documents.

A

(A) Yes, because the routine destruction of the documents occurred before the duty to preserve arose.

160
Q

A woman filed suit against a man for strict liability because she was injured by a lion that the man owned. At the close of the woman’s evidence, the woman moved for judgment as a matter of law because discovery before trial showed that the man admitted that he owned the lion that caused the injury to the woman. Specifically, she argued the man would be unable to provide any defense when he presented his evidence in response.

Should the court grant the woman’s motion?

(A) Yes, because discovery showed the man had clearly owned the lion.
(B) Yes, because there was no possibility the man could raise a valid defense.
(C) No, because the validity of a potential defense is up to the court.
(D) No, because the woman is the plaintiff in the case.

A

(D) No, because the woman is the plaintiff in the case.

161
Q

A corporation that was incorporated in State A and had its principal place of business in State B filed suit against a college student who had hacked into the corporation’s computer and stolen valuable trade secrets that she posted on the internet. The suit was brought in federal court and alleged damages of more than $75,000. On the day the case was filed, the student resided in State B, but had the intent to return home to State C when she graduated the following spring. This intent to return to State C had been true ever since she moved from State C to State B for school. State C was the only other place she had ever lived prior to going to college. The day after the suit was filed, the student secured employment in State B following graduation. Therefore, she decided to stay in State B indefinitely.

If the student moves to dismiss the case for lack of subject-matter jurisdiction, should the court grant the motion?

(A) Yes, because the student resided in State B at the time the case was filed.
(B) Yes, because the student became a citizen of State B while the case was pending.
(C) No, because the student was a student from State C on the date the case was filed.
(D) No, because the student was a citizen of State C when the case was filed.

A

(D) No, because the student was a citizen of State C when the case was filed.

162
Q

A man had a flower shop built in State A and was planning to move there in three days, when the flower shop was to be open for business. He also bought a house in State A. At the time, he was living in his house in State B, which was to be sold next month, while he finished the preparations for the move. As the man was about to drive a moving truck with the last of his belongings from his house in State B, he was hit by a car driven by a State B resident. The man sued for damages in federal court, claiming diversity jurisdiction based on the fact that there was over $75,000 in damages, the car driver was a State B resident, and the man was now a State A resident because he owned a home there, had most of his things there, and intended to move there permanently in the next few days.

Can the man bring his suit in federal court?

A No, because the man still had property in State B.
B No, because the man was not physically in State A.
C Yes, because the man intends to remain in State A permanently.
D Yes, because there is complete diversity of citizenship.

A

The correct answer is: No, because the man was not physically in State A.

Discussion of correct answer: In order for diversity to exist, the dispute must involve citizens of different states. A party will be considered the citizen of the state of his or her domicile. Domicile requires the person to have both physical presence in the state and the intent to remain in that state indefinitely. Importantly, the person’s intent has to coincide with the person’s physical presence in the state. Consequently, even though the man has the intent to stay in State A permanently, has most of his stuff there, and has a house there, he has not yet established a State A domicile and retains his State B domicile.

163
Q

A musician, a citizen of State A, believes that a commercial that a corporation made to promote its services contained music that the musician had created. The corporation is incorporated in State B and has its headquarters in State A. He sued the corporation for copyright infringement in state court, claiming $30,000 in damages. A federal act regulates all copyright issues involving music. The corporation filed a motion for removal to federal court.

Should the court grant the author’s motion?

(A) Yes, because commercials are regulated by federal law.
(B) Yes, because the claim is based on a federal question.
(C) No, because the amount in controversy was $30,000.
(D) No, because there is not complete diversity of citizenship.

A

(B) Yes, because the claim is based on a federal question.

164
Q

A man and his wife were injured by a truck driven by an employee of S Corporation, a company that manufactures widgets. The accident occurred in State C, where the man and his wife were living at the time. S Corporation is incorporated in State D, maintains its principal manufacturing plant in State C, and is controlled from its offices in State F. After the accident, the man and his wife moved to State F, where they planned to retire. After moving, they contacted a lawyer, who filed suit against S Corporation in the appropriate U.S. District Court where the man and his wife now lived, alleging diversity jurisdiction. The suit alleged that each plaintiff had suffered personal injuries in excess of $75,000. S Corporation moved to dismiss the case for lack of subject-matter jurisdiction.

Should S Corporation’s motion be granted by the district court?

A No, because diversity subject-matter jurisdiction exists.
B No, because S Corporation is a citizen of State D only.
C Yes, because S Corporation’s headquarters are in State F.
D Yes, because the plaintiffs were citizens of State C when they were injured.

A

The correct answer is: Yes, because S Corporation’s headquarters are in State F.

Discussion of correct answer: The requirements for diversity subject-matter jurisdiction are not satisfied based on these facts, because the parties are not diverse. The plaintiffs are citizens of State F, and S Corporation is a citizen of State D (where it is incorporated) and State F (where the officers control the corporation). Because State F is “in common” amongst the parties, there is not complete diversity. Note that it is irrelevant that the corporation’s principal manufacturing plant is in State C. Although the diversity statute links corporate citizenship to the corporation’s principal place of business, the Supreme Court has interpreted that phrase to mean the corporation’s “nerve center”–i.e., the place from where the corporation is controlled.

165
Q

A man, who lived in the Northern District of State B, sued a doctor, who lived in the Northern District of State C, and a lawyer, who lived in the Northern District of State A, for an accident that occurred in Canada. The man filed his suit in the Northern District of State A based on diversity jurisdiction. All parties agree to personal and subject-matter jurisdiction.

Is venue proper in this action?

(A) Yes, because the venue for foreign accidents is State A federal court.
(B) Yes, because the lawyer lives in the Northern District of State A.
(C) No, because the defendants are not residents of the same state.
(D) No, because a substantial portion of events occurred in Canada.

A

(B) Yes, because the lawyer lives in the Northern District of State A.

166
Q

A corporation sued a defendant for breach of contract because the defendant reneged on a contract to sell the corporation the mineral rights to his property. The corporation brought the action in federal court. The defendant considered answering the complaint instead of filing a motion to dismiss.

If the defendant answers the complaint, which of the following will he not be able to include in his answer?

(A) A motion for a more definite statement.
(B) The defense of improper service.
(C) The defense of improper venue.
(D)The defense of failure to state a claim upon which relief can be granted.

A

(A) A motion for a more definite statement.

167
Q

A musician, a citizen of State A, believes that a commercial that a corporation made to promote its services contained music that the musician had created. The corporation is incorporated in State B and has its headquarters in State A. He sued the corporation for copyright infringement in state court, claiming $30,000 in damages. A federal act regulates all copyright issues involving music. The corporation filed a motion for removal to federal court.

Should the court grant the author’s motion?

(A) Yes, because commercials are regulated by federal law.
(B) Yes, because the claim is based on a federal question.
(C) No, because the amount in controversy was $30,000.
(D) No, because there is not complete diversity of citizenship.

A

(B) Yes, because the claim is based on a federal question.

168
Q

Paul, a citizen of Idaho, and Greg, a citizen of Oregon, sued Darryl, a citizen of Montana, for breach of contract occurring in Montana. The action was filed in the U.S. District Court in Oregon, and Darryl was improperly served. Upon learning of the suit, Darryl made a successful motion to transfer the case to the U.S. District Court in Montana. Which of the following is true?

(A) The federal court properly transferred the action to Montana.
(B) The federal court in Oregon should have dismissed the action, because venue in the U.S. District Court in Oregon was improper.
(C) The federal court in Oregon should have denied Darryl’s motion to transfer the case, because he was improperly served.
(D) The federal court improperly transferred the action to Montana.

A

(A) The federal court properly transferred the action to Montana.

169
Q

A man, a citizen of State A, bought a life insurance policy from an insurance company located in State C. The company only sold policies to customers in State A, State C, State D, and State E and had no customers or business interests in any other states.

Several years later, the man moved to State B. The man retained the policy, and the insurance company continued to accept premiums and keep the policy in effect although it had no other customers in State B. The man then sued the company in State B court for fraud and other violations regarding his policy.

Does the State B court have personal jurisdiction over the company?

(A) Yes, because the policy remained in effect, and so sufficient minimum contacts exist.
(B) Yes, because the company consented to jurisdiction in State B by accepting premium payments.
(C) No, because the company did not have sufficient minimum contacts with State B.
(D) No, because it was not reasonably foreseeable that the company would be sued in State B.

A

(A) Yes, because the policy remained in effect, and so sufficient minimum contacts exist.

170
Q

A plaintiff wanted to sue two defendants for damages stemming from the defendants’ reprinting of her novel in violation of copyright laws. The defendants reprinted and sold the novel in the Northern District of Illinois. One defendant resides in the Eastern District of Michigan, while the other resides in the Western District of Illinois. The plaintiff resides in the Northern District of Iowa. Where would venue be proper for this action?

(A) The Northern District of Illinois.
(B) The Western District of Illinois or the Eastern District of Michigan.
(C) The Northern District of Illinois, the Western District of Illinois, or the Eastern District of Michigan.
(D) The Northern District of Illinois, the Western District of Illinois, the Northern District of Iowa, or the Eastern District of Michigan.

A

(A) The Northern District of Illinois.

171
Q

A pharmaceutical company wishes to sue an insurance company on a breach of contract claim, alleging damages in the amount of $1 million. The pharmaceutical company has its headquarters in State A and is incorporated in State B, while the insurance company has its headquarters in State C and is also incorporated in State B.

May the pharmaceutical company bring the lawsuit in federal court?

(A) No, because both it and the insurance company are incorporated in the same state.
(B) No, because state courts have exclusive jurisdiction over breach of contract claims.
(C) Yes, because the companies’ respective principle places of business are in different states.
(D) Yes, because the case presents a federal question.

A

(A) No, because both it and the insurance company are incorporated in the same state.

172
Q

A fashion designer and a pattern maker entered into a services contract. The fashion designer’s place of business was located in Somerset County, which is in State A. The contract contained a provision stating that any dispute arising out of the contract would be litigated either in a state court located in Somerset County, or in the federal district court located in Somerset County. The pattern maker was a resident of State B, and the services that were the subject of the contract were to be performed in State B. A few months after the contract was signed, the fashion designer sued the pattern maker, alleging breach of the contract and citing damages in the amount of $150,000. The fashion designer brought the suit in the federal district court in Somerset County. The pattern maker argued that the venue was improper, and that the suit should have been brought in State B.

Will the court accept the pattern maker’s argument?

(A) Yes, because the services that were the subject matter of the contract were to be performed in State B.
(B) Yes, because a forum selection clause cannot override federal law regarding venue.
(C) No, because the pattern maker consented to the forum.
(D) No, because one of the litigants is located in the forum state.

A

(A) Yes, because the services that were the subject matter of the contract were to be performed in State B.

173
Q

A woman sued a musician in federal court because the musician negligently struck her with his van while she was riding a bike. The musician told his attorney that he had been drinking alcohol while performing at a concert and was “really pretty drunk when he drove home that night.” The woman sent interrogatories to the musician asking whether he had been drinking before the accident, how much he had been drinking, and what exactly he had been drinking.

Does the musician have to answer the interrogatories?

(A) Yes, because the interrogatories are relevant to the woman’s claim.
(B) Yes, because the interrogatories involve questions of fact.
(C) No, because the musician’s statements are protected by attorney-client privilege.
(D) No, because the musician’s statements are protected by the work-product rule.

A

(B) Yes, because the interrogatories involve questions of fact.

174
Q

A supermarket was accused by a civil rights group of discriminatory hiring and promotion practices. The situation escalated when there was a violent incident at the protest rally being held by the civil rights group outside of the supermarket. The supermarket commenced an action against the civil rights group in the appropriate U.S. District Court, claiming $95,000 in damages as a consequence of the civil rights group’s conduct and violent actions of some of its members during the protest in violation of the National Labor Relations Act and applicable state law. The supermarket was a State A corporation, and the civil rights group was an unincorporated association which has members who were domiciled in every state. The civil rights group answered by denying the supermarket’s allegations and filing a $90,000 counter-claim of conversion asserting that some of the supermarket’s security guards burned down the headquarters of the civil rights group.

If the supermarket moves to dismiss civil rights group’s counterclaim for lack of subject-matter jurisdiction, is the court likely to grant the motion?

(A) Yes, because the supermarket has asserted a state cause of action.
(B) Yes, because diversity does not exist.
(C) No, if the civil rights group’s counterclaim is compulsory in nature.
(D) No, if the civil rights group’s counterclaim is permissive in nature.

A

(C) No, if the civil rights group’s counterclaim is compulsory in nature.

175
Q

A corporation that was incorporated in State A and had its principal place of business in State B filed suit against a college student who had hacked into the corporation’s computer and stolen valuable trade secrets that she posted on the internet. The suit was brought in federal court and alleged damages of more than $75,000. On the day the case was filed, the student resided in State B, but had the intent to return home to State C when she graduated the following spring. This intent to return to State C had been true ever since she moved from State C to State B for school. State C was the only other place she had ever lived prior to going to college. The day after the suit was filed, the student secured employment in State B following graduation. Therefore, she decided to stay in State B indefinitely.

If the student moves to dismiss the case for lack of subject-matter jurisdiction, should the court grant the motion?

(A) Yes, because the student resided in State B at the time the case was filed.
(B) Yes, because the student became a citizen of State B while the case was pending.
(C) No, because the student was a student from State C on the date the case was filed.
(D) No, because the student was a citizen of State C when the case was filed.

A

(D) No, because the student was a citizen of State C when the case was filed.

176
Q

A State A plaintiff sued two State B defendants in State A. The plaintiff’s claim against each defendant exceeded $100,000. Ten days after being served, the defendants were served notice that the plaintiff had added a third defendant, who was from State A. All three defendants agreed to remove the case, and did so within the next 10 days. The plaintiff successfully challenged the removal in federal court and the case was remanded back to State A court. Forty days later, the plaintiff settled her case against the State A defendant.

At this point, what recourse do the remaining defendants have?

A They can remove the case immediately, because they have 30 days from when the grounds for removal become apparent.
B They can remove the case immediately, because this is a diversity case and the date for removal was within one year of when the suit was filed.
C They cannot remove, because they are time-barred.
D They cannot remove, because although they are not time-barred, removal can only occur once.

A

The correct answer is: They can remove the case immediately, because they have 30 days from when the grounds for removal become apparent.

Discussion of correct answer: The defendants can remove after the third defendant has been dropped from the suit. A party can remove within 30 days of when the grounds for removal become apparent, and in diversity cases, removal must always be within one year of the date when the suit was filed. Once the plaintiff settled with the nondiverse defendant, the remaining defendants now had grounds for removal, and had 30 days in which to do so.

177
Q

A woman worked briefly as a hairdresser in State A. While there, she routinely advised her clients, most of whom she did not see on a regular basis, that they try her friend’s homemade hair conditioner. She was only able to sell a few containers of the hair conditioner, and soon became disenchanted with the business of hair care and personal grooming in general. She quit her job and moved to State B, where she met a man who worked as an independent IT consultant. She married the man and she joined his business. Almost all of their clients were in State B, with a small but significant percentage being in the Northern District of State C.

Meanwhile, one of the few customers who had purchased the hair conditioner developed a slow-forming disease that he believed was attributable to the chemicals in the product. The customer filed a product liability lawsuit against the woman in state court in the Southern District of State C, where he now lived. The customer called the woman and asked her to meet him in State C to discuss a huge business opportunity for her, but when she arrived, he instead merely served her with process. The woman had no other contacts with State C other than the portion of her business regularly conducted there and intermittently being physically present in State C to conduct that business. The woman filed a motion to dismiss the customer’s lawsuit, arguing that the court did not have personal jurisdiction over her because service of process was improper.

Is she likely to prevail?

A No, because she has systematic and continuous contacts with State C.
B No, because the woman’s contacts with State C were limited to the Northern District.
C Yes, because she was served while physically present in State C.
D Yes, because her presence in State C was obtained through fraud.

A

The correct answer is: Yes, because her presence in State C was obtained through fraud.

Discussion of correct answer: Ordinarily, if a defendant is physically present in a jurisdiction and is served with process while there, the court’s exercise of personal jurisdiction over that defendant constitutional. It does not matter if the defendant’s presence at the time is temporary or unrelated to the lawsuit. However, the defendant’s presence must be voluntary and the plaintiff cannot have coaxed the defendant into the state under false pretenses. Had the customer served the woman while she happened to be in the state on one of her own business trips, this would have been enough to provide the court with personal jurisdiction over her. Because the customer instead deceived the woman into entering the state in order to physically serve her, such service is improper and the court will not have personal jurisdiction over the woman.

178
Q

A man and woman attended a local art show. The art show featured a world-renowned photographer who was famous for taking pictures of people he saw on his daily walk around the city. The artist would then paint slogans on the photos. As the man and woman walked around the show, they discovered that the artist had taken a picture of them while they were dressed up in matching American flag shirts. Below the couple was painted the slogan “War Pigs.” The woman became so upset that she fainted, injuring herself when she hit the ground. The man and woman joined as plaintiffs under Rule 20 and sued the artist in federal court. The man was a citizen of Connecticut, the woman was a citizen of Rhode Island, and the artist was a citizen of New York. The man’s claim was for $10,000 under the Federal Misuse of Likeness Act, which created a civil cause of action when one person uses another person’s image for monetary gain. The woman’s claim was for $20,000 under state laws for intentional infliction of emotional distress. The artist moved to dismiss the woman’s claim for lack of subject-matter jurisdiction.

How should the court rule?

A The court should grant the motion, because the amount in controversy was only $20,000.
B The court should grant the motion, because the woman’s claim was based on state law.
C The court should deny the motion, because the woman’s claim arose from the transaction or occurrence underlying the man’s claim.
D The court should deny the motion, because the woman’s claim, in effect, alleged a violation of a federal law.

A

The correct answer is: The court should deny the motion, because the woman’s claim arose from the transaction or occurrence underlying the man’s claim.

Discussion of correct answer: A federal court must have subject-matter jurisdiction over each cause of action alleged in the complaint. Nevertheless, claims that do not provide subject-matter jurisdiction may be combined with appropriate claims through the court’s supplemental jurisdiction, provided that they arise out of the same case or controversy [28 U.S.C. Sec. 1367]. Claims arise out of the same case or controversy when they derive from: (1) the same transaction or occurrence; or (2) a common nucleus of operative facts. Here, the man was suing under a federal statute; thus, his claim was a federal question properly heard in federal court. The woman was suing based on a state law claim. While the woman’s Rhode Island citizenship was diverse from the artist’s New York citizenship, the amount in controversy was less than $75,000. Normally, her suit would be inappropriate for federal court. However, both the man’s and the woman’s claims arose out of a common nucleus of operative facts, because they were both based on the artist’s use of their likenesses in his art show. The fact that the man and the woman suffered different injuries does not change the analysis.

179
Q

A man is a citizen of State A and filed a defamation action in federal court against a reporter and an editor, both of whom were citizens of State B. The claims against the two defendants, the reporter and the editor, arose from the same events. Plaintiff alleged that the amount in controversy with respect to each defendant was $100,000. State law caps damages in this type of case against defendants like the editor at $25,000. The editor filed a motion to dismiss for lack of subject matter jurisdiction.

Should the court grant the motion to dismiss for lack of subject matter jurisdiction?

(A) Yes, because there is an insufficient amount in controversy between the man and the editor.
(B) Yes, because the reporter and the editor are from the same state.
(C) No, because there is supplemental jurisdiction over the claim against the editor.
(D) No, because the Court looks to the man’s allegations to determine the amount in controversy.

A

(A) Yes, because there is an insufficient amount in controversy between the man and the editor.

180
Q

A pharmaceutical company sued two engineers for patent infringement, a federal law claim. The pharmaceutical company brought the lawsuit in State C, in the judicial district in which one of the engineers lived. The other engineer also resided in State C. The alleged patent infringement took place in State B, which is the state in which the pharmaceutical company is incorporated and has its principal place of business. The pharmaceutical company alleges that it had suffered damages in the amount of $60,000 arising from the alleged infringement. One of the engineers moved to dismiss the lawsuit on the basis of improper venue.

Will the engineer’s motion to dismiss be successful?

(A) No, because the plaintiff is not a resident of the same state as the defendants.
(B) No, because both engineers are domiciled in the forum state.
(C) Yes, because the alleged infringement took place outside of the forum.
(D) Yes, because the amount in controversy is only $60,000.

A

(B) No, because both engineers are domiciled in the forum state.

181
Q

A State A publicist ordered 100 ukuleles from a State B ukulele manufacturer with the intention of placing the ukuleles in gift bags given out to VIP guests at a music industry awards event. The ukuleles were to be imprinted with the name and image of a State A singer who was famous for her ukulele playing, and were intended to promote the singer’s new tour. The manufacturer put the image of an entirely different singer on the ukuleles, however, and as a result the publicist failed to collect $65,000 in publicity fees. The publicist had also paid $12,000 to the manufacturer upfront for the ukuleles for which she now had no use. The publicist filed suit against the manufacturer in state court in State A, alleging $65,000 in damages relating to her lost payment from the publicity contract and $10,000 in attorney fees, which is the likely amount she could recover under the applicable law. The publicist failed to include the $12,000 in damages for the unusable ukuleles, although it is evident that she would be able to pursue that claim as well. The manufacturer has removed the case to federal court, and the publicist then sought to remand.

If the federal court applies the majority approach to determining amount in controversy, and if the manufacturer has presented no evidence that the publicist’s damages are less than what she claims, who will prevail?

A The publicist, because the amount in controversy does not include the recovery of attorney fees.
B The publicist, because the court will determine the amount in controversy based on the amount sought by the plaintiff.
C The manufacturer, because the plaintiff’s recovery, if any, will likely include some portion of the $12,000 paid for the ukuleles.
D The manufacturer, because the amount in controversy will include the likely recovery of attorney fees.

A

The correct answer is: The publicist, because the court will determine the amount in controversy based on the amount sought by the plaintiff.

Discussion of correct answer: The majority approach that federal courts take in determining the amount in controversy is to look at what damages are sought by the plaintiff, as opposed to the value of the consequences that might result from the litigation. Here, the plaintiff seeks $65,000 in damages from the loss of the payment from the singer, as well as $10,000 in attorney fees (which may be included in the amount in controversy). This equals $75,000, but the amount in controversy must be over $75,000, so the amount in controversy will not be met here.

182
Q

An Illinois man wanted to gain personal jurisdiction over a California businessman who had been quite evasive in avoiding service in Illinois. He called the businessman’s office, lied as to who he was, and asked to speak to the businessman. His secretary said he was not there, mentioned that he was set to board a flight, but said that he had a short layover in Chicago and asked if the businessman could call him back around 3:00 P.M. CST. The man then went online and found the businessman’s flight.

The man immediately dispatched a process server to the airport with a copy of the complaint. As the businessman was disembarking from the plane, the process server handed him a copy of the complaint and said, “Welcome to Chicago. You’ve just been served.”

Would an Illinois court have jurisdiction over the businessman?

A No, because the tactics used shock the conscience.
B No, because the man employed fraudulent tactics to serve the businessman.
C Yes, because the businessman was served while in the jurisdiction.
D Yes, because the businessman was being evasive, which caused unfair prejudice to the plaintiff.

A

The correct answer is: Yes, because the businessman was served while in the jurisdiction.

Discussion of correct answer: While a defendant cannot be fraudulently lured into a jurisdiction in order to be served, there is no rule that prevents the plaintiff from being disingenuous.

183
Q

The plaintiff filed a complaint under the interpleader statute in the United States District Court for the District of State A. The plaintiff was a citizen of State A. The defendant was a citizen of State B and had no purposeful contacts with State A. The plaintiff correctly named the defendant as a claimant of the property that was the subject of the interpleader. The defendant was personally served with process at his home in State B. The defendant filed a motion to dismiss for lack of personal jurisdiction and lack of venue.

Should the court grant the defendant’s motion to dismiss for lack of personal jurisdiction?

A Yes, because the defendant did not have any purposeful contacts with State A.
B Yes, because the defendant was not personally served in State A.
C No, because the defendant waived the defense of lack of personal jurisdiction by joining with it a motion to dismiss for lack of venue.
D No, because the court had personal jurisdiction over the defendant.

A

The correct answer is: No, because the court had personal jurisdiction over the defendant.

Discussion of correct answer: By statute, federal courts are empowered to issue process and have it served nationwide, in any district where a claimant might be found, in a statutory interpleader case [28 U.S.C. Sec. 2361]. Under Federal Rule of Civil Procedure 4(k)(1)(c), service of the summons and complaint establishes personal jurisdiction over a defendant when the service is authorized by such a federal statute. The defendant would not have any constitutional objection to personal jurisdiction because the defendant resided in State B and thereby obviously had sufficient purposeful contacts with the United States.

184
Q

Able Company, incorporated and doing business in State A, did business with a Baker Corporation, incorporated and doing business in State B, for several years. Baker started having financial difficulties and failed to pay Able for product that Baker received. On March 1, 2012, Baker filed for bankruptcy. One year later, the bankruptcy court discharged Baker’s debts. On April 1, 2013, Able sued Baker in federal district court for the unpaid product costs, amounting to $175,000. Able served the summons and complaint on Baker three weeks later. Baker subsequently filed an answer, denying all claims in the complaint. After the initial court conference, where the parties and the court set a discovery schedule, but prior to commencement of discovery, Baker filed a motion to dismiss the complaint because all debts had been discharged in bankruptcy, including the debt to Able. Will the court grant Baker’s motion to dismiss?

(A) Yes, because the debt has been discharged and Able is not entitled to collect on the debt.
(B) Yes, because Baker asserted bankruptcy discharge in a motion to dismiss before discovery began.
(C) No, because Baker did not assert bankruptcy discharge in its answer.
(D) No, because Able will be prejudiced if it is not allowed to prosecute its case.

A

(C) No, because Baker did not assert bankruptcy discharge in its answer.

185
Q

A businessman in West Virginia sold his lawn care business to an Ohio corporation that was attempting to build a nationwide conglomerate of lawn care service providers. As part of the transaction, the corporation agreed to a service contract with the businessman by which he would continue to provide maintenance on the lawn care equipment he sold to the corporation for two years. The businessman believed he was fraudulently induced into accepting a lower contract price than he should have been given, and filed suit against the corporation in federal district court based on diversity jurisdiction. He alleged $90,000 in damages. The corporation filed an answer arguing only one affirmative defense, which was that the businessman had failed to state a claim upon which relief had been granted. The case went to trial, and the corporation’s lawyers elicited testimony indicating that the actual amount of damages was, at most, only $50,000. The businessman agreed that his alleged damages did not exceed $50,000.

If the corporation now seeks to have the case dismissed for lack of subject-matter jurisdiction, will it prevail?

(A) No, because the corporation waived the defense by failing to raise it in the answer or on a motion to dismiss.
(B) No, because the amount in controversy need only be met at the time the case is filed.
(C) Yes, because the court may dismiss a case for lack of subject-matter jurisdiction at any time.
(D) Yes, because the corporation included an affirmative defense in its answer.

A

(C) Yes, because the court may dismiss a case for lack of subject-matter jurisdiction at any time.

186
Q

A man is friends with an individual, both of whom live in State A. The individual got into a bar fight with a gang member, who was visiting from State B, and was badly hurt. The next day, the man was in his car, and had to make a sudden stop. The gang member was driving behind the man, and hit the man’s car. The man subsequently sued the gang member in federal district court for $175,000 in damages. The man served the summons and complaint on the gang member. The gang member filed an answer, denying the man’s claims. During discovery, the gang member found out that the man’s hospital bills only amounted to $2,000, and that the damage to his car was only $500. The gang member also found out that the man was friends with the individual, who was in the process of pressing criminal charges against the gang member. The gang member filed a motion for sanctions with the court, claiming that the man had grossly overstated his damages in order to harass the gang member, and to get him into federal court.

Will the federal district court grant the gang member’s motion for sanctions?

A Yes, because the court can impose sanctions on the man sua sponte.
B Yes, because the man grossly overstated his damages.
C No, because the man is entitled to prove his good faith basis for the complaint.
D No, because the gang member did not first serve the motion for sanctions on the man.

A

The correct answer is: No, because the gang member did not first serve the motion for sanctions on the man.

Discussion of correct answer: Sanctions are a serious step. Therefore, safeguards exist to allow a party that has acted in a sanctionable manner to correct that action. To that end, a party must serve a motion for sanctions on the opposing party first, which allows that party the opportunity to correct the action and avoid sanctions. The gang member did not serve the motion on the man prior to filing the motion. The court may ultimately grant the motion, but not before the gang member follows the proper procedure. Therefore, the court will not now, in this context, grant the gang member’s motion for sanctions. If the court chooses to grant sanctions sua sponte, it will be, as the term illustrates, of its own accord, and not as a result of the gang member’s improperly filed motion.

187
Q
In a federal court product liability trial, a class of consumers who used an antibiotic sued the manufacturer of the antibiotic for failing to warn them that the product could make users drowsy. The plaintiffs were people who had not known of the potential drowsiness and who had been at fault in vehicle or machinery accidents because they were drowsy. A jury was empaneled. Several weeks later, while the jury had a day off from hearing the case, a juror stopped by her office at an advertising agency and discovered that the manufacturer was currently a client of the advertising agency. The juror told the judge. On learning of this, the manufacturer's counsel moved to remove the juror, but the judge denied the motion because the juror stated that she had been unaware of the business relationship and that she could be impartial despite it. A week has passed since the jury delivered a verdict of $5 million against the manufacturer. The manufacturer believes that the juror was not impartial.

What would be the most appropriate motion for the manufacturer to file with the trial court?

(A) 	A motion for remittitur.
(B) 	A motion for a new trial.
(C) 	A motion for relief from judgment.
(D) 	A motion for injunctive relief.
A

(B) A motion for a new trial.

188
Q

A driver, a citizen of State A, traveled to State B. While in State B, the driver rented an RV from a rental company incorporated and doing business in State B. The driver drove the RV in State B and State C. While in State C, the accelerator on the RV stuck, and the driver got into a single vehicle accident, sustaining major injuries and totaling the RV. The driver was taken to a hospital in State C. When the driver got out of the hospital, he sued the company in federal district court in State B for renting a faulty RV to him, and asked for $300,000 in medical expenses and pain and suffering. The company counterclaimed for $70,000, the cost of the RV that was totaled in the accident. The driver filed a motion to dismiss the company’s counterclaim.

Will the federal district court grant the driver’s motion to dismiss?

A Yes, because the counterclaim is permissive, and therefore the court does not have jurisdiction over the company’s claim against the driver.
B Yes, because the claim arises out of an occurrence that happened in State C, so the case was filed in the incorrect venue.
C No, because the counterclaim is compulsory, and therefore the court has supplemental jurisdiction over the company’s claim against the driver.
D No, because the driver is a citizen of State A who chose to bring the lawsuit in federal district court located in State B, and therefore the company, a citizen of State B, may bring its claim for the cost of the RV.

A

The correct answer is: No, because the counterclaim is compulsory, and therefore the court has supplemental jurisdiction over the company’s claim against the driver.

Discussion of correct answer: The company’s claim against the driver is specifically for damage caused to the RV as a result of the accident, which is an arising incident of the original lawsuit. If the company failed to raise the claim, it could lose the right to ever raise it in the future. Therefore, the counterclaim is compulsory. Compulsory counterclaims are reachable through supplemental jurisdiction, even if there is not an independent basis for federal jurisdiction for the counterclaim.

189
Q

A plaintiff filed an action to quiet title to a three-foot-wide strip of property between his property and his neighbor’s property. Although the strip of land actually belonged to the neighbor, the plaintiff had been using it as part of his garden for over a decade. Several months ago, the plaintiff planted a privacy hedge on the strip of land which resulted in the current dispute.
The plaintiff filed his cause of action in a neighboring county where his full-time residence was located. The process server attempted to deliver a copy of the summons and complaint to the neighbor. Because the neighbor was not home, the process server left the summons and complaint with the neighbor’s 13-year-old son. The neighbor filed an answer to the plaintiff’s complaint, alleging that the plaintiff had permission to use the strip of land for a garden and, therefore, the plaintiff could not have acquired title to the property. Two months after filing the answer, the neighbor filed a Fed. R. Civ. P. 12 motion to dismiss the plaintiff’s complaint.
What is the most appropriate basis for the Rule 12 motion?

(A) Lack of personal jurisdiction, because the neighbor does not reside in the county in which the plaintiff filed the claim.
(B) Insufficient service of process, because the process server left the summons and complaint with the neighbor’s minor son.
(C) Improper venue, because the property at issue is not located in the county in which the plaintiff filed the claim.
(D) Failure to state a claim upon which relief can be granted, because the plaintiff had permission to use the property as a garden.

A

(D) Failure to state a claim upon which relief can be granted, because the plaintiff had permission to use the property as a garden.

190
Q

A supermarket was accused by a civil rights group of discriminatory hiring and promotion practices. The situation escalated when there was a violent incident at the protest rally being held by the civil rights group outside of the supermarket. The supermarket commenced an action against the civil rights group in the appropriate U.S. District Court, claiming $95,000 in damages as a consequence of the civil rights group’s conduct and violent actions of some of its members during the protest in violation of the National Labor Relations Act and applicable state law. The supermarket was a State A corporation, and the civil rights group was an unincorporated association which has members who were domiciled in every state. The civil rights group answered by denying the supermarket’s allegations and filing a $90,000 counter-claim of conversion asserting that some of the supermarket’s security guards burned down the headquarters of the civil rights group.

If the supermarket moves to dismiss civil rights group’s counterclaim for lack of subject-matter jurisdiction, is the court likely to grant the motion?

(A) Yes, because the supermarket has asserted a state cause of action.
(B) Yes, because diversity does not exist.
(C) No, if the civil rights group’s counterclaim is compulsory in nature.
(D) No, if the civil rights group’s counterclaim is permissive in nature.

A

(C) No, if the civil rights group’s counterclaim is compulsory in nature.

191
Q

A State A songwriter submitted several of her compositions to a website service that offered to host the material to television and film producers who were looking to purchase material for their programming. The songwriter was never contacted through the service, so she was surprised to see three of her songs on two different TV shows: the show “Cactus Dogs” had used two of her songs, and the show “Biltmore Days” had used one of her songs. She assessed her damages to be $30,000 per song, and sued the producers of Cactus Dogs as well as the producers of Biltmore Days as co-defendants in federal district based on her federal copyright claims against all of them.

If the co-defendant producers argue that the court does not have subject-matter jurisdiction, will they prevail?

A Yes, because the plaintiff must have at least one claim that meets the amount-in-controversy requirement.
B Yes, because the plaintiff’s claims against the co-defendants are not common and undivided.
C No, because the plaintiff may aggregate her claims to meet the amount-in-controversy.
D No, because the plaintiff’s claims arise under federal law.

A

The correct answer is: No, because the plaintiff’s claims arise under federal law.

Discussion of correct answer: This is a bit of a trick question in that it was designed to make you think about amount-in-controversy requirements, but recall that amount-in-controversy only applies to diversity jurisdiction cases. The plaintiff here is alleging a federal copyright claim, so her complaint meets the requirements for federal question jurisdiction, and no amount-in-controversy is necessary.

192
Q

Former employees of a local college planned to file an action in state court against the college’s board of trustees. On April 15, they met with the university president, who was a member of the board, and told him that they were planning to sue. They claimed that they had been unlawfully discharged due to their political beliefs. The plaintiffs filed their complaint on April 25. Three of the 12 defendants were served with process on the same day. The nine others were served on May 19. The first three who were served and eight of the nine others then petitioned for removal to federal court on May 25.

The one defendant who had been served on May 19 but who did not join in the petition for removal on May 25 was the university president. He had been travelling out of state. The president joined in the petition for removal on June 18. April has 30 days, and May has 31 days. This court follows the majority rule regarding such petitions, and did not make any adjustments for weekends.

For the case to be properly removable to federal court, when must the president have joined the petition for removal?

A May 25, 30 days from the time the first defendant was served.
B June 18, 30 days from the time the president was served.
C May 15, 30 days from the time the president knew about the lawsuit.
D May 29, 15 days from the time the president was served, because he was served in the second round of service.

A

The correct answer is: June 18, 30 days from the time the president was served.

Discussion of correct answer: In an action based on federal-question jurisdiction, the action is removable without regard to the citizenship or residence of the parties. However, all defendants must consent or join in removal for removal to be proper [28 U.S.C. Sec. 1446(b)]. There is a split of authority amongst federal courts as to when the time for removal begins to run and when such time expires in multiple defendant cases. A slight majority follows the intermediate, or McKinney, rule that, in multiple defendant cases, all defendants served within 30 days of the first defendant being served must join in an existing removal action or file their own. However, individual defendants have 30 days from the time they are served with process or with a complaint to join in an otherwise valid removal petition [McKinney v. Bd. of Trustees of Md. Cmty. Coll., 955 F.2d 924 (4th Cir. 1992)]. Here, because the president was served on May 19, he would have had until June 18 to join the petition.

193
Q

A company sold a motorcycle to a man in State A. The man decided after he had bought the motorcycle that he would travel on the motorcycle across the country to State B. The man was seriously injured in an accident when the motorcycle had its front tire blow out while he was riding on a highway in State B. The man sued the company in federal court in state B. Diversity jurisdiction existed. The company filed a timely motion to dismiss for lack of personal jurisdiction. The long-arm statute in State B extended the reach of its courts to the limits of the United States Constitution. The accident in State B was the only contact that the company had with State B.

Should the court grant the motion to dismiss for lack of personal jurisdiction?

(A) Yes, because the man purchased the motorcycle in State A.
(B) Yes, because the company did not have any purposeful contact with State B.
(C) No, because company should have foreseen that a mobile product could end up in State B.
(D) No, because the company caused injury in State B.

A

(B) Yes, because the company did not have any purposeful contact with State B.

194
Q

A web designer filed a federal patent claim against a technology company, alleging that the company infringed on her patent after she shared information with the company while seeking employment at the company. The web designer also asserted a claim arising under State X employment law that provides a private cause of action for any misappropriation of information by an employer from a potential employee during the job seeking process. Her state law claim was based on the same facts as her patent claim. The web designer sought $25,000 in damages based on the patent claim, and $5,000 based on the state employment law claim. The web designer filed her action in State X state court, and the technology company sought to remove the action to federal court.

If the web designer is a resident of State X and the technology company is a resident of State Y, can a federal court exercise jurisdiction over the web designer’s claims?

A No, the court may not exercise jurisdiction over either claim.
B Yes, but only the employment law claim.
C Yes, but only the patent claim.
D Yes, the court may exercise jurisdiction over both claims.

A

The correct answer is: Yes, the court may exercise jurisdiction over both claims.

Discussion of correct answer: The federal court has federal question jurisdiction over the federal patent claim as it arises under federal law. The court may also exercise supplemental jurisdiction over the state law claim as it arises under the same occurrence as the federal law claim, in that both relate to the alleged infringement on her work by the technology company. Note that the court could decline to exercise supplemental jurisdiction but the question here asked whether the court could exercise jurisdiction.

195
Q

D-Corp, a Michigan corporation, sells its automobiles through independent dealers. D-Corp’s principal place of business is in Detroit. Each dealer has a separate dealership agreement, which they respectively executed with D-Corp at the time they were appointed to be dealers. Their agreements entitle them to sell all automobiles manufactured by D-Corp in their areas. This year, D-Corp has produced a new sports car that is a radical departure from its previous automobiles. D-Corp decided that it will not sell this car to its existing dealers, but will instead enter into agreements with new dealers to handle the new sports car line. Twelve dealers in California jointly filed actions for breach of contract, seeking an injunction against D-Corp in the appropriate U.S. District Court. D-Corp moved to dismiss the case for lack of subject-matter jurisdiction.

How should the court rule on D-Corp’s motion?

A The court should grant D-Corp’s motion to dismiss, because a party seeking an injunction cannot satisfy the amount-in-controversy requirement of diversity jurisdiction.
B The court should grant D-Corp’s motion to dismiss, because the plaintiffs are not diverse.
C The court should deny D-Corp’s motion to dismiss, because a party seeking an injunction does not need to satisfy the amount-in-controversy requirement of diversity jurisdiction.
D The court should deny D-Corp’s motion to dismiss, because the diversity requirement has been met in this case.

A

The correct answer is: The court should deny D-Corp’s motion to dismiss, because the diversity requirement has been met in this case.

Discussion of correct answer: The fact that two or more plaintiffs are citizens of the same state, or two or more defendants are citizens of the same state, does not destroy diversity. Diversity requires only that no plaintiff be a citizen of the same state as any defendant. As such, the case will not be dismissed for lack of subject-matter jurisdiction.

196
Q

A buyer, a resident of State A, bought a car from a car dealer located and incorporated in State B. The buyer found the car and the car dealer through the car dealer’s website, which he found and reviewed on his computer at home in State A. The buyer drove the car for three months. At that time, the buyer got into a car accident because he could not stop for a red light. Upon examination of the car, a mechanic discovered that the brake pads were not properly installed on two of the tires, which caused the brakes to fail. That same day, a truck owned by a corporation incorporated and doing business in State B, swerved in front of the buyer when he was walking across the street from the mechanic’s shop, and the buyer broke his leg. The initial brake pad installation took place at the car dealer. The car dealer’s advertising reached State A and State B consumers. Fifteen percent of its cars were sold to residents of State A. The buyer sued the car dealer in federal district court located in State A for $70,000, the amount of damages sustained due to the car accident. The buyer also sued the corporation for $10,000 for damages. The car dealer filed a motion to dismiss.

Will the federal district court grant the car dealer’s motion to dismiss?

A Yes, because the federal district court in State A does not have personal jurisdiction over the car dealer.
B Yes, because the amount of damages does not reach the threshold required for federal court jurisdiction in diversity cases.
C No, because the federal district court in State A does have personal jurisdiction over the car dealer.
D No, because the amount of damages reaches the threshold required for federal court jurisdiction in diversity cases.

A

The correct answer is: Yes, because the amount of damages does not reach the threshold required for federal court jurisdiction in diversity cases.

Discussion of correct answer: Although altogether the buyer’s lawsuit seeks damages in the amount of $80,000, that total is split between two parties and two claims that may not be aggregated. Therefore, neither claim reaches the threshold minimum amount in controversy required for a court case to be heard in federal court pursuant to diversity jurisdiction.

197
Q

A man recently released from prison as a result of DNA evidence showing his innocence filed a Section 1983 action against a police officer and a prosecutor for violating his constitutional rights in connection with arresting and prosecuting him years earlier. The man filed the action in a state court located in the Southern District of State X, where the man lived and where he was arrested originally. The prosecutor resided in the Northern District of State X, and the police officer resided in State Y. The police officer wanted to remove to federal court, but the prosecutor was against the idea. Two months later, the man named the county as an additional defendant. The county’s lawyers made a strong case to the prosecutor that the case should be removed to federal court, and all three defendants agreed that the case should be removed.

Can the defendants remove the case, and, if so, to what court?

A The defendants may not remove the case.
B The defendants can remove the case to the Southern District of State X.
C The defendants can remove the case to any district court in State X.
D The defendants can remove the case to any district court in State X or State Y.

A

The correct answer is: The defendants can remove the case to the Southern District of State X.

Discussion of correct answer: The defendants are able to remove the case here, because whenever a new defendant is added to the case the defendants have the option of removing the case, even if the previous group of defendants had chosen not to remove the case. The venue where the case can be removed to, however, is the district court where the state court is located, so the defendants may only remove to the Southern District of State X.

198
Q

A student sued a company that sold a defective weight-loss supplement in federal court for product liability. Diversity jurisdiction existed. In discovery, the student did not seek records of safety tests performed by the company on the product. The student lost the jury trial and did not appeal or file a motion for new trial. Six months after entry of the judgment on the jury’s verdict, the student discovered that the company had conducted tests of the product which showed that the product was unsafe. The student moved for relief from the judgment.

May the court grant the motion for relief from judgment?

(A) No, because the motion was not made within 28 days of the entry of judgment.
(B) No, because the student could have sought the test results in discovery.
(C) Yes, because the failure to produce the test results was misconduct by the company.
(D) Yes, because the new test results are newly discovered evidence.

A

(B) No, because the student could have sought the test results in discovery.

199
Q

A man was a resident of State A and wanted to sue a car manufacturer in federal court based on diversity jurisdiction. The car manufacturer was incorporated in State B, had its home office and factories in State C, and conducted business in many states, including State A.

May the man bring his suit in federal court?

A Yes, because the car manufacturer is engaged in interstate commerce.
B Yes, because there is complete diversity between the parties.
C No, because both the man and the car manufacturer are citizens of State A.
D No, because corporations have more than one principal place of business.

A

The correct answer is: Yes, because there is complete diversity between the parties.

Discussion of correct answer: A corporation is a citizen of both the state of its incorporation and the state where it has its principal place of business. A corporation can have only one principal place of business, usually found at the corporation’s “nerve center.” Here, that state is State C. Consequently, the car manufacturer is a citizen of State B, where it is incorporated, and State C, where its nerve center is. The fact it conducts business in other states does not change the analysis.

200
Q

Strong Corporation is in the business of providing exercise instruction and exercise facilities and equipment based on techniques developed by Joe Strong in the 1920s. Strong Corporation owns two registered trademarks, “The Strong Way,” which designated its exercise instruction services, and “Strong Studios,” which designated facilities for exercise and physical conditioning. Strong Corporation wanted to sue The Strong Institute for trademark infringement for naming its business The Strong Institute and for selling videos and other equipment that purported to teach the Strong method. The Strong Institute was incorporated in Delaware and had its principal place of business within the Central District of Pennsylvania. All of The Strong Institute’s studios are in Pennsylvania, and it does the majority of its business there. However, within the past six months, The Strong Institute started working on a plan to open studios in New York City. In furtherance of that plan, The Strong Institute had started advertising on New York City buses and sent targeted direct-mail brochures to New York City gyms and gym members. Strong Corporation wants to sue The Strong Institute in the Southern District of New York, where New York City is located and Strong Corporation resides. Is venue appropriate there?

(A) No, because the majority of the trademark infringement occurred in Pennsylvania.
(B) No, because The Strong Institute, the defendant, resides in the Central District of Pennsylvania.
(C) Yes, because Strong Corporation, the plaintiff, resides in the Southern District of New York.
(D) Yes, because The Strong Institute targeted New York City through its advertising.

A

(D) Yes, because The Strong Institute targeted New York City through its advertising.

Discussion of correct answer: For civil actions where jurisdiction is based in part on a federal question, such as trademark, the available venues are: (1) the district where any defendant resides, provided that all defendants are located in the same state; (2) the district where a substantial portion of events or omissions giving rise to the claim occurred; or (3) the district where any defendant can be found. Courts have found that if a defendant targets a particular district by advertising and actively pursuing efforts to market the offending product there, such actions may be regarded as constituting a substantial part of events giving rise to the claim. Because The Strong Institute targeted the Southern District of New York through advertising and direct mail, venue would be proper in that distric

201
Q

An employee who was fired plans to sue her former employer, claiming that the employer is liable for both wrongful termination under state law as well as violation of federal employment discrimination statutes, claiming total damages in the amount of $50,000. The employer and employee are citizens of the same state.

May the employee assert these claims together in federal district court?

A) No, because the amount in controversy does not exceed $75,000.
B) No, because the federal court lacks subject matter jurisdiction over a state law claim between two citizens of the same state.
C) Yes, because the federal court has federal question jurisdiction over the federal statutory claim and supplemental jurisdiction over the state law claim.
D) Yes, because federal courts have subject matter jurisdiction over all claims joined with federal statutory claims.

A

C) Yes, because the federal court has federal question jurisdiction over the federal statutory claim and supplemental jurisdiction over the state law claim.

202
Q

A sales representative working for a manufacturing company had an employment contract under which the representative agreed to work exclusively for the company for three years. The representative resides in State A. The manufacturing company is incorporated in State B and has its principal place of business in State C. Six months before expiration of the contract, the representative left the manufacturing company and went to work for a competitor of the company. The competitor is incorporated in State C and has its principal place of business in State A. The manufacturing company files an action in federal district court against the representative and the competitor, alleging breach of contract and tortious interference with contractual relations.

Does complete diversity exist among the parties?

A) Yes, because the representative is a citizen of State A, the manufacturing company is a citizen of State B, and the competitor is a citizen of State C.
B) Yes, because the representative is a citizen of State A, the manufacturing company is a citizen of State C, and the competitor is a citizen of State A.
C) No, because the manufacturing company and the competitor are both citizens of State C.
D) No, because the representative and the competitor are both citizens of State A.

A

C) No, because the manufacturing company and the competitor are both citizens of State C.

203
Q

Marcus hit Megan’s son Sammy with his car while she was watching. Megan sued Marcus on behalf of herself for negligent infliction of emotional distress. Marcus’s attorney filed an answer and began discovery. As part of discovery, Marcus’s attorney takes the deposition of Megan’s neighbor, who stated that she saw Megan in her backyard at the time of the accident, and thus she couldn’t possibly have seen the accident. All pleadings have been filed in the case and discovery has since closed. If there are no further triable issues of fact, Marcus should file a motion:

(A) for a more definite statement.
(B) to dismiss the action for failure to state a claim.
(C) for judgment on the pleadings.
(D) for summary judgment.

A

(D) for summary judgment.

204
Q

Peter, a politician, files suit for defamation against The Times, a local newspaper. At the conclusion of Peter’s case, counsel for The Times makes a motion for a directed verdict. The court will grant the motion if:

(A) the Times presents sufficient evidence that it is not liable for defamation.
(B) there are findings of fact and conclusions of law to support The Times’s motion.
(C) Peter has presented insufficient evidence to prove The Times is liable for defamation.
(D) Peter has not established that The Times is liable for defamation beyond a reasonable doubt.

A

(C) Peter has presented insufficient evidence to prove The Times is liable for defamation.